0% found this document useful (0 votes)
376 views

2014 2010 PDF

This document contains a 5th year final exam for 2014 that covers topics related to obstetrics and gynecology. It includes 33 multiple choice questions testing knowledge of conditions like gonorrhea, oligohydramnios, gestational diabetes, risk factors for preeclampsia, causes of bleeding after delivery, and complications of procedures like D&C. The questions cover anatomy, pathophysiology, diagnosis, management and treatment options for a wide range of obstetric and gynecologic conditions.

Uploaded by

ملك عيسى
Copyright
© © All Rights Reserved
We take content rights seriously. If you suspect this is your content, claim it here.
Available Formats
Download as PDF, TXT or read online on Scribd
0% found this document useful (0 votes)
376 views

2014 2010 PDF

This document contains a 5th year final exam for 2014 that covers topics related to obstetrics and gynecology. It includes 33 multiple choice questions testing knowledge of conditions like gonorrhea, oligohydramnios, gestational diabetes, risk factors for preeclampsia, causes of bleeding after delivery, and complications of procedures like D&C. The questions cover anatomy, pathophysiology, diagnosis, management and treatment options for a wide range of obstetric and gynecologic conditions.

Uploaded by

ملك عيسى
Copyright
© © All Rights Reserved
We take content rights seriously. If you suspect this is your content, claim it here.
Available Formats
Download as PDF, TXT or read online on Scribd
You are on page 1/ 339

1.

5th Year Final Exam 2014:


1. Gonorrhea can infect all the following except:
- (+) Nose
- Mouth
- Cervix
- Vagina
- Urethra
** N gonorrhoeae causes:
1. Urogenital infections in women in the form of Cervicitis [most common], Urethritis, PID
[salpingitis, endometritis, and/or tubo-ovarian abscess, and perihepatitis known as Fitz-Hugh-
Curtis syndrome] and Bartholinitis. It is also a less common cause of Vaginitis.
2. Urogenital infections in men in the form of Urethritis and Epididymitis or epididymo-
orchitis.
3. Extragenital infections in the form of Proctitis, Pharyngitis, Disseminated Gonococcal
infection [endocarditis, meningitis or gonococcal dermatitis-arthritis syndrome: arthralgia,
tenosynovitis, and painless non-pruritic (non-itchy) dermatitis], and Conjunctivits [Neonatal,
and adult].

2. All of the following can cause Oligohydramnios except:


- Potter Syndrome
- (+) Anencephaly
- Pulmonary Hypoplasia
**Polyhydramnios causes:
- Twin gestation with twin-to-twin transfusion syndrome (increased amniotic fluid in the
recipient twin and decreased amniotic fluid in the donor) or multiple gestations
- Fetal anomalies, including esophageal atresia (usually associated with a tracheoesophageal
fistula), tracheal agenesis, duodenal atresia, and other intestinal atresias
- CNS abnormalities and neuromuscular diseases that cause swallowing dysfunction
(anencephaly)
- Congenital cardiac-rhythm anomalies associated with hydrops, fetal-to-maternal hemorrhage,
and parvovirus infection
- Poorly controlled maternal diabetes mellitus (Oligohydramnios may also be seen if severe
vascular disease is present.)
3
- Chromosomal abnormalities, most commonly trisomy 21, followed by trisomy 18 and trisomy
13.
- Fetal akinesia syndrome with absence of swallowing
**Oligohydramnios causes:
- Fetal urinary tract anomalies, such as renal agenesis (Potter syndrome), polycystic kidneys, or
any urinary obstructive lesion (eg, posterior urethral valves)
- PROM [most common cause] and chronic leakage of the amniotic fluid
- Placental insufficiency, as seen in pregnancy-induced hypertension (PIH), maternal diabetes,
or postmaturity syndrome when the pregnancy extends beyond 42 weeks' gestation
- Maternal use of prostaglandin synthase inhibitors or ACE inhibitors
**Oligohydramnios adversely affects fetal lung development, resulting in pulmonary
hypoplasia that typically leads to death from severe respiratory insufficiency.

3. One of the following is X-lined:


- Huntington's Disease [AD]
- (+) G6PD
- Cystic fibrosis [AR]
- Achondroplasia [AD]

4. All of the following are true about the risk malignancy index (RMI) except:
- Higher score in postmenopausal compared to premenopausal
- Score of malignancy referral > 200
- Used to differentiate between malignant and benign ovarian tumors
- US rather MRI score is used
- Serum CA125 is important

**ALL of them seem CORRECT!


**RMI = U x M x CA125
- The ultrasound result is scored 1 point for each of the following characteristics: multilocular
cysts, solid areas, metastases, ascites and bilateral lesions. U = 0 (for an ultrasound score of 0),
U = 1 (for an ultrasound score of 1), U = 3 (for an ultrasound score of 2–5).
- The menopausal status is scored as 1 = pre-menopausal and 3 = post-menopausal
- The classification of 'post-menopausal' is a woman who has had no period for more than 1
year or a woman over 50 who has had a hysterectomy.
- Serum CA125 is measured in IU/ml and can vary between 0 and hundreds or even thousands
of units.

4
5. The least anticonvulsant to cause congenital anomalies during pregnancy:
- Valproate [Most teratogenic]
- Carbamazepine
- Sodium valproate
-(+) Phenothiazine
- Phynotoin

6. All of the following are true about the failure rate of contraception except:
- (+) implanon 2-4%

**Natural methods: 15% Failure Rate


Mini pills: 2-3%
Combined pills: 1-2%
Injectable progesterone (Depo-Provera): <1%
Implanon: 1%

7. Which one doesn't cause restricted growth:


- Advanced DM
- Connective tissue diseases
- Rh isoimmunization

**ALL seem correct!


**Causes of IUGR:
1. Fetal factors (reduced fetal growth potential): Chromosome defects (trisomy 18, triploidy),
Single gene defect (sickle's syndrome), Structural abnormalities (renal agensis), Infections
(CMV, Toxoplasmosis)
2. Maternal factors (reduced fetal growth support): Undernutrition (poverty, eating disorder),
Maternal hypoxia (altitude, CHD), Drugs (smoking, alcohol, cocain), Advanced diabetes, Uterine
malformation
3. Placental factors (reduced fetal growth support): Reduced uteroplacental perfusion
(inadequate trophoblast invasion, antiphospholipid syndrome, DM, sickle-cell disease, multiple
gestation), Reduced feto-placental perfusion (single umbilical artery, twin-twin transfusion
syndrome), Infarction, Abruption, placental tumors TTTs, Velamentous cord insertion

5
8. The presentation that's associated with abnormal lie:
- (+) Shoulder
- Face
- Breech
- Vertex
9. Which of the following is not an abnormal presentation:
- Face
- Breech
- Brow
- (+) Occipitoposterior [It means that presentation is vertex since Occipit is the dominator in
vertex presentation]

10. Not true about the management of septic abortion:


- (+) Immediate curettage [Should be after 12 hours after antibiotic therapy]

11. Which one of the following is not a risk factor for preeclampsia:
- (+) Smoking
- Multiparity

12. The tumor marker for granulosa cell tumor:


- (+) Inhibin
- AFP
- Ca125

13. Mismatch:
- Granulosa cell tumor >> Sex cord
- (+) Thecoma >> Germ cell [sex cord]
- Yolk sac tumor >> Germ cell
- Dysgerminoma >> Germ cell

14. One of the following doesn't cause post-coital bleeding:


- (+) Chlamydia trachomatos [causes intermesntrual bleeding]
- Cervical polyp
- Ectropion
- Cervical intraepithelial neoplasm

6
15. A patient complains of bleeding 12 days after vaginal delivery, one of the following is
true:
- (+) Put the patient on antibiotics [as endometritis is one of the causes of secondary PPH]

16. A patient complains of vaginal bleeding 10 days after vaginal delivery, what's the most
likely cause:
- (+) Retained products of conception [Most common cause of secondary PPH]

17. All of the following are true about regional anethesia in obstetrics except:
- Epidural needs higher dose than spinal
- Can cause meningitis
- Epidural can cause headache
- (+) Epidural can cause hypotension less than spinal
- Contraindicated in coagulopathy

18. All of the following can cause organic menorrhagia except:


-(+) PCO
- Copper IUCD
- Endometriosis
- Adenomyosis

19. One of the following is false about hypertension in pregnancy:


- Eclampsia doesn't necessarily need to have high blood pressure

**Eclampsia in the absence of hypertension with proteinuria has been demonstrated to occur
in 38% of cases reported in the United Kingdom. Similarly, hypertension was absent in 16% of
cases reviewed in the United States. [Medscape] but make sure of it!

20. One of the following is false about incomplete abortion:


- Diagnosed clinically

**Inevitable and incomplete abortions are diagnosed clinically, US is essential for the diagnosis
of threatened and missed abortions.

7
21. One of the following is correct about gestational DM:
- (+) Increase the incidence of shoulder dystocia
- Induce delivery at 37 weeks
- Most of them treated with insulin
- Always present after 24 gestational weeks

22. A patient with retinopathy and nephropathy may have all the following when getting
pregnant except:
- Placental insufficiency
- (+) Fetus weight more than 90th percentile [as it is advanced DM]
- Congenital anomalies
- Restricted growth

23. All of the following are associated with ectopic pregnancy except:
- Tachycardia
- Bathroom sign
- (+) Heavy vaginal bleeding
- Excited cervix

24. All of the following are part of early investigations of urinary incontinence except:
- (+) Urodynamic study
- Pelvic US
- Bladder diary
- History
- Urinalysis

25. Which of the following genital prolapsed is considered a true hernia:


- (+) Enterocele

26. One of the following is the latest sign of puberty:


- Pubic hair
- Axillary hair
- Breast enlargement
- (+) Menstruation [according to Tanner staging]
- Growth spurt

8
27. All are correct about ITP except:
- Increase the risk of fetal intracranial hemorrahge
- Delivery by CS has no improvement on fetal outcome
- Steroid initially used in the treatment
-(+) Platelet transfusion is common
- Treatment is needed only if platelet count is less than 50,000

28. Not a contraindication for induction of labor:


- Previous 1 classical CS
- (+) Vaginal prolapse [Previous repair of prolapse is a contraindication]
- Abnormal CTG
- Transverse lie

29. Progesterone-only contraceptives cause all of the following except:


- Weight gain
- Menstrual irregularity
- Acne
- Breast tenderness
- (+) Complex cysts [it causes simple cysts]

30. About fibroid, all are true except:


- More in blacks
- (+) COCPs are a risk factor
- Family history is a risk factor
- Long reproductive age before menopause [Early menarche is a risk factor]

31. Wrong about dysgerminoma:


- Bilateral in 10%
- Radiosensitive
- (+) Associated with hirsutism
- Common before the age of 30
32. All can cause fetal distress except:
- Supine position
- Lupus erythromatosus
- Abruption placenta
- Pethidine
- (+) Renal disease
9
33. Placental abruption all true except:
- PPROM can be a cause and a result
34. All are side effects of D&C except:
- (+) Endometriosis

**Complications of D&C: 1) uterine perforation which may lead to uterus rupture in the
subsequent pregnancy 2) cervical tear and excessive cervical dilatation which may lead to
cervical incompetence 3) infection which may lead to infertility and Asherman's syndrome 4)
excessive curettage which may lead to adenomyosis

35. 26 years old lost 16 kg of her weight in the last month, and she was doing heavy exercise,
you need to do all these investigations except:
- TFT
- Prolactin leve
-(+) bHCG
- Brain CT scan

36. Incomplete abortion underwent E&C, few months later presented with amenorrhea, the
most likely diagnosis:
- (+) Asherman's syndrome [It could be! Depending on the other options that couldn't be
remembered]
- PID

37. Placental abruption, you do all these investigations except:


- KFT
- (+) Urinalysis and culture
- CBC
- 24 hour urine collection [as is can be a consequence of PET]

38. All about benign GTN are true except:


- (+) Blood group has a role in the scoring
- Passage of the vesicles is diagnostic
- Suction and curettage are the TOC for benign GTN
- Induction of delivery is contraindicated

10
39. One of the following is not a normal complaint in pregnancy:
- (+) Itching
- Heartburn
- Abdominal pain
- Leg cramp
40. About menopause:
- Mean age 51
- Smoking lead to earlier menopause
- Premenopausal hysterectomised patients also enter menopause at 51
- Radiotherapy for breast can lead to earlier menopause
- Hot flashes are common in the transition and postmenopausal period
- HRT are contraindicated in women with fibroid

**Wallahi ALL of them seem correct!!

41. Goes with benign ovarian mass:


- Unilateral
- Adherent

42. Favors the diagnosis of placenta previa:


- Transverse lie
- Soft and non-tender abdomen
- Alive baby

**I think this question was supposed to be with except

43. All are causes of uterine atony except:


- Precipitous labor
- Multiple pregnancy
- (+) Oligohydraminous

44. Not a cause of infertility in endometriosis:


- Tubal obstruction
- Sperm motility will be affected
- Infrequent coitus
- (+) Hyperprolactinemia

11
45. All are correct about CVS disease in pregnancy except:
- (+) Usually CS is preferable (something like that)
- Mitral stenosis is the most common lesion in rheumatic heart disease
- Rhemutic heart disease is the most common in pregnancy
- Correction of anemia can help the patient

**The indication for cesarean section is based solely upon obstetric indications because vaginal
delivery is generally preferred in mothers with most CVDs. Elective cesarean section has been
reported to increase the risk of hemorrhage, thrombosis, and infection. [Medscape]
**In the past, Rheumatic heart disease was the most common form of cardiac disease in
pregnant women, it still predominates in devloping coutries and in immirgrant populations in
the US. Congenital heart disease is now the most common of heart disease complicating
pregnancy in the US, in part because advances in the treatment of CHDs have made it possible
for more affected children to reach adulthood and attempt pregnancy. [UpToDate]

46. Not part of management of fetal distress:


- Put the patient on her side
- O2 mask
- IV fluid
- (+) Oxytocin infusion

47. One of the following has the least teratogenic effect:


- Valporate
- Valporic
-(+) Phenytoin
- Phenobarbital
- Carbamazipen

48. One of the following is false:


- Female with epilepsy has higher chance of fetal malformation
- Increase the demise outcome of delivery
- (+) Lactation is contraindicated
- Phenobarbital is the drug of choice in the first trimester

**Most experts believe that taking AEDs doesn't generally contraindicate breast feeding, as
probable benefits outweigh risks. [UpToDate]

12
49. 22 years old female sexullay active presented with vaginal discharge and deep
dysparunia, gonorrehia culture is negative, the most likely cause is:
- Gonorrehia
- (+) Chlamydia
- Bacterial vaginosis
- Candidal infection

51. Not a cause of Fetal Growth Restriction:


- Advanced Diabetes
- Inadequate maternal nutrition
- PET
- Collagen disorders
- Microvascular disease

** ALL of them seem correct!!

52. About ovarian tumor one is wrong:


- (+) Most common histological is muscinous [most common is serous]

53. About postmenopausal bleeding, all true except:


- (+) Always pathological
- Estrogen can be the cause and the treatment
- Most common cause is atrophic vaginitis
- Always take biopsy

54. Time needed for uterus to become pelvic organ:


- (+) 2 weeks
- 4 weeks
- 6 weeks
**I think the question is asking about postpartum period.

55. Ruptured Tubal pregnancy, one is true:


- Pain can be upper abdominal [?? - it could be in rare cases!!]
- Coincides with vaginal bleeding

13
- Colicky pain
- (+) Always unilateral

56. All can be used in the treatment of DUB except:


- Progesterone only pills
- OCPs
- Mefenamic acid
- Tranexamic acid [Antifibrinolytic]
- IUCD

**ALL of them can be used!!

57. One is false about normal pregnancy:


- Breech presentation at 28 weeks [May mean that breech presentation is most common at 28
weeks]
- Fundus is usually first felt at 20 weeks
- (+) At 36 weeks the fundus is at level of umbilicus [it should be higher]

58. All are signs of ruptured ectopic except:


- Shoulder tip and cervical pain
- (+) Massive vaginal bleeding [??]
- Bathroom sign
- (+) Dysurea
- Vasomotor symptoms

59. One drug is teratogenic:


- Diazepam
- (+) ACEI
- Baby aspirin

60. Gonorrhea infection doesn't cause:


- Conjunctivitis
- Skin discoloration [Gonorrhea can cause dermatitis in the form of pustular or vesicopustular
usually, but hemorrhages macules, papules, bullae, or nodules may rarely occur. Also it can
rarely cuase erythema multiforme or nodosum. - UpToDate]
- Painful swollen testicles

14
61. All are risk factors for fibroid except:
- Nulliparity
- African
- (+) OCP
- Increase reproductive years

62. One is mismatched:


- OCP: 1%
- IUCD: 2%
- (+) Implanon: 3-5%

63. In Placenta previa, the main fatal complication is:


- (+) Prematurity
- IUGR
- Anemia

64. Inhibin is a tumor marker of:


- (+) Granulosa cell tumor
- Fibroma
- Dysgerminoma
- Sinus tumor

65. Not a step in the expectant management of 32-week placenta previa:


- Watch for bleeding
- Correct anemia
- (+) Tocolytic agent
- Steroids
- US

66. First sign of DIC:


- (+)Thrombocytopenia
- Bleeding from the cannula site
- Hematuria
-Non-clotting blood

15
67. Not in biophysical profile:
- Breathing
- Fine movement
- Gross movement
- Amount of liquor
- (+) Doppler fetal monitor
68. One is not part of fetal biophysical profile:
- AFI
- Fetal heart rate
- Gross movement
- Breathing tone
- (+) Umbilical doppler ultrasound
69. Woman presented 12 days post partum with vaginal bleeding and lower abdominal pain,
what is included in the management:
- (+) IV antibiotics
70. Neisseria gonorrhea is:
- (+) Bacteria
- Virus
- Protozoa
- Fungus
71. Ritodrine:
- Has comparable effectiveness to atosiban
- (+) Has an effect mainly on beta-1 receptors
- Beta agonist
- Usually causes tachycardia
- Not used anymore as first line because of its side effects
72. All are recommended treatment of endometriosis, except:
- Continuous COCP
- (+) Laser
- GnRH agonists
73. About bacterial vaginosis, which is false?
- (+) Usually symptomatic
- Fishy odor
- Increase in pH is one point of Amsel criteria
- Should be treated in pregnancy to prevent complications

16
Obstetrics and Gynecology Final Exam 2014

Note: The exam was two forms, 100 questions, 2 hours. This is From 1.

1) All of the following are benefits of ANC,


except:
A. Early PET detection
B. Prevents Anemia
C. Prevents Prematurity
D. Prevents Congenital anomalies

Explaination :

What are the complications of pregnancy which can be prevented or


minimized by good antenatal care??

1) Anemia

2) UTI

3) Pregnancy-induced hypertension: you can’t prevent it completely, but


you can reduce its effects and complications.

4) Preterm labour and delivery: you can’t really prevent preterm labour,
since it could be due to human error of calculation of the EDD (Expected
Delivery Date).

5) IUGR (Intrauterine Growth Restriction): Again if we had any chronic


complications and we managed to treat them we can minimize fetal risks
of IUGR.

6) STDs (Sexually-Transmitted diseases).

7) Rh Isoimmunization.

8) Fetal Macrosomia (If it was due to non-constitutional causes (like due to


diabetes) , we can reduce it).

9) Hypoxia and fetal death from post-term birth, again by knowing exact
EDD.
10) Breech presentation at term: Since if the patient doesn’t do regular
Antenatal care, she could be coming to you suddenly with breech
presentation and preterm labour and have many complications. While if
you already know about it you could easily bring her in at 37th week and
do her an external cephalic version and turn baby to a cephalic
presentation. (ANC LECTURE NOTES – DR NAEL)

2) Comparing abortion to ectopic, only one of the following is true:


A. Shedding of the decidua can be mistaken for products of
conception
B. Endometrial curette can rule out
ectopic
C. Abortion causes more pain
D. More vaginal bleeding in ectopic

Explaination :
Ectopic pregnancy usually presented with abdominal pain 99% of patients
(central or unilateral 35%, and could be generalized 45%, or shoulder tip
25%, continuous constant pain in nature, then vaginal bleeding.
Abortion will usually start with vaginal bleeding then abdominal pain.

3) A pregnant lady, 7 week gestation, presented with mild vaginal


bleeding and severe abdominal pain. In which condition of the following
methotrexate can be used:
A. B-hCG level 3500, no intrauterine gestational sac is seen by Abdominal
US
B. B-hCG level 2500, no intrauterine gestational sac is seen by
transvaginal US
C. Adnexal mass with positive heart beat

Explaination :

And as we know that the minimum value for β-hCG to detect pregnancy
by transvaginal ultrasound is 1500 mIU/ml (this value differ between
institutions). Ectopic pregnancy rarely present before this value, so a
patient with serum β-hCG of 1500 and you do a transvaginal ultrasound
and you don’t find a gestational sac, then you must think of an ectopic
pregnancy. (ECTOPIC PREGNANCY LECT NOTES)

4) In the management of infertility, All of the following are true except:


A. Unexplained in 30%
B. Hypothalamic pituitary dysfunction is the cause in PCOS
C. Management should be directed more toward females than males.
D. Male factor is the cause in 40% of cases
5) Which of the following is an advantage of myomectomy
over hysterectomy in the management of fibroids?
A. Spares reproductive function
B. Less bleeding
C. Less infection
E. Spares hormonal function

Explaination :
Complication of hysterectomy :
1. Heavy bleeding (1%-3%)
2. Infection (4%-10%)
3. Early menopause resulting from hormonal change

6) A patient with secondary amenorrhea who is being


investigated infertility, which of the following is not an important
investigation? A. FSH
B. Serum testosterone
C. Serum androstenedione
D. Urinary hCG
F. Prolactin

Explaination :
Routine investigation for infertility for female :

- Day 2 hormonal profile (FSH, LH, prolactin, TSH, testosterone, estradiol).


If PCOS is suspected, you might also order DHEAS level.

7) All are true regarding Rh alloimmunization, except:


A. If a sensitized Rh –ve woman, undergoes amniocentesis, she
should receive anti-D after the procedure
B. Negative KB test for Rh –ve woman, she doesn’t need anti­D
prophylaxis
C. External cephalic version is an indication for prophylaxis
D. More likely to occur if mother and fetus are ABO compatible

Explaination :
Rh isoimmunisation usually happen when there is a break in mother-
fetal placental barrier such as amniocentesis, abortion, external
cephalic version etc

8) The main maternal complication for severe placental abruption


is: A. IUFD
B. Real tubular necrosis C.
Renal cortical necrosis
D. Hemorrhagic shock
E. Amniotic fluid embolism

Explaination :
Because maternal shock is a sign of grade 3 placental abruption

9) Pregnant lady presented at 33 weeks, singleton, cephalic, with severe


placental abruption, all should be done except:
Urinary catheter
A. Vaginal examination
B. Give steroids

Explaination :
We need to rule out the position of the placenta first

10) 25-year old pregnant, presented with severe placental abruption,


CTG showed multiple decelarations. Your management is:
A. Left lateral position and O2 mask
B. Induction of labor
C. Emergent C/S

Explaination :

Assessment of fetal wellbeing, CTG. If the baby is distressed, aim for


immediate delivery. (ANC LECTURE NOTES)

11) Regarding placenta previa, what is false?


A. Speculum examination is contraindicated
B. Increased risk of IUGR
C. Increased risk of congenital anomalies

Explaination :
Vaginal exam is contraindicated in placenta previa

12) All are side effects of progesterone-only pills EXCEPT


A. Acne
B. Weight gain
C. Functional ovarian cyst
D. Inhibits lactation

Explaination :
POP are indicated in :
1. Women who’re contraindicated with estrogen
2. Women who are lactating
3. Women >35 years old who are smoking
4. Women with migraine
5. Women with endometriosis and sickle cell disease

13) All are disadvantages and/or contraindications for


Copper-releasing IUCD, except:
A. Reduces menstrual bleeding
B. Uterine malformation
C. History of PID
E. History of ectopic pregnancy

Explaination : IUCD will increase blood loss

14) All are modalities of tubal ligation except


A. Rings
B. Clips
C. Hysterectomy
D. Saplingectomy
E. Diathermy

15) Monozygotic twins with uncompensated AV shunts, all are true


except:
A. Donor twin will have polyhydramnios
B. Donor will have have mirocardia
C. Diagnosis when Hb difference is 50 g/L

Explaination :
Twins with AV shunt will have :

1. Intertwin weight differences greater than 20%


2. polyhydramnios in the large twin and cardiomegally and ascites
3. Oligohydramnios in the smaller one ( stuck twin) and absent bladder
and small and anemia
4. Hemoglobin differences greater than 5g/dl

(not really sure about this)

16) All are true regarding twin pregnancy, EXCEPT


A. Antenatal betamethasone decrease perinatal mortality
B. Cervical cerclage prevents preterm labor
C. Twenty percent are breech presentation
D. Second twin should be delivered within 30 minutes of the first

17) All are true regarding GTN except:


A. B-hCG is always higher than pregnancy levels
B. Almost always presents with large for date uterus
C. Methotrexate is indicated for low risk malignant GTN
D. Most common karyotype for complete mole is 46 XX
E. Usually presents with vaginal bleeding in the first trimester

- 50% only have large for date uterus


18) All are true regarding GTN except
A. Chemotherapy is indicated if B-hCG levels plateau for 2 weeks
B. Serum B-hCG is used for follow-up
C. Incomplete mole contains fetal
tissue
D. Snowstorm appearance is characteristic
E. Suction evacuation is the treatment of choice
All of them is absolutely correct except for A

19) All of the following HPV serotypes are associated with cervical cancer
except:
A. 16
B. 6
C. 31
D. 33
E. 18

- HPV 6 is low risk viruses associated with genital warts

20) All are true regarding cervical cancer EXCEPT:


A. Surgery is the cornerstone of treatment for all stages
B. Postcoital bleeding is a common presentation
C. Caused by persistent high-risk HPV infection

21) All are true regarding colposcopy, except:


A. 3-5% alcohol is used to shed debris
B. Mosaic pattern and punctation are abnormal
findings C. Colposcope is a stereotactic binocular
microscope D. Columnar epithelium appears reddish
and grape-like
- 3% acetic acid is used to shed debris

22) All of the following are true regarding Cervical smear EXCEPT
A. HPV testing is beneficial in ASCUS
B. HSIL should be referred for colposcopy
C. Squamous epithelium stains mahogany brown by Lugol’s iodine
D. Leukoplakia is a normal finding
D. Indicated at 21 years or 3 years after the onset of sexual activity

- Leukoplakia is an abnormal finding and is directly proportional to


severity of dysplasia (not really sure)

23) Regarding Tension-free Vaginal Tape (TVT), Only one of the following is
correct
A. Used for overflow
incontinence B. Success rate 65%
C. Should be done under local analgesia
D. Detrusor instability is a known complication
E. Dysfunctional voiding is a side effect in 15%

- TVT is a surgical procedure to treat


stress incontinence. It use a mesh
tape which is placed at the level of
mid-urethra that acts like a sling or
hammock to keep it in its position. It is
done under LA.
- Most common complication is :
1. Difficulty urinating after surgery
2. Injury to bladder or urethra

24) All of the following are true regarding urinary incontinence, except
A. Substracted cystometry is a useful investigation
b. Anticholinergics are useful for treatment of stress incontinence
C. 20-30% of adult female population are affected

- Anticholinergics is used in urge incontinence


25) All are symptoms of urinary incontinence,
except:
A. Frequency
B. Straining to void
C. Urge incontinence
D. Stress incontinence E.
Overflow incontinence

26) All of the following are taken in consideration in the choice


of endometriosis management, except:
A. Age
B. Desire to get pregnant
C. Size of endometrioma
E. Score of disease
severity E. Severity of
symptoms

- The management
of endometriosis
depend on some
considerations :
1. Certainty of
diagnosis
2. Severity of
symptoms
3. Extent of
disease
4. Future fertility
5. Age
6. Threat to
gastrointestinal
or urinary tract
or both

27) Regarding endometriosis, only one is correct


A. 65% affects the ovaries
B. Fixed retroverted uterus is a normal variant
C. Extent of disease is related to severity of symptoms

- There is no clear relationship between the stage of


endometriosis and frequency or severity of the symptoms
(Hacker and Moore’s)
- Common site of endometriosis in decreasing frequency :
1. Ovary
2. Cul-de-sac
3. Uterosacral ligament
4. Broad ligament
5. Fallopian tubes
6. Uterovesical folds
7. Round ligament
8. Vermiform appendix
9. Vagina
10. Rectovaginal septum

28) 35 year old, para 1 by C/S before 6 years, came complaining of


secondary dysmenorrhea, pelvic U/S was normal. What is the most likely
diagnosis?
A. Adenomyosis
B. Endometriosis
D. Submucous
fibroid
E. Psychosomatic
- Endometriosis
is occasionally
seen in
laparotomy
scars,
especially after
caesarean
deliveries or
myomectomy
(Hacker &
Moore’s)

29) About GnRH analogue, all are true except:


A. Used in precocious puberty
B. Optimal management of adenomyosis
C. It is anti-estrogenic

30) Pregnant (10 weeks), presented with severe lower


abdominal pain. On examination, uterus is consistent with 10
weeks, she was found to have open external os, the most likely
diagnosis is
A. Inevitable abortion
B. Missed
abortion
C.
Complete
abortion
D. Ectopic
pregnanc
y

31) All of the following could be a cause of recurrent abortions


except:
A. Toxoplasomosis
B. Thromboplilia
C. Submucus fibroid
D. PCOS
- Etiology of recurrent abortion :
a) General Maternal Factors
1. Infection with Mycoplasma, Listeria o Toxoplasma
2. Maternal smoking and alcohol consumption
3. Medical disorders (DM, hypothyroidism, SLE)
4.Increase maternal age
b) Local Maternal Factors
1. Uterine abnormalities (cervical incompetence)
2. Submucous fibroid
3. Intrauterine adhesions
c) Fetal Factors
1. Genetic abnormality
2 Chromosomal abnormalities

32) Regarding viral infections in pregnancy, All of the


following are true except:
A. Toxoplasmosis is a viral infection.
B. CMV transmission risk is 40%

33) A young patient presented with mild vaginal bleeding with


severe
abdominal pain, what is your first step in management:
A. Assess hemodynamic stability
B. Serum BhCG
C. Ultrasound

Reasons : For expectant management


34) All of the following can cause hemolytic disease of
newborn,
EXCEPT:
A. G6PD deficiency
B. Hexokinase deficiency
C. Sickle cell trait
D. Hereditary spherocytosis

Reasons : Sickle cell is a autosomal recessive. The patient will


not have a symptom if he’s / she’s only a carrier
35) All of the following are true about sickle cell disease in
pregnancy except:
A. Chronic anemia
B. Bone marrow erythroid hyperplasia
C. IUGR
D. Iron deficiency anemia

Reasons : Sickle cell dz can cause A,B, C. IDA is a common


anemia in pregnancy as the iron storage was utilized for the
baby. Sickle cell will not cause IDA

36) All of the following are caused by chromosomal


abnormalities
EXCEPT:
A. Turner syndrome
B. Edward syndrome
C. Potter’s syndrome
D. Myeloid leukemia
E. Sickle cell anemia
Reasons :

1) Turner : 45 XO
2) Edward : Cromosome 18
3) Potter : Potter Syndrome (PS*) is a term used to describe a
typical physical appearance, which is the result of a
dramatically decreased amniotic fluid volume
(oligohydramnios) secondary to renal diseases such as bilateral
renal agenesis (BRA). Other causes are obstruction of the
urinary tract, autosomal recessive polycystic kidney disease
(ARPKD), autosomal dominant polycystic kidney disease
(ADPKD) (in rare cases) and renal hypoplasia.

4) Sickle : autosomal recessive pattern of inheritance from


parents

37) All of the following are true about endometrial cancer are
true except:
A. Decreasing in incidence
B. Up to 25% occur in premenopausal women
C. Confined to the endometrium in stage 1a
D. Mean age is 61
E. Most common type is adenocarcinoma

Reasons : Its increasing in incidence.

RF : Obesity (Androgen converted to estrogen)


Nulliparity
Late Menopause
DM
FMHX of breast, colon, ovarian
Chronic unopposed estrogen stimulation
Tamoxifen

C : Its true
D : Hackers said 58
E : 75% are adenocarcinoma

38) Regarding endometrial hyperplasia, Which one of the


following is not true:
A. In young women it is treated by endometrial ablation
B. Old people can be treated by hormonal drugs safely
C. It is diagnosed only by biopsy
D. It is a premalignant lesion

Reasons : 80% occur in postmenopausal. Mean age is 61, 58


according to hackers. Hormonal TT is not enough. Should be
followed up by transvaginal U/S. Should be less than 4mm

39) Ca125 may be raised in all of the following except:


A. Menstruation
B. PID
C. Pregnancy
D. In 80% of mucinous cystadenoma
E. Endometriosis
40) US in a 21yearold
female showed an ovarian mass with calcification
and bony spicule, what is the most probable diagnosis ?
A. cystic teratoma
B. Clear cell carcinoma

Reasons : cystic teratoma can contains mature skin that is


complete with sweat glands, hair follicles as also pockets of
stale blood, fat, bone, cartilages, nails, teeth, and even traces
of thyroid tissues

41) All of the following are true about granulosa tumors except:
A. It is known to have low grade malignant potential
B. Progesterone secreting tumor

Reasons : Its estrogen releasing tumor

42) A young women presented with SOB, found to have pleural


effusion and bilateral ovarian masses, what is your diagnosis?
A. Fibroma

Reasons : I don’t know whats are the other options. But clearly
this patient have mets as a consequence from those cancer
43) All of the following are true about PCOS, except
A. It may be linked to atherosclerosis
B. Increased serum 17hydroxyprogesterone
C. Increased risk of development of diabetes type II
D. Ovulation may be normal
E. It can cause ovulatory amenorrhea

Reasons : Its anovulatory amenorrhea

44) All of the following are true about HELLP syndrome except:
A .More in primigravida
B. May be confused with hemolytic uremic sydrome
C. LDH can be elevated
D. Liver enzymes can be elevated
E. Presents with vague epigastric pain

Reasons : Its more common in multigravida

45) All45) All of the following are indications of severity in PET


except:
A. Intracranial hemorrhage
B. IUGR
C. 5 gm proteinuria in 24 hours
D. Polyuria

Reasons : Severe PET will cause Oligouria

46) All of the following are changes in PET, except:


A. hemodilution

Reasons : PET will cause hemoconcentration

47) All of the following are true about PET except:


A. Diazepam is used to prevent seizures

Reasons : Diazepam can abort eclmapsia but its not used as a


prophylactic

48) Which of the following is not true about MgSO4:


A. Can cause respiratory depression
B. Can cause cardiac toxicity
C. Absence of deep tendon reflexes is indication to increase
the
Dose

Reasons : MgSo4 has small therapeutic window. Its dose should


be monitored carefully. One of the indication of toxicity of
MgSo4 is absent of DTR

49) All of the following increase the risk of breech presentation


except:
A. Congenital malformation
B. Multiple pregnancy
C. Full maternal bladder

Reasons : Full maternal bladder is not a cause of breech.

Causes of breech presentation : Prematurity (most common),


pelvic tumor, anatomy of the uterus, contracted maternal
pelvis, multiple gestation

50) All of the following are risk factors for face presentation,
EXCEPT
A. Multiple pregnancy
B. Multipari
C. Contracted maternal pelvis
D. Anencephaly
E. Fetal goiter
Reasons : Causes of face presentation
a) Extreme prematurity
b) Multiparity
c) Fetal goitre
d) Anencephaly – anencephaly should be ruled out when face
presentation is suspected

51) All of the following are true regarding transverse lie EXCEPT:
A. In most of the cases the back is anterior
B. 15% risk of cord prolapse
C. 10% Associated with placenta previa
D. Transverse lie of the second twin is an indication for CS
E. In most of the cases the head is on the maternal left side
Reasons : Transverse lie is associated with shoulder
presentation. Not face, brow @ vertex

52) A lady presented after 72 hours with fever of 38 degree, All


of the
following are true except:
A. UTI is a cause in 5% of cases
B. Breast engorgement is a common cause
C. Length of labor is not relevant
D. DVT should be ruled out

Reasons :
Length of labour is relevant as the longer the duration of labour
might cause multiple PV exams which will eventually cause
puerperal sepsis.

53) Which of the following is NOT true about lactation


A. Oxytocin is released from the anterior pituitary
B. Prolactin is released from the anterior pituitary

Reasons : Oxytoxin – Posterior, Prolactin – anterior

54) All of the following are used in the management of uterine


atony
except:
A. Blynch
procedure
B. Bakri balloon
C. MgSO4
D. Progesterone analogue

Reasons : MgSo4 is used as a prophylactic for any eclamptic


episodes.

55) A patient just delivered an alive baby, had a massive


vaginal bleeding,
which of the following is correct in her management?
A. IM ergometrine
B. IV oxytocine with uterine massage
C. bimanual compression
D. exploratory laparotomy

Reasons : Most common cause of PPH is uterine atony. First line


management is bimanual uterine massage and oxytoxin
infusion.

56) All of the following are true regarding PPH except:


A. Most of the secondary PPH occurs during the first week
B. Most common cause of primary PPH is atony
C. Most common cause of secondary PPH

Reasons : Majority at the 2nd week


57) An a febrile ( it was written like this in the exam) presented
10 days after delivery with lower abdominal pain and mild
vaginal bleeding, What is true regarding her management
A. Reassurance
B. Immediate evacuation
C. Antibiotics orally for 7 days then review her condition
D. IV antibiotics followed by evacuation after 12 hours

Reasons : This is the case of RPOC

58) A P1 patient, her first delivery by CS, just gave birth to a live
child vaginally and the placenta was completely delivered, is
now having heavy vaginal bleeding and generalized
abdominal pain, what is your diagnosis:
A. Uterine Atony
B. Uterine Inversion
C. Uterine rupture
D. Retained products of conception

Reasons :

Atony – Does not come with pain


RPOC – The placenta is completely delivered
Uterine inversion – it doesn’t fit with generalized abd pain
Uterine rupture : Generalized abd pain (But im not really sure if
this the answer as uterine rupture occur during delivery.

59) All of the following are true regarding Hyperthyroidism in


pregnancy, all
of the following are true except:
A. Could be a cause of preterm labor and stillbirth
B. neonatal hyperthyroidism in offspring of grave’s is often
transient
C. Agranulocytosis is an indication for withdrawal of thioamides
D. May be caused by Hashimoto’s thyroiditis
E. PTU is the drug of choice

Reasons : Hashimoto’s will cause HYPOTHYROIDISM

60) Which of the following is NOT true about cardiac diseases in


pregnancy?
A. Uncomplicated coarctation of the aorta has 515%
mortality rate
B. Isolated ASD carries 30% mortality
C. Rheumatic heart disease is the most common cause in
developing countries
D. Pulmonary hypertension may cause sudden mortality death
Reasons : Isolated ASD carry only 1% mortality

61) Regarding Cardiovascular changes in pregnancy and


lactation, only one of the following is correct:
A. Peripheral vascular resistance initially rises during pregnancy
B. 40% increase in CO by 12 weeks
C. Cardiac output decreases in the third trimester
D. Cardiac output increases during labor
E. Breastfeeding increases cardiac output

Reasons : CO peaks in early 2nd trim, then its stable

62) Regarding regional anesthesia, all of the following are true


except:
A. Meningitis is a complication
B. Headache is a complication of spinal anesthesia
C. Epidural works faster than spinal
D. Coagulopathy is a contraindication for spinal anesthesia.

Reasons : Spinal work faster than epidural as its in subarachnoid


area

63) Pregnant lady in labor, 4 cm dilation, she is having 3


contractions per
10 minutes each lasting for 45 seconds. All of the following are
true
EXCEPT:
A. Pethidine use is safe
B. Should be augmented with oxytocin
C. May take 2 hours in primi
D. She is in the active phase of the first stage

Reasons : She’s progressing well. No need augmentation.

(4-5 contractions every 10 minutes that last 45s – 60s)

64) Regarding amniotomy, only one of the following is true:


A. Speeds up the progress of labor

Reasons : Amniotomy is AROM. AROM will stimulate


prostaglandin that will eventually stimulate uterine contractions
65) Regarding the second stage of labor, which one of the
following is not true
A. Fetal heart should be assessed every 5 minutes
B. Caput is diagnostic of CPD
C. Epidural can prolong the second stage of labor

Reasons : CPD can be dx with unsuccessful trial of labour. A


physical examination that measures pelvic size can often be
the most accurate method for diagnosing CPDPresent of caput
is not the definite dx of CPD

Causes of caput on fetus :


1) difficult labor
2) Uses of vaccum
3) PROM

66) All of the following are complications of postmaturity


except:
A. Meconium aspiration syndrome
B. RDS Comment [1]:

C. Oligohydroamnios

Explaination : complications of postmaturity mainly because of


two causes
(1) placenta insufficiency & hypoxia —leading to increase risk of
perinatal mortality, meconium aspiration syndrome,
oligohydroamnios and cord compression
(2) increase in fetal weight and ossification of skull with
decrease moulding leading to prolonged labor and shoulder
dystocia
67) All of the following can cause polyhydroamnios, EXCEPT:
A. Esophageal atresia
B. Diabetic mother
C. Anencephaly
D. ―easy answer‖ Comment [2]:

Explaination : causes of polyhydroamnios


(1) increased fetal urine production —maternal diabetes, twin-
twin transfusion syndrome(recipient), fetal hydrops
(2) fetal inability to swallow or absorb amniotic fluid —fetal GI
tract obstruction, fetal neurological or muscular abnormalities,
idiopathic
Source : (Oxford Handbook of O&G pg 140)

68) All of the following are true regarding Preterm labor except:
A. Complicates 10% of pregnancies
B. Home uterine activity monitoring was proved to
decrease the risk Comment [3]:

C. Positive fetal fibronectin indicates high risk for preterm


delivery
D. Previous preterm is a strong risk factor

Explaination:
Preterm labor is defined as one occurring after 20 weeks but
before 37 weeks of gestation.
The most recent test to be developed is cervical and vaginal
metal fibronectin. a positive fetal fibronectin test at 22 to 24
weeks predicts more than half of the spontaneous preterm
births that occur before 28 weeks.
A positive fetal fibronectin is also associated with short cervix,
vaginal infections and uterine activity. A negative test means
low risk of preterm delivery.
(Hacker’s pg 147)

69) All of the following are true about PPROM except:


A. Can be caused by cervical insufficiency
B. Can be caused by candida vaginal infection Comment [4]:

C. Erythromycin is the antibiotic of choice

Explanation:
PPROM : preterm with ruptured membranes, with or without
contractions.
Infections that associated with PPROM are gonorrhoea,
chlamydia, bacterial vaginosis,trichomoniases, among others.
The aetiologies are mainly vaginal and cervical infections,
abnormal membrane physiology, incompetent cervix and
nutritional deficiencies.
DOC for antibiotics are ampicilin or erythromycin.
(Hacker’s pg 150-152)

70) All of the following are true about gonorrhea except:


A. Can be effectively prevented by vaccination Comment [5]:

B. Best treated by penicillin


C. Gram negative cocci
D. Incubation period is 28 days
E. Can cause more disseminated disease in females than
males
Explanation:
gonorrhea : intracellular gram negative diplococcus
there’s no vaccine available for gonorrhoea.
mostly people infected are asymptomatic, often diagnosed at
screening.
treated with ceftriaxone and start on simultaneous treatment
on chlamydia (azithromycin)
treatment of all sexual contacts within 60 days before diagnosis
abstinence of intercourse for 7 days after starting antibiotics
(Hacker’s pg 271)

71) All of the following are true about STDs except:


A. Chlamydia is the leading cause of blindness in the world
B. Gonorrhea primarily infects the vagina
C. Syphilis is a systemic disease caused by a spirochete

Explanation:
chlamydia can cause reiter’s syndrome-more common in men
(a triad or arthritis, conjunctivits and urethritis)
gonorrhea mainly affects cervix(mucopurulent cervicitis),
vagina (Bartholin gland’s abscess) and possibly urethra (acute
urethritis)
sphilis caused by Treponema Pallidum, spirochete.

72) All of the following are risk factors for GBS infection except:
A. Previous GBS infection
B. Breech presentation Comment [6]:

C. ROM > 18 hours


D. Fever > 38

Explanation:

GBS could be transmitted from mother to her infant by vertical


transmission at delivery, preterm labor, PROM, LBW, prolonged
ruptured of membrane (more than 12 to 18 hours before
delivery), intrapartum fever and history of previously delivering
an infected infant
(Hacker’s pg 212)

73) SGA newborns are at the risk of one of the following :


A. Anemia

B. Hypothermia Comment [7]:

C. Hyperglycemia

Explanation:

SGA complications
I. Antenatal complications
1. metabolic changes (hypoxia, acidosis)
2. oligohydroamnipos
3. abnormal fetal heart
4. abnormal droppler
5. IUFD
II. Intrapartum complications
1. abnormal CTG
2. meconium stained liquor
3. increase incidence of instrumental and csec
4. fetal death
III. Neonatal complications
1. related to hypoxia and acidosis : meconium aspiration,
persistent fetal circ, hypoxic ischemic encephalopathy
2. metabolic : hypoglycemia, hypocalcemia,
hyperviscosity syndrome
IV. Related to etiology : chromosomal abnormalities, infection,
congenital anomalies
(from lecture notes)

74) All of the following are true about small for gestational age
(SGA) except:
A. Uterine artery doppler study can detect high risk infants
B. Biophysical profile assessment is less important for at risk
patients Comment [8]:

C. Long term risk of DM

Explanation:

A (true) ―elevations of umbilical artery and uterine artery


doppler assessments (increased resistance) as early as mid
pregnancy are asso cited with a greater risk for IUGR as
pregnancy progresses—Hacker’s pg 154

B (false) Hacker’s page 154

C (true) IUGR infants are at greater risk for adult onset diseases
such as hypertension, diabetes and atherosclerosis—Hacker’s
page 157
75) All of the following can cause deep dyspareunia except:
A. Endometriosis
B. Ovarian cyst
C. Vaginismus Comment [9]:

D. Fibroid

Explanation:

Deep dyspareunia caused by endometriosis, interstitial cystitis,


pelvic adhesions or neoplasm.
Vaginismus is defines as severe pain or involuntary spasm of
distal vaginal and pelvic floor muscles during attempted
penetration.
(Hacker’s pg 320)

76) All of the following are associated with low GnRH except:
A. Turner syndrome Comment [10]:

B. McCuneAlbright syndrome
C. Anorexia nervosa
D. Kallman syndrome

Explanation:

McCune Albright Syndrome —is a pseudosexual precocious


puberty disease. the precocious puberty are due to estrogen
exposure, independent of the HPaxis. (Hacker’s pg 351)

Anorexia nervosa can result in hypogonadotropic


hypogonadism and delayed puberty (Hacker’s pg 354)

Kallman syndrome present with hypogonadotropic


hypogonadism and anosmia or hyposmia. it may result from
mutation of KAL gene on the X chromosome or from autosomal
mutations that prevent the embryologic migration of GnRH
neutrons into the hypothalamus. These individuals may have
other anomalies of midline structures of the head. (Hacker’s pg
354)
77) The highest number of oogonia is found at which age:
A. 1 month in utero
B. 5 months in utero
C. At birth Comment [11]:

D. Puberty
E. 21 years

Explanation:

Active mitosis produces thousands of oogonia during fetal life.


No oogonia form postnatally. Although many oogonia
degenerate before birth, 2 million or so enlarge to become
primary oocytes before birth.
(Before We Are Born pg 178)

78) All of the following are contraindications for vacuum


EXCEPT:
A. Frank breech
B. Previous fetal scalp sampling
C. Mother is hemophilia carrier
D. Less than 33 weeks
Explanation:

Vaccum is contraindicated in preterm and face and breech


presentation. (Hacker’s pg 224-225)
It should not be used at <34weeks of gestation (Oxford
Handbook of O&G pg 294)

Comparing between vacuum and forceps, forceps


complications are more to the mother, so the answer could be
C.

79) Regarding instrumental vaginal delivery, one of the


following is true :
A. Knowing the position is not important
B. Submentobregmatic presentation can be delivered by
vacuum
C. High cavity instrumental delivery is the most common
D. Kielland forceps is used for rotation and traction Comment [12]:

Explanation:

(A) its obvious :)


(B) submentobregmatic is face presentation, contraindicated
in vacuum. (Hacker’s 225)
(C) the most common is mid cavity instrumental delivery —not
sure
(D) Kielland forceps-rotational, Simpson’s -non rotational
(Oxford Handbook of O&G pg 294)

80) All of the following are complications of forceps delivery


EXCEPT:
A. Sphincter injury
B. Cephalohematoma
C. Facial nerve injury
D. Spiral vaginal tear
E. Facial laceration Comment [13]:

Hacker’s page 225


Oxford Handbook of O&G page 293

81) All of the following are true about fibroids except:


A. More in caucasians Comment [14]:

B. Submucosal fibroids can cause sub fertility

Explanation:
Risk factors of developing leiomyomas include increasing age in
reproductive years, ethnicity ( African American), nulliparity and
family history.
Submucosal fibroids may be associated with an increased
incidence of infertility because of placentation challenges —
Hacker’s pg 243

82) All of the following are true about fibroids except:


A. The most common presentation in pregnancy is red
degeneration
B. HRT can be used in postmenopausal women with uterine
fibroid
C. Menorrhagia is the most common presentation Comment [15]:

D. Therapeutic Hysteroscopy can be used in the


management of submucosal fibroid
Explanation:
most uterine leiomyomas cause no symptoms. (Hacker’s page
243)

83) A 30 years old patient with DUB, all of the following are
possible treatment modalities for treatment except:
A. Endometrial ablation Comment [16]:

B. Mirena
C. Mefenamic acid
Explanation:
Endometrial ablation is indicated for the treatment of
menorrhagia in premenopausal or perimenopausal women
with normal endometrial cavities.
(http://emedicine.medscape.com/article/1618893-
overview#a3)

84) Regarding congestive dysmenorrhea, one of the following


is true
A. Starts in the first day and ends in the 3rd day
B. Can be ovulatory or anovulatory
C. Associated with diarrhoea and vomiting
D. Caused by Increased Prostaglandins in the second half
of the cycle
E. COCP is a specific treatment Comment [17]:

Explanation:
congestive dysmenorrhea is secondary dysmenorrhea (organic
pelvic disease)
(A) false, secondary dysmenorrhea is not limited to the menses
and can occur before as well as after the menses.
(Hacker’s page 258)
(B) false.not relevant. this is for DUB
(C) false. this is in primary dysmenorrhea, where the associated
symptoms are altered bowel habits, nausea, fatigue,
dizziness and headache (Hacker’s page 256)
(D) false. this is also in primary dysmenorrhea, where there is
higher than normal level of prostaglandins (especially
PGF2a and PGE2), can be reduced by NSAIDS. (Hackers
page 256)
(E) true. One of the benefit of COCP is decrease
dysmenorrhea. (Hackers page 256)

85) Regarding abdominal pain in pregnancy, all of the


following are true except:
A. Cholecystitis is the second most common cause
B. Red degeneration is treated by myomectomy Comment [18]:

Explanation:
Acute cholecystitis is the second most common surgical
condition in pregnancy (progesterone diminished smooth
muscle tone and predisposes to cholestasis leading to gallstone
formation (Oxford Handbook O&G page 96)

Red degeneration management : These women should be


treated conservatively as any surgical procedure during the
pregnancy can result in preterm delivery and foetal loss.
Although myomectomy in early pregnancy has been
successfully done, it is not recommended because of the risk of
maternal haemorrhage and foetal loss. Other associated
problems include placenta previa and intrauterine growth
restriction, fetal obstruction with malpresentation or obstructed
labour, postpartum haemorrhage, and puerperal infection.
(http://www.hindawi.com/journals/ogi/2012/539365/)

86) Regarding appendicitis in pregnancy, All of the following


are true except:
A. Incidence is less during pregnancy Comment [19]:

B. Perforated appendix has higher maternal mortality


C. Difficult to diagnose
Explanation:
Pregnant women have the same risk of appendicitis as non-
pregnant women. (Oxford Handbook page 94)
Its difficult to diagnose by CBC, since the increase of WBC
count in normal pregnancy further confuses the condition.
However, we can use ultrasound or MRI to confirm the
diagnosis. (Hacker’s page 215)

87) Regarding cord prolapse, one of the following is true


A. A common cause for intrapartum stillbirth
B. Incidence is 1 in 5000
C. The initial management is elevation of the presenting part
and expedited delivery
Comment [20]:

Explanation:
In case of cord prolapse, the foetus should be delivered as
rapidly as possible, either by instrumental delivery or caesarian.
During transfer for CS, we prevent further cord compression by
- knee to chest position
- fill the bladder with about 500 ml of warm normal saline to
displace the presenting part upwards
- a hand in vagina to push up the presenting part (may not
always be practical)
- prevent spasm by avoiding exposure of the cord, reduce
cord into vagina to maintain body temperature and insert
warm saline swab to prevent cord coming back out.
(Oxford Handbook of O&G page 401)

88) Regarding shoulder dystocia, one of the following is true:


A. Site of obstruction is the pelvic outlet
B. Many affected babies are of normal birth weight Comment [21]:

C. 70% risk of recurrence


Explanation:
Usually the anterior shoulder is impacted against symphysis
pubis (failure of internal rotation), posterior shoulder is impacted
against sacral promontory (causing bilateral impaction)
Estimated that only 50% of shoulder dystocia is associated with
a birth weight of >4kg.
(Oxford Handbook of O&G page 376)

89) Which one of the following can reduce C/S incidence


A. Regular antenatal care
B. External cephalic version Comment [22]:

C. External fetal monitoring


D. Internal heart monitoring
Explanation:
Two clinical interventions have been shown to reduce cesarean
delivery rates: external cephalic version (ECV ) and vaginal birth
after cesarean delivery (VBAC).
(Hacker’s page 226)

90) Comparing LSCS with classical C/S, all of the following are
advantages of LSCS except:
A. Less intestinal adhesions
B. Less bleeding
C. Less ureteric injury Comment [23]:

D. Less paralytic ileus


E. Less risk of spontaneous rupture in subsequent
pregnancies
Explanation:
LSCS carries a risk of lateral extension into the uterine vessels
and haemorrhage, more risk of bladder and ureteric injury
compared to classical CS (Oxford Handbook of O&G page
309)

91) All of the following are useful in the diagnosis of NTD except:
A. Maternal serum AFP
B. Amniotic fluid AFP
C. Amniotic fluid acetylcholinesterase
D. Karyotyping Comment [24]:

E. Pelvic US

Explanation:
NTD is a multifactorial disorders which means that both genes
and the environment play a role.
By ultrasound, anencephaly can be detected by 20 weeks,
there will be absence of cranium and buldging eyes (frog like
appearance). Spina Bifida will show frontal bone scalloping
(lemon sign), abnormal shaped cerebellum (banana sign).

92) Regarding HBV infection during pregnancy, All of the


following are true except:
A. Newborn of infected mother should receive
immunoglobulins and vaccine soon after birth
B. HBV vaccine is contraindicated in pregnancy
Comment [25]:
C. Infection during the third trimester has a higher risk of
transmission
Explanation:
Pregnant women at high risk for becoming infected with
hepatitis B who test negative for the HBsAg should be offered
vaccination.
(Hackers page 210)

93) Which one of the following patients should be managed


expectantly with steroids:
A. 32 weeks placenta previa, mild vaginal bleeding, no
other
comorbidities Comment [26]:

B. 37 weeks placenta previa with vaginal bleeding

Explanation:
Antenatal corticosteroid therapy for fetal pulmonary
maturation reduces mortality and the incidence of RDS and
intraventricular hemorrhage (IVH) in pre- term infants. These
benefits extend to a broad range of gestational ages (24 to 34
weeks) and are not limited by gender or race.
(Hackers page 154)

94) A patient with poorly controlled insulin dependent DM is at


higher risk of all of the following except:
A. Acute pyelonephritis
B. Hypertension
C. Caudal regression syndrome
D. Klinefelter syndrome Comment [27]:
Explanation:

Klinefelter syndrome is a kind of aneuploidy, 47XXY where the


testicular predominance occurs with the addition of a single Y
chromosome, even in the face of multiple X chromosomes. The
individual is almost always sterile and may have hypogonadism,
tall figure and occasionally reduced IQ. (Hackers page 248,
Oxford Handbook of O&G page 110)

95) All of the following are poor prognostic factors for


endometrial cancer EXCEPT:
A. Elderly
B. Hypertension and diabetes Comment [28]:

C. Invasion of lymphatics
D. Cervical involvement
Explanation:
Prognosis is dependent on several variables, including uterine
size, histologic type, grade of tumor, depth of myometrial
penetration, status of lymph nodes, status of peritoneal
cytologic features, and presence or absence of occult adnexal
or upper abdominal metastases.
Serous and clear cell endometrial carcinomas have a
particularly bad prognosis, and both of these histologic types
are prone to early dissemination.
Five-year survival rates for these tumor types are less than 50%,
even for patients with stage I disease. (Hackers page 432)

96) Regarding gestational diabetes, Only one of the following is


true
A. Fasting blood sugar may be normal Comment [29]:

B. Increase the risk of congenital anomalies

Explanation:
- Overnight fasting (8 hours minimum)
- 75g glucose load in 250-300ml water
- plasma glucose measured fasting and at 2h
*results
• Diabetes
• fasting glucose more or equal to 7.0 mmol/L
• 2h glucose more or equal to 11.1 mmol/L
• IGT
• fasting glucose less than 7.0 mmol/L
• 2 hours more or equal to 7.8, but less than
11.0mmol/L

NOTE : one one value needs to be abnormal to make the


diagnosis

(Oxford Handbook of O&G page 241)

97) Regarding OCPs All of the following are true except:


A. Inhibits ovulation
B. Alter cervical mucus
C. Decrease the risk of endometrial cancer
D. Increase the risk of ovarian cancer Comment [30]:

Explanation:

The use of oral contraceptive has been found to protect


against ovarian cancer, possibly because of suppression of
ovulation. (Hacker’s page 412)

Credits and gratitude to all dedicated members of


Group C Medicomaniax. We are sorry for any mistake
done.
‫بسم ميحرلا نمحرلا هللا‬
Obstetrics and Gynecology Final Exam
2014
‫ إلى طلبة كلية الطب البشري‬8002 ‫هذا إهداء من دفعة حياة‬

Note: The exam was two forms, 100 questions, 2 hours. This is From 1.

1) All of the following are benefits of ANC, except:


A. Early PET detection
B. Prevents Anemia
C. Prevents Prematurity
D. + Prevents Congenital anomalies

Booking investigation :
1. FBC – screen for screening anaemia and thrombocytopenia
2. blood group and red cell antibodies – will help cross matching blood
at later date if emergency arises. Women found to be rhesus D –ve will
be offered prophylavtic anti D administration
3. urine analysis – MSU sample to detect asymptomatic bacteriuria ( treat
earlier to prevent pyelonephritis )
4. screen for rubella/ hepatitis B / HIV / syphilis / heamoglobin studies
5. screening for fetal abnormalities ( down syndrome / neural tube
defect / structural congenital anomalies
6. screening for gestational diabetes ( if risk is present , the women should
be offered OGTT at 24-28 week)
7. screen for pre eclampsia by measuring BP and urinanalysis
8. women with history of preterm birth should be routinely offered
screening test foe preterm labour eg bacterial swab n cervical length
scan
9. fetal growth and well being measured by symphysis-fundal height
performed with tape measure and ultrasound screening
Ps : we only can screen for fetal abnormalities not for prevention . we can
prevent by controlling diet, avoid alcohol / radiation/ specific
medication, and vaccinated for rubella

2) Comparing abortion to ectopic, only one of the following is true:


A. + Shedding of the decidua can be mistaken for products of
conception
B. Endometrial curette can rule out ectopic
C. Abortion causes more pain
D. More vaginal bleeding in ectopic

Sometimes a small piece of decidua tissue becomes


loose and disintegrates through some unknown cause,
causing spotting. It‟s usually a hormonally stimulated
collection of menstrual-like tissue that can be confused
with a miscarriage. If it‟s just tissue debris, it can mean
nothing. No one knows why such a phenomenon
occurs, but it is harmless., such shedding of only
decidual tissue has had no impact on whether a
pregnancy would miscarry. If it‟s actual tissue of the
pregnancy (fetal or placental), then there should be
serious concern, because now this “threatened
miscarriage” is re-labeled “incomplete miscarriage.”
3) A pregnant lady, 7 week gestation, presented with mild vaginal
bleeding and severe abdominal pain. In which condition of the
following methotrexate can be used:
A. B-hCG level 3500, no intrauterine gestational sac is seen by Abdominal
US
B. + B-hCG level 2500, no intrauterine gestational sac is seen by
transvaginal US
C. Adnexal mass with positive heart beat

The discriminatory zone of β-HCG (ie, the level above which an imaging
scan should reliably visualize a gestational sac within the uterus in a
normal intrauterine pregnancy) is as follows

BHCG of 1200-2000 : IUP detectable with TVUS


BHCG of 5000 : IUP detectable with abdominal US
An abnormally rising hcg level > 2000 with no gestational sac seen on
tranvaginal ultrasound is diagnostic of ectopic pregnancy

Methothrexate is an option for early ectopic pregnancy and unruptured


Indication :
1. Heamodynamically stable
2. <3.5 cm unruptured mass
3. Desire future fertility
4. <15000 mIU/mL
5. No fetal cardiac motion
6. Able for follow up

Contraindication :
1. Fetal cardiac activity
2. BHCG > 15000 mIU /mL
3. > 3.5 cm gestational sac
4. Active renal / hepatic / peptic ulcer disease
5. Blood disease
6. Allergic
7. breastfeeding

4) In the management of infertility, All of the following are true except:


A. Unexplained in 30%
B. Hypothalamic pituitary dysfunction is the cause in PCOS
C. Management should be directed more toward females than males.
D. Male factor is the cause in 40% of cases

In 5-10% of couples no explanation can be found ( hacker ms 372 )

Hypothalamic pituitary dysfunction


Eg : kallman‟s syndrome
Sheehan syndrome
Hyperprolactinemia
Gnrh deficiency because of excessive exercise
Pituitary disease ( accumulation of iron )
Ovarian :
Eg : Pcos
Premature ovarian failure
Oocyte aging

Maybe B? not sure

5) Which of the following is an advantage of myomectomy over


hysterectomy in the management of fibroids?
A. + Spares reproductive function
B. Less bleeding
C. Less infection
D. Spares hormonal function
Hysterectomy is the surgical removal of the uterus. It may also involve
removal of the cervix, ovaries, fallopian tubes and other surrounding
structures.
Myomectomy is the surgical removal of fibroids from the uterus. It allows
the uterus to be left in place and, for some women, makes pregnancy
more likely than before. Myomectomy is the preferred fibroid treatment
for women who want to become pregnant

6) A patient with secondary amenorrhea who is being investigated


infertility, which of the following is not an important investigation?
A. FSH
B. Serum testosterone
C. Serum androstenedione
D. + Urinary hCG
E. Prolactin

Secondary amenorrhea is when a woman who has been having normal


menstrual cycles stops getting her periods for 6 months or longer. Women
who are pregnant, breastfeeding, or in menopause are not considered
to have secondary amenorrhea.

7) All are true regarding Rh alloimmunization, except:


A. + If a sensitized Rh –ve woman, undergoes amniocentesis, she should
receive anti-D after the procedure
B. Negative KB test for Rh –ve woman, she doesn‟t need anti-D
prophylaxis
C. External cephalic version is an indication for prophylaxis
D. More likely to occur if mother and fetus are ABO compatible

Once a women who is D rhesus negative has been sensitized , no


amount of anti D will ever turn the clock back . ( useless once
sensitization has occurred )
8) The main maternal complication for severe placental abruption is:
A. IUFD
B. Real tubular necrosis
C. Renal cortical necrosis
D. + Hemorrhagic shock
E. Amniotic fluid embolism

Complications in severe cases can include:


1.Decreased oxygen to the baby, which could lead to
brain damage
2. Stillbirth
3.Maternal blood loss leading to shock
4. Emergency hysterectomy (surgical removal of the
uterus) if the bleeding cannot be controlled
5.Maternal death from severe blood loss

9) Pregnant lady presented at 33 weeks, singleton, cephalic, with severe


placental abruption, all should be done except:
A. Urinary catheter
B. Vaginal examination
C. + Give steroids
Severe placenta abrubtion :
delivery is often the safest course of action. If the
fetus is stable, vaginal delivery may be an option. If
the fetus is in distress or the mom is experiencing
severe bleeding, then a cesarean delivery would
be necessary. Unfortunately, there is no treatment
that can stop the placenta from detaching and
there is no way to reattach it.

10) 25-year old pregnant, presented with severe placental abruption,


CTG showed multiple decelarations. Your management is:
A. Left lateral position and O2 mask
B. Induction of labor
C. + Emergent C/S

Treatment depends on the amount of blood loss and the status of the
fetus. If the fetus is less than 36 weeks and neither mother or fetus is in any
distress, then they may simply be monitored in hospital until a change in
condition or fetal maturity whichever comes first.
 If your baby shows any signs of distress ( DECELARATION ) , your health
care provider may induce your labor early. If you cannot give birth
vaginally, you will need a cesarean delivery (C-section).
Severe placental abruption is an emergency. You will need to deliver
right away, usually by C-section. It is very rare, but a baby can be stillborn
if there is a severe abruption.

11) Regarding placenta previa, what is false?


A. + Speculum examination is contraindicated
B. Increased risk of IUGR
C. Increased risk of congenital anomalies
A pelvic examination should not be performed until placenta previa has
been excluded , a sterile speculum examination can be safely done to
rule out genital tears or lesion that may be responsible for the bleeding .
A digital examination may be performed to determine whether cervical
dilatation is present.

Hacker ms128

12) All are side effects of progesterone-only pills EXCEPT


A. Acne
B. Weight gain
C. Functional ovarian cyst
D. + Inhibits lactation

Side effect of minipills ( progesterone only pill ):

1.breast tenderness
2.ovarian cyst
3.Continuous spotting
4.post pill amenorrhea
5.increase ectopic pregnancy
6. acne n weight gain
7. metabolic effect and osteoporosis

13) All are disadvantages and/or contraindications for Copper-releasing


IUCD, except:
A. + Reduces menstrual bleeding
B. Uterine malformation
C. History of PID
D. History of ectopic pregnancy

Contraindicate for copper releasing IUCD :


1. Pregnancy
2. Postpartum puerperal sepsis
3. Active STD or PID
4. Undiagnosed vaginal bleeding
5. Uterine anomalies
6. Copper allergic
7. Active liver disease
8. Immediately after septic abortion
9. Previous ectopic

14) All are modalities of tubal ligation except


A. Rings
B. Clips
C. + Hysterectomy
D. Saplingectomy
E. Diathermy

With traditional tubal ligation, the surgeon severs the tubes, and then ties
(ligates) them off thereby preventing the travel of eggs to the uterus.
Other methods include using clips or rings to clamp them shut, or severing
and cauterizing them.

15) Monozygotic twins with uncompensated AV shunts, all are true


except:
A. + Donor twin will have polyhydramnios
B. Donor will have have mirocardia
C. Diagnosis when Hb difference is 50 g/L .
twin to twin transfusion syndrome
( the presence of unbalanced anatomoses in the placenta typically
arterial- venous connection)

donor : recipient :
1. anemic 1. Polycythemic and
circulatory overload
2. growth restriction, microcardia 2. Cardiomegaly and
congestive heart failure
3. hypovolemia 3. hypervolemia
4. hypotension 4. hypertension
5. oligohydromnios 5. Polyhydromnious
6. oliguric 6. polyuric

diagnose by ultrasound :
1. same sex
2. intertwin weight different >20%
3. poly vs oligohydromnious
4. hb different >5g/dl
5. placenta vascular connection

16) All are true regarding twin pregnancy, EXCEPT


A. Antenatal betamethasone decrease perinatal mortality
B. Cervical cerclage prevents preterm labor
C. + Twenty percent are breech presentation
D. Second twin should be delivered within 30 minutes of the first

In the third period from the 36th gestational week onward, the incidence
of cephalic and breech presentation remain stable, i.e. breech
presentation around 3-4% and cephalic presentation approximately 95%

17) All are true regarding GTN except:


A. B-hCG is always higher than pregnancy levels
B. + Almost always presents with large for date uterus
C. Methotrexate is indicated for low risk malignant GTN
D. Most common karyotype for complete mole is 46 XX
E. Usually presents with vaginal bleeding in the first trimester

Complete mole :
About half of patient with molar pregnancy with uterus bigger than
expected date, whereas about one fourth have size compatible or
smaller than gestational age

Incomplete mole :
Uterine enlargement is much less common, most patient with small for
date

18) All are true regarding GTN except


A. + Chemotherapy is indicated if B-hCG levels plateau for 2 weeks
B. Serum B-hCG is used for follow-up
C. Incomplete mole contains fetal tissue
D. Snowstorm appearance is characteristic
E. Suction evacuation is the treatment of choice

„If the blood HCG level begins to rise or is still detectable after a
reasonable time (often around 4 to 6 months), it means that the patient
has persistent GTD (such as an invasive mole or choriocarcinoma). This
will need to be treated with chemotherapy „

19) All of the following HPV serotypes are associated with cervical
cancer except:
A. 16
B. + 6 C.
31
D. 33
E. 18

High-risk HPV strains :16 and 18, 31, 35, 39, 45, 51, 52, 58, and a few others.
Low-risk HPV strains, such as HPV 6 and 11

20) All are true regarding cervical cancer EXCEPT:


A. + Surgery is the cornerstone of treatment for all stages
B. Postcoital bleeding is a common presentation
C. Caused by persistent high-risk HPV infection

Treatment according to the stage

21) All are true regarding colposcopy, except:


A. + 3-5% alcohol is used to shed debris
B. Mosaic pattern and punctation are abnormal findings
C. Colposcope is a stereotactic binocular microscope
D. Columnar epithelium appears reddish and grape-like

Applying 3-5% dilute acetic acid for aceto white changes

22) All of the following are true regarding Cervical smear EXCEPT
A. HPV testing is beneficial in ASCUS
B. HSIL should be referred for colposcopy
C. Squamous epithelium stains mahogany brown by Lugol‟s iodine
D. + Leukoplakia is a normal finding
E. Indicated at 21 years or 3 years after the onset of sexual activity

Leukoplakia or hyperkeratosis is a white, well-demarcated area on the


cervix that may be apparent to the unaided eye, before the
application of acetic acid. The white colour is due to the presence of
keratin and is an important observation. Usually leukoplakia is
idiopathic, but it may also be caused by chronic foreign body
irritation, HPV infection or squamous neoplasia. No matter where the
area of leukoplakia is located on the cervix, it should be biopsied to
rule out high-grade CIN or malignancy

23) Regarding Tension-free Vaginal Tape (TVT), Only one of the following
is correct
A. Used for overflow incontinence
B. Success rate 65%
C. + Should be done under local analgesia
D. Detrusor instability is a known complication
E. Dysfunctional voiding is a side effect in 15%

The tension-free vaginal tape (TVT) procedure is designed to provide


support for a sagging urethra so that when you cough or move
vigorously or suddenly, the urethra can remain closed with no accidental
release of urine. ( stress incontinence)
In TVT surgery, a mesh tape is placed under your urethra like a sling or
hammock to keep it in its normal position. The tape is inserted through
tiny incisions in your abdomen and vaginal wall. No sutures are required
to hold the tape in place. TVT surgery takes about 30 minutes and may
be done under local anesthesia so you can cough at the surgeon's
request to test the tape's support of your urethra.

24) All of the following are true regarding urinary incontinence, except
A. Substracted cystometry is a useful investigation
b. + Anticholinergics are useful for treatment of stress incontinence C. 20-
30% of adult female population are affected

treatment :
stress incontinence : urge incontinence:
1. Kegel exercise 1. Anticholinergic / calcium
channel blocker
2. Estrogen therapy tricyclic antidepressant
3. a adrenergic agonist 2. Time voiding
4. surgical repair including : 3. Avoid stimulant n
diuretics

-anterior vaginal corpporraphy


- tension free vaginal tape
- suburethral sling
-retropubic bladder suspension
– periurethral injection

25) All are symptoms of urinary incontinence, except:


A. Frequency
B. + Straining to void
C. Urge incontinence
D. Stress incontinence
E. Overflow incontinence

Symptoms of urinary incontinence:

1.Not being able to control when you pass urine (stress n


overflow )
2.Having to urinate often during the day and night (
frequency )
3. Needing to urinate suddenly and urgently ( urge )

26) All of the following are taken in consideration in the choice of


endometriosis management, except:
A. Age
B. Desire to get pregnant
C. + Size of endometrioma
D. Score of disease severity
E. Severity of symptoms

The management of endometriosis depend on


1. The certainty of diagnosis
2. Severity of the symptoms
3. The extent of the disease
4. The desire of fertility
5. Age
6. The threat to gastrointestinal or urinary
hacker

27) Regarding endometriosis, only one is correct


A. + 65% affects the ovaries
B. Fixed retroverted uterus is a normal variant
C. Extent of disease is related to severity of symptoms

Exp:
A-yes, 2/3 affect ovary
B-no, anteroverted uterus is normal
C-no, extend of disease is related to type of symptoms eg:
dysmenorrhea: at retroperitoneal, dyspareunia at cul-de-
sac,Uterosacral ligament, dyschezia at retrosigmoidal
colon etc)

28) 35 year old, para 1 by C/S before 6 years, came


complaining of secondary dysmenorrhea, pelvic U/S was
normal. What is the most likely diagnosis?
A. Adenomyosis
B. Endometriosis
C. Submucous fibroid
D. Psychosomatic

Exp:
A-very likely answer: adenomyosis usually multipara,late
30,early 40s,not clear on U/S
B-endometriosis usually nullipara,30s,sx of 2ndary
amenorrhea
C-Submucous fibroid usually nullipara,>35y/o,can be seen
under U/S
D-osychosomatic usually associated with primary
dysmenorrhea

29) About GnRH analogue, all are true except:


A. Used in precocious puberty
B. + Optimal management of adenomyosis
C. It is anti-estrogenic

Exp:
A-yes
B-no, because treatment of adenomyosis is conservative(
NSAID,hormonal control,vaginal ring,DMPA,levonogestrel
IUD,continuous OCP and if failed, surgical is definitive
treatment.
GnRh is for treat menorrhagia
C-yes.
Gonadotropin-releasing hormone (GnRH), also known
as luteinizing hormone-releasing hormone (LHRH)
andluliberin, as well as gonadorelin (INN), is
a trophic peptide hormone responsible for the release
of follicle-stimulating hormone (FSH) and luteinizing
hormone (LH) from the anterior pituitary. GnRH is
synthesized and released from GnRH neurons within
the hypothalamus
(wiki)

30) Pregnant (10 weeks), presented with severe lower


abdominal pain. On examination, uterus is consistent with
10 weeks, she was found to have open external os, the
most likely diagnosis is

A. + Inevitable abortion
B. Missed abortion
C. Complete abortion
D. Ectopic pregnancy

Exp:
A-yes, pain , open ext os, uterus correct size for date
B-no, no pain and close ext os, uterus small for date
C-no, bleeding then pain and will stop, close ext os, empty
uterine cavity or RPOC
D-no, pain then bleeding, empty uterine cavity

31) All of the following could be a cause of recurrent


abortions except:
A. + Toxoplasomosis
B. Thromboplilia
C. Submucus fibroid
D. PCOS

Exp:
Recurrent abortion: 3 or more consecutive spontaneous
abortion,presented clinically as any type of abortion

Causes :
1-idiopathic
2-chromosomal disorders
3-anatomical disorders-cervical incompetence (cong or
acquired), uterine(submucous fibroid,uterine anomalies
&asherman‟s syndrome)
4-medical disorders
5-endocrine disorder (DM,thyroid,PCOS,corpus leteum
insufficiency)
6-immunological disorders( anticardiolipin syndrome, SLE)
7-Thrombophilia (congenital def of protein C&S and
antithrombin 3 & presence of factor leiden 5)
8-infection (ToRCH-CMV ToRH not cause rec abortion),
bacterial tract ifx eg:bacterial vaginosis)
9-Rh-isoimmunization

32) Regarding viral infections in pregnancy, All of the


following are true except:
A. + Toxoplasmosis is a viral infection.
B. CMV transmission risk is 40%

Exp:
1-no, toxoplasmosis is caused by single cell parasite
infection
B-yes its true
33) A young patient presented with mild vaginal bleeding
with severe abdominal pain, what is your first step in
management:
A. + Assess hemodynamic stability
B. Serum B-hCG
C. Ultrasound

34) All of the following can cause hemolytic disease of


newborn, EXCEPT: A. G6PD deficiency
B. Hexokinase deficiency
C. + Sickle cell trait
D. Hereditary spherocytosis

Refer
https://www.urmc.rochester.edu/encyclopedia/content.a
spx?ContentTypeID=90&ContentID=P02368

35) All of the following are true about sickle cell disease in
pregnancy except:
A. Chronic anemia
B. Bone marrow erythroid hyperplasia
C. IUGR
D. + Iron deficiency anemia (not associated with sickle)

Refer
http://contemporaryobgyn.modernmedicine.com/conte
mporary-obgyn/news/sickle-cell-disease-
pregnancy?page=full

36) All of the following are caused by chromosomal


abnormalities EXCEPT: A. Turner syndrome
B. Edward syndrome
C. Potter‟s syndrome
D. Myeloid leukemia
E. Sickle cell anemia

Exp:
A-no because Turner syndrome is caused by a missing or
incomplete X chromosome. People who have Turner
syndrome develop as females
B-no because Edwards syndrome (also known
as trisomy 18 [T18]) is a chromosomal abnormality caused
by the presence of all, or part of, an extra 18th
chromosome.
C-yes because potter not chromosomal abnormalities ,it is
atypical physical appearance of a fetus or neonate due
to oligohydramnios experienced in the
uterus.[1] Oligohydramnios is the decrease in amniotic
fluid volume sufficient to cause deformations
in morphogenesis of the fetus.
D-no . myeloid is chromosomal abnormalities
Chromosomal translocation (t) is the process by which a break in at least
two different chromosomes occurs, with exchange of genetic material.
Reciprocal translocation refers to an exchange between two or more
chromosomes in which there is no obvious overall loss of chromosomal
material. An example of a reciprocal translocation is the Philadelphia
chromosome, t(9;22)(q34;q11.2), seen in chronic myeloid leukemia.
●Chromosomal deletion (del) means loss of chromosomal material. An
interstitial deletion results from two breaks in a single chromosome with
the loss of intervening material. An example of an interstitial deletion is
the 5q- syndrome, or myelodysplastic syndrome with an isolated del(5q),
in which a variable portion (often the segment between bands q14 and
q33) of the long arm of chromosome 5 is lost.
E-no because sickle is mutation in a gene
on chromosome 11 that codes for the beta subunit of the
hemoglobin protein

8
37) All of the following are true about endometrial cancer
are true except: A. + Decreasing in incidence
B. Up to 25% occur in premenopausal women
C. Confined to the endometrium in stage 1a
D. Mean age is 61
E. Most common type is adenocarcinoma

Exp:
A-false , increasing in incidence
B-yes, 25% premenopause, 75% postmenopause
C-
D-yes true
E-yes, 80% adenocarcinoma, clear cell and papillary
serous type common in eder >70y/o and thin

38) Regarding endometrial hyperplasia, Which one of the


following is not true:
A. In young women it is treated by endometrial ablation
B. Old people can be treated by hormonal drugs safely
C. It is diagnosed only by biopsy
D. It is a premalignant lesion

Exp:
A-not true because Endometrial ablation is not a good
treatment for hyperplasia because it may mask
hyperplasia. Because ablation scars the inside of the
uterus, it stops the normal growth and proliferation of the
endometrium, so the endometrium cannot be evaluated
to see if it is reverting to normal or if the hyperplasia is
progressing.
B- may be true (not sure)
C-true
D-true

39) Ca-125 may be raised in all of the following except:


A. Menstruation
B. PID
C. Pregnancy
D. + In 80% of mucinous cystadenoma
E. Endometriosis

Exp:
D-mucinous cystadenoma- rise in CA 19.9
Rise CA-125-pregnancy, menstruation,endometriosis,
pleural effusion

40) US in a 21-year-old female showed an ovarian mass


with calcification and bony spicule, what is the most
probable diagnosis ?
A. + cystic teratoma
B. Clear cell carcinoma

41) All of the following are true about granulosa tumors


except:
A. It is known to have low grade malignant potential
B. Progesterone-secreting tumor

Exp:
Granulosa stromal tumor
-low grade malignancy
-secrete estrogen
-women of all ages
-marker:inhibin

42) A young women presented with SOB, found to have


pleural effusion and bilateral ovarian masses, what is your
diagnosis?
A. + Fibroma

43) All of the following are true about PCOS, except


A. It may be linked to atherosclerosis
B. Increased serum 17-hydroxyprogesterone
C. Increased risk of development of diabetes type II
D. Ovulation may be normal
E. It can cause ovulatory amenorrhea

Exp:
D-false ovulation not normal
TRIAD PCOS
1-hyperinsulin-increase in body due to less used in body
2-hyperandrogenism-due to hyperinsulinism
3-presence of cyst seen on U/S

44) All of the following are true about HELLP syndrome


except:
A. + More in primigravida
B. May be confused with hemolytic uremic sydrome
C. LDH can be elevated
D. Liver enzymes can be elevated E. Presents with vague
epigastric pain

Exp:
HELLP syndrome:
-hemolysis, elevated liver enzyme, low platelets
-more likely in multipara
>25 y/o, 36 w GA

45) All of the following are indications of severity in PET


except:
A. Intracranial hemorrhage
B. IUGR
C. 5 gm proteinuria in 24 hours
D. + Polyuria

Exp:
D-false because PET severerity/complications
-Usually oligouria <500ml/24 hr, +3 dipstick
-cerebral/visual disturbance
-pulmonary edema/cyanosis
-epigastric/RUQ pain
-impaired liver function
-DIC,HELLP
-Renal failure,hypoprotenemia
-Preterm labor
-stilbirth (IUFD)
-intrapartum fetal distress
-placenta abruption
-uteroplacental insufficiency
(hypoxia, IUGR,oligohydromnions)
46) All of the following are changes in PET, except:
A. + hemodilution

47) All of the following are true about PET except:


A. + Diazepam is used to prevent seizures

48) Which of the following is not true about MgSO4:


A. Can cause respiratory depression
B. Can cause cardiac toxicity
C. + Absence of deep tendon reflexes is indication to
increase the dose

Exp: (side effects of MgS04)


1-feeling of warmth&flushing at first administration
2-respiratory depression
3-cardiac conduction defect/arrest
4-neonate effects-loss of ms tone&drowsiness (lower
apgar),proong preterm bcoz decrease renal clearance
5-loss of calcium (long term use)-give prophylaxis ca
theraphy

49) All of the following increase the risk of breech


presentation except:
A. Congenital malformation
B. Multiple pregnancy
C. + Full maternal bladder

Exp: (risk of breech)


1-maternal: nulliparity,old
age,fibroid,polyhydromnions,oligohydromnions,bicornuate
/septate uterus, hx of breech,uterine/pelvis tumors
2-fetal: prematurity,IUGR, large babies, multiple
gestations,fetal abnormalities,congenital
abnormalities,short umbilical cord, cephalo-pelvis
disproportion
50) All of the following are risk factors for face
presentation, EXCEPT
A. + Multiple pregnancy
B. Multiparity
C. Contracted maternal pelvis
D. Anencephaly
E. Fetal goiter

51) All of the following are true regarding transverse lie


EXCEPT:
A. + In most of the cases the back is anterior
B. 15% risk of cord prolapse
C. 10% Associated with placenta previa
D. Transverse lie of the second twin is an indication for CS
E. In most of the cases the head is on the maternal left side

52) A lady presented after 72 hours with fever of 38


degree, All of the following are true except:
A. UTI is a cause in 5% of cases
B. Breast engorgement is a common cause
C. + Length of labor is not relevant
D. DVT should be ruled out

53) Which of the following is NOT true about lactation


A. + Oxytocin is released from the anterior pituitary
B. Prolactin is released from the anterior pituitary
Exp:
Oxytocin released from posterior pituitary

By: NAH

54) All of the following are used in the management of uterine


atony except:
A. B-lynch procedure
B. Bakri balloon
C. + MgSO4
D. Progesterone analogue
Explanation: Uterine atony management (1)Vigorous
massage and oxytocin (2) Bimanual compression (3)
Uterotonic agents (Ergot, Misoprostol, Hemabate,
Carboprost) (4) Bakri ballon (5) Injection of thrombogenic
materials (6) Surgical intervention (ligate uterine arteries, B-
lynch suture, supracervical/TAH.
Source: PPH lecture notes (Dr. Haifa)

55) A patient just delivered an alive baby, had a massive vaginal


bleeding, which of the following is correct in her management?
A. IM ergometrine
B. + IV oxytocine with uterine massage
C. bimanual compression
D. exploratory laparotomy
Explanation: First management in PPH if suspected uterine atony, is
commence vigorous massage + therapeutics oxytocin (rapid
continuous IV infusion of dilute oxytocin :- 40-80U in 1L of NS > to
increase uterine tone.
Source: PPH lecture note (Dr.Haifa)

56) All of the following are true regarding PPH except:


A. + Most of the secondary PPH occurs during the first week
B. Most common cause of primary PPH is atony
C. Most common cause of secondary PPH

Explanation: Majority of secondary PPH occurs between 7-


14 days (which is second week)
Source : Lecture note 5 th year (Dr Haifa)

57) An a febrile ( it was written like this in the exam) presented 10 days
after delivery with lower abdominal pain and mild vaginal bleeding,
What is true regarding her management
A. Reassurance
B. Immediate evacuation
C. Antibiotics orally for 7 days then review her condition
D. + IV antibiotics followed by evacuation after 12 hours
58) A P1 patient, her first delivery by CS, just gave birth to a live child
vaginally and the placenta was completely delivered, is now having
heavy vaginal bleeding and generalized abdominal pain, what is your
diagnosis:
A. Uterine Atony
B. Uterine Inversion
C. + Uterine rupture
Retained products of conception .E

Explanation: Previous C/S is risk factor for Uterine Rupture. Uterine


rupture usually comes with sudden onset of intense abd pain and
some vaginal bleeding. After rupture, there is free of pain
momentarily, then diffuse pain thereafter.

59) All of the following are true regarding Hyperthyroidism in pregnancy,


all of the following are true except:
A. Could be a cause of preterm labor and stillbirth
B. neonatal hyperthyroidism in offspring of grave‟s is often
transient
C. Agranulocytosis is an indication for withdrawal of
thioamides
D. + May be caused by Hashimoto’s thyroiditis
E. PTU is the drug of choice

Explanation:
A. (Hacker and Moore‟s, Page 195)
B. Neonatal Hyperthyroidism is transient for 2-3 months but
with high neonatal mortality rate about 16% (Hacker &
Moore‟s, Page 196)
C. PTU can cause agranulocytosis (Ten Teachers, Page165)
D. Hashimoto‟s thyroiditis more common cause
hypothyroidism (Ten Teachers, Page 165)
E. Ten Teachers, Page 165

60) Which of the following is NOT true about cardiac diseases


in pregnancy?
A. Uncomplicated coarctation of the aorta has 5-15%
mortality rate
B. + Isolated ASD carries 30% mortality
C. Rheumatic heart disease is the most common cause in
developing countries
D. Pulmonary hypertension may cause sudden mortality death

61) Regarding Cardiovascular changes in pregnancy and lactation,


only one of the following is correct:
A. Peripheral vascular resistance initially rises during pregnancy
B. 40% increase in CO by 12 weeks
C. Cardiac output decreases in the third trimester
D. + Cardiac output increases during labor
E. Breastfeeding increases cardiac output
Explanation: The cardiac output is elevated at the onset of
labour to over 7.0L/min, rising further within labor, with a
30% increase in demand in the final stage.
Source: Ten Teachers Pages 27

62) Regarding regional anesthesia, all of the following are true


except:
A. Meningitis is a complication
B. Headache is a complication of spinal anesthesia
C. + Epidural works faster than spinal
D. Coagulopathy is a contraindication for spinal
anesthesia

Explanation: Spinal works faster than epidural


Source: Table 8-8 Hacker & Moore‟s Page 113

63) Pregnant lady in labor, 4 cm dilation, she is having 3 contractions per


10 minutes each lasting for 45 seconds. All of the following are true
EXCEPT:
A. Pethidine use is safe
B. + Should be augmented with oxytocin
C. May take 2 hours in primi
D. She is in the active phase of the first stage
Explanation: Only ARM (amniotomy).
Sources : Hacker & Moore‟s, Page 99-100

64) Regarding amniotomy, only one of the following is true:


A. + Speeds up the progress of labor
Explanation: ROM cause increase of uterine contractility
Source: Hacker & Moore‟s Page 100

65) Regarding the second stage of labor, which one of the


following is not true
A. Fetal heart should be assessed every 5 minutes
B. + Caput is diagnostic of CPD
C. Epidural can prolong the second stage of labor

Explanation: Amount of moulding will be more pronounced in the


presence of CPD. Caput is a localized, edematous swelling of the
scalp caused by pressure of the cervix on the presenting portion of the
fetal head
Source: Hacker & Moore‟s Page 101
66) All of the following are complications of postmaturity
except:
A. Meconium aspiration syndrome
B. + RDS

Oligohydromnios

Explanation: Postterm pregnancies increase the risk of problems such as


difficult labor, need for cesarean delivery, and passage of meconium (the
fetus‟s first stool) before delivery. Meconium can sometimes be inhaled
before or during delivery, causing the baby to have difficulty breathing
shortly after birth. In the postmature fetus, soft tissues (such as muscle) may
waste away. The fetus or newborn may be deprived of oxygen, or die.

Typically, tests are started at 41 weeks to evaluate the


fetus‟s movement and heart rate and the amount of
amniotic fluid, which decreases markedly in postterm
pregnancies
Source: http://www.msdmanuals.com/home/women-s-
health-issues/complications-of-labor-and-
delivery/postterm-pregnancy-and-postmaturity

67) All of the following can cause polyhydromnios, EXCEPT:


A. Esophageal atresia
B. Diabetic mother
C. Anencephaly
D. + Placenta Previa
Explanation: Polyhydromnious (AFI >23cm), sign of poor
control in a diabetic pregnancy or an indication that the
fetus may have an anomaly.
Source: Hacker & Moore‟s Page 89
68) All of the following are true regarding Preterm labor
except:
A. Complicates 5-10% of pregnancies
B. + Home uterine activity monitoring was proved to
decrease the risk
C. Positive fetal fibronectin indicates high risk for preterm delivery
D. Previous preterm is a strong risk factor

69) All of the following are true about PPROM except:


A. Can be caused by cervical insufficiency
B. + Can be caused by candidal vaginal infection
C. Erythromycin is the antibiotic of choice

70) All of the following are true about gonorrhea


except:
A. + Can be effectively prevented by vaccination
B. Best treated by penicillin
C. Gram negative cocci
D. Incubation period is 2-8 days
D. Can cause more disseminated disease in females than males
Explanation: There‟s no vaccine available.
Source: Hacker & Moore‟s Page 271

71) All of the following are true about STDs except:


A. Chlamydia is the leading cause of blindness in the world
B. + Gonorrhea primarily infects the vagina
C. Syphilis is a systemic disease caused by a spirochete

Explanation: Gonorrhea is a bacterial infection which


primarily affects the vagina, cervix, uterus, and fallopian
tubes in women and affects the urethra, epididymis, and
testes in men.
Sources: http://www.iaac.ca/en/163-666-sexually-
transmitted-diseases-std-s-and-infertility-4
72) All of the following are risk factors for GBS infection except:
A. Previous GBS infection
B+ Breech presentation
C. ROM > 18 hours
D. Fever > 38
Explanation: Intrapartum antibiotics prophylaxis would be
indicated for pregnant women with (1)prev infant with
invasive GBS disease, (2) GBS bacteruria during the current
pregnancy (3)positive GBS screening culture during the
current pregnancy (4) unknown GBS status with one of the
high-risk factor sucah as intrapartum fever ≥38°C, preterm
delivery <37 weeks, or prolonged membrane rupture ≥18
hours.
Source: Hacker & Moore‟s Page 213

73) SGA newborns are at the risk of one of the following: A.


Anemia
B. + Hypothermia
C.Hyperglycemia
Explanation: Growth-restricted fetuses are particularly
prone to problems such as meconium aspiration, asphyxia,
polycythemia, hypoglycemia, and mental retardation.
Source: Hacker & Moore‟s Page 153

74) All of the following are true about small for gestational age (SGA)
except:
A. Uterine artery doppler study can detect high risk infants
B. + Biophysical profile assessment is less important for at risk patients
C. Long-term risk of DM
Source: Hacker & Moore‟s Page 153-154

75) All of the following can cause deep dyspareunia except:


A. Endometriosis
B. Ovarian cyst
C. + Vaginismus
D. Fibroid
Explanation: Vaginismus classified as pain with intromission.
Vaginismus is defined as severe pain or involuntary spasm
of the distal vaginal and pelvic floor muscles during
attempted penetration.
Deep-thrust dyspareunia often due to endometriosis,
interstitial cystitis, pelvic adhesion, or neoplasm)
Source: Hacker & Moore‟s Page 320

76) All of the following are associated with low GnRH except:
A. + Turner syndrome
B. McCune-Albright syndrome C.
Anorexia nervosa
D. Kallman syndrome
Explanation: Patients with hypergonadotropic
hypogonadism have form of failed gonadal development
or premature gonadal failure and will have elevated FSH
levels. The differential diagnosis includes 45XO (Turner
syndrome)…..
Source: Hacker & Moore‟s Page 357

77) The highest number of oogonia is found at which age:


A. 1 month in utero
B. + 5 months in utero
C. At birth
D. Puberty
E. 21 years
Explanation: Maximum number of oogonia occurs at about
the fifth month of development. Differentiation into
oogonia begins once the primordial germ cells have
arrived in the gonad of a genetic female. After undergoing
a number of mitotic divisions, these fetal cells form a cluster
in the cortical part of the ovary. Some of these oogonia
differentiate into the larger primary oocytes, which by the
third month of development are found in the deeper layers
of the gonad. The primary oocytes begin meiosis to
produce secondary oocytes. At the same time, the number
of oogonia continues to increase to about 6,000,000 in the
fifth month. At this time, most of the surviving oogonia and
some of the oocytes become atretic. However, the
surviving secondary oocytes (400,000 to 1,000,000) become
surrounded by epithelial cells and form the primordial
follicles by the seventh month. During childhood there is
continued atresia so that by puberty only about 40,000
secondary oocytes remain.
Source: http://aippg.net/threads/oogonia-reach-their-
maximum-number-at-which-of-the-following.92445/

78) All of the following are contraindications for vacuum EXCEPT


A. Frank breech
B. Previous fetal scalp sampling
C. +Mother is hemophilia
D. Less than 33 weeks
Explanation :
(A) General contraindication
(B) Relative contraindication
(C) –
(D) Relative contraindication
(Prematurity)

Source : http://emedicine.medscape.com/article/271175-
overview#a4

79) Regarding instrumental vaginal delivery, one of the following is true


: A. Knowing the position is not important
B. Submentobregmatic presentation can be delivered by
vacuum
C. High cavity instrumental delivery is the most common
E. Kielland forceps is used for rotation and traction

80) All of the following are complications of forceps delivery EXCEPT:


A. Sphincter injury
B. Cephalohaematoma
C. Facial nerve injury
D. Spiral vaginal tear
E. Facial laceration
Explanation : All can be the complication of forceps delivery.

81) All of the following are true about fibroids except:


A. + More in caucasians
B. Submucosal fibroids can cause subfertility
Explanation: Risk factors for developing leiomyomas include
increasing age during the reproductive years, ethnicity
(African American women have a least a twofold to
threefold inced risk compared with white women),
nulliparity and family history.
Source: Hacker & Moore‟s Page 241

Credits and gratitude to all dedicated


members of Group C Medicomaniax. We
are sorry for any mistake done.
82) All of the following are true about fibroids except:
A. The most common presentation in pregnancy is red degeneration
B. HRT can be used in postmenopausal women with uterine fibroid
C. Menorrhagia is the most common presentation
D. Therapeutic Hysteroscopy can be used in the management of submucosal fibroid

83) A 30yearold patient with DUB, all of the following are possible treatment modalities for
treatment except:
A. Endometrial ablation
B. Mirena
C. Mefenamic acid

84) Regarding congestive dysmenorrhea, one of the following is true


A. Starts in the first day and ends in the 3rd day
B. Can be ovulatory or anovulatory
C. Associated with diarrhea and vomiting
D. Caused by Increased Prostaglandins in the second half of the cycle
E. COCP is a specific treatment

85) Regarding abdominal pain in pregnancy, all of the following are true except:
A. Cholecystitis is the second most common cause
B. Red degeneration is treated by myomectomy

86) Regarding appendicitis in pregnancy, All of the following are true except:
A. Incidence is less during pregnancy
B. Perforated appendix has higher maternal mortality
C. Difficult to diagnose

34
87) Regarding cord prolapse, one of the following is true
A. A common cause for intrapartum stillbirth
B. Incidence is 1 in 5000
C. The initial management is elevation of the presenting part and expedited delivery

88) Regarding shoulder dystocia, one of the following is true:


A. Site of obstruction is the pelvic outlet
B. Many affected babies are of normal birth weight
C. 70% risk of recurrence

89) Which one of the following can reduce C/S incidence


A. Regular antenatal care
B. External cephalic version
C. External fetal monitoring
D. Internal heart monitoring

90) Comparing LSCS with classical C/S, all of the following are advantages of LSCS except:
A. Less intestinal adhesions
B. Less bleeding
C. Less ureteric injury
D. Less paralytic ileus
E. Less risk of spontaneous rupture in subsequent pregnancies

91) All of the following are useful in the diagnosis of NTD except:
A. Maternal serum AFP
B. Amniotic fluid AFP
C. Amniotic fluid acetylcholinesterase
D. Karyotyping
E. Pelvic US
35
92) Regarding HBV infection during pregnancy, All of the following are true except:
A. Newborn of infected mother should receive immunoglobulins and vaccine soon after birth
B. HBV vaccine is contraindicated in pregnancy
C. Infection during the third trimester has a higher risk of transmission

93) Which one of the following patients should be managed expectantly with steroids:
A. 32 weeks placenta previa, mild vaginal bleeding, no other comorbidities
B. 37 weeks placenta previa with vaginal bleeding

94) A patient with poorly controlled insulindependent DM is at higher risk of all of the following
except:
A. Acute pyelonephritis
B. Hypertension
C. Caudal regression syndrome
D. Klinefelter syndrome

95) All of the following are poor prognostic factors for endometrial cancer EXCEPT:
A. Elderly
B. Hypertension and diabetes
C. Invasion of lymphatics
D. Cervical involvement

96) Regarding gestational diabetes, Only one of the following is true


A. Fasting blood sugar may be normal
B. Increase the risk of congenital anomalies

97) Regarding OCPs All of the following are true except:


A. Inhibits ovulation
36
B. Alter cervical mucus
C. Decrease the risk of endometrial cancer
D. Increase the risk of ovarian cancer

”‫ والعاجز من أتبع نفسه هواها وتمنى على هللا األماني‬،‫“الكّيس من دان نفسه وعمل لما بعد الموت‬

3. 5th Year Final Exam 2013:


Note: The exam was 100 questions, 2 hours, 2 forms. Most of the questions were the same in both
forms.

1. A lady had her last menstrual period on 27/12/2012. What is her expected date of delivery?

A. 03/10/2013
B. 20/09/2013
C. Cannot be calculated

Note: This Q seems easy and most of us had chosen "A", but the problem is that it didn't
mentioned whether the Cycle was regular or not… "The "Sure" criteria  "

2. A lady presented with uterine contractions (3 per 10 minutes). The cervix is 4 cm dilated and 1
cm long, what’s the stage of labor?

A. Active first stage

B.+Latent phase

C. Active second stage

D. Third stage

E. She is not in labor

3. A baby with breech presentation. Hips are flexed and knees are extended, what’s the type of
breech?

37
5th Year Final Exam 2013:
1. A lady presented with uterine contractions (3 per 10 minutes). The
cervix is 4 cm dilated and 1 cm long, what’s the stage of labor?
A. Active first stage
B.+ Latent phase
C. Active second stage
D. Third stage
E. She is not in labor

Explanation: (fully effaced cervix indicate end of latent phase.(slide


normal labor,thn 5,dr laila)

2. A lady had her last menstrual period on 27/12/2012. What is her


expected date of delivery?
A. 03/10/2013
B. 20/09/2013
C. Cannot be calculated

Explanation: This Q seems easy and most of us had chosen "A", but
the problem is that it didn't mentioned whether the Cycle was
regular or not… "The "Sure" criteria.

3. A baby with breech presentation. Hips are flexed and knees are
extended, what’s the type of breech?
A. Complete
B. Incomplete
C. +Frank

Explanation: breech: (look at hip and knee joint)

1) complete breech: hip and knee joint flex


2) frank breech: hip joint flex, knee joint extend
3)incomplete breech: has one or both hip extended so the foot or
knee lies below breech in the birth canal (could be single footling
breech or double footling breech)
(lecture note malpresentation,thn 5,and Blueprint)
4. In vertex presentation, what’s the bony denominator of the
presenting part?
A. +Occiput
B. Sinciput
C. Chin

Explanation: fetal position:


1) vertex;occiput = suboccipital-bregma (9.5 cm)
2) face;chin =subment0-bregma (9.5cm)
3) brow;sinciput = mento-vertical (13 cm)
(lecture note malpresentation,thn 5)

5. The relation of the presenting part of the fetus to maternal ischial


spines is called….
A. Lie
B. Position
C. +Station
D. Presentation

Explanation: station=lowest bony part of fetal to imaginery line


between 2 ischial spine. (lecture note malpresentation thn 5)

6. All of the following are correctly matched, except:


A. Vertex --- Suboccipito-bregmatic

B. +Brow --- Occipito-frontal

C. Face --- Submento-bregmatic

Explanation: brow;sinciput=mento-vertex=13cm. (morning gyne


session lect note,thn 5)

7. Regarding normal labor, all of the following are true, except:


A. Second stage starts with full cervical dilation and ends with
delivery of the baby
B. +True uterine contractions can be suppressed by epidural
anesthesia
C. Third stage is delivery of placenta and the membranes
D. Second stage takes longer in primigravida compared to
multiparous
E. False contractions don‘t cause cervical dilation

Explanation: epidural anesthetic does not commonly remove all


sensation and can actually be detrimental to the ability to push
during during the second stage if it does so.
(blueprint OnG,pg 56)

8. Regarding the mechanism of normal labor, all of the following are


true, except:
A. Fetus enters the pelvis in the transverse diameter
B. +Internal rotation is for fetal shoulders
C. The anterior shoulder is delivered before the posterior
D. The head is delivered in extension

Explanation: internal rotation; occipital rotate anteriorly,sagittal


suture AP diameter in pelvic outlet
(gyne morning session,lecture thn 5)

9. All of the following are classes of tocolytics, except:


A. Calcium channel blockers
B. +Anticholinergics
C. Oxytocin antagonists
D. Prostaglandin inhibitors

Explanation:
calcium channel blocker: nifedepine (blueprint,pg 80)
oxytoxin antagonist: atosiban (blueprint pg 80)
prostaglandin inhibitor: indomethacin (blueprint pg 80)

10. All of the following are correctly matched, except:


A. Face --- presentation
B. Flexed --- Attitude
C. Transverse ---- Lie
D. + Vertex --- Position

Explanation:
lie: relation between long axis of fetal to long axis of mother,thus it is
suppose to be longitudinal not vertex (as vertex is part of
presentation)
lie could be either longitudinal,oblique or transverse.
presentation could be either
vertex,cephalic,face,brow,breech,shoulder or compound.
position: relationship between denominator of fetal bony part to
pelvic cavity
(lect malpresentation,obsetric examination thn 5)

11. Which of the following is considered an abnormal finding in CTG?


A. Early deceleration in second stage
B. +Late deceleration
C. Baseline heart rate of 120
D. Absence of accelerations

Explanation:
Late deceleration= pathology;due to placenta insufficiency.

13. Which of the following is NOT part of the routine partogram?


A. Vital signs
B. +Pelvimetry
C. Cervical dilation
D. Strength of contractions

Explanation:
Pelvimetry= pelvimetry: measurement of the dimensions of the bony
pelvis, undertaken chiefly to help determine whether a woman can
give birth normally or will require a caesarean section (dr ta pernah
ckp pon tiap kali explain pasal partogram)

14. Regarding fetal skull in relation to maternal pelvis, all are true,
except:
A. Platypeloid pelvis is likely to cause obstructed labor
B. Android is the typical male pelvis
C. +Gynecoid pelvis forces the baby into an occipito-posterior
position

Explanation:
Platypeloid pelvis is likely to cause obstructed labor= the evidence is
in gyne mornin session lecture page 18,under the tittle pelvic types

Android is the typical male pelvis= the evidence is in gyne mornin


session lecture page 18,under the tittle pelvic types
15. All of the following are true regarding spinal anesthesia, except:
A. + Can cause hypertension
B. Meningitis is a possible complication
C. Contraindicated in case of coagulopathy
D. Contraindicated if skin is infected at the site of needle

Explanation:
one of the complication is hypotension,not hypertension. slide
obstetric analgesic and anasthethic.

16. Regarding the use of pethidine in labor, all are true, except:
A. Can cause neonatal respiratory depression
B. Can cause neonatal bradycardia
C. Easily administered
D. Naloxone is the antidote
E. + Inhibits uterine contractions

Explanation:
Does not innhibits uterine contractions (slide obstetric analgesic and
anesthethic)

17. A baby in breech presentation, where would you listen to the


fetal heart?
A. + Above the umbilicus on the side of the back
B. Above the umbilicus opposite to the back
C. Below the umbilicus on the side of the back
D. Below the umbilicus opposite to the back
E. At the umbilicus

18. All of the following are risk factors for vaginal candidiasis,
except:
A. + Thyrotoxicosis
B. Oral contraceptive use
C. Pregnancy
D. Diabetes

Explanation:
Oral contraceptive : causing hormonal imbalance(
http://www.healthline.com/health/vaginal-yeast-infection#Causes2
)
Pregnancy: eventhough there is increase in WBC but they arent
function properly(kertas physiological changes in pregnancy,thn 5)

Diabetes: impaired immunity,(general knowledge)

19. Regarding Trichomonas vaginalis infection, only one of the


following is correct
A. Commonly causes salpingitis
B. + It‘s a parasitic STD that causes a pruritic discharge.
C. Clue cells are characteristic

Explanation:
Commonly causes salpingitis: cervicitis which cause by neisseria
gonorrhea and chlamydia trachomatis (buluprint textbook)

Clue cells are characteristic: criteria for bacterial vaginosis which is


cause by gardenella vaginalis
(blueprint textbook)

20. Regarding vulvovaginal candiasis, only one of the following is


correct:
A. + can be microscopically diagnosed by using KOH
B. Muco-purulent vaginal discharge

Explanation: the evidence is in blueprint textbook page 222

21. By definition, PID is:


A. Infection of the vagina
B. Infection of the cervix
C. + Endometritis and salpingo-oophoritis

Explanation: PID : Ascending infection of the female upper genital


tract (the female structures above the cervix). Define by
MedicineNet.com

22. The main postpartum complication of placental abruption is:


A. + Post partum hemorrhage
B. Sheehan syndrome
C. Renal tubular necrosis
D. Renal cortical necrosis
E. Endomyometritis

Explanation:
Post partum hemorrhage: placenta abruption is the risk factor for
PPH (slide PPH 5 yr)

Sheehan syndrome: complication of PPH (slide PPH 5yr)

Renal tubular necrosi: complication of PPH (slide PPH,5yr)

Renal cortical necrosis: complication of PPH (lecture


APH,PID,endometriosis 5 yr)

Endomyometritis: complication of ceserean delivery but rare in


vaginal delivery (blueprint textbook)

23. Placenta previa is associated with increased risk of all of the


following except:
A. IUGR
B. Primary PPH
C. + Prolonged pregnancy

Explanation:
Prolonged pregnancy: not even mentioned in
APH,PID,endometriosis lecture note

24. Regarding postpartum hemorrhage, all of the following are true,


except:
A. Atony is the most common cause of primary PPH
B. Infection of RPOC is the most common cause of secondary PPH
C. + Secondary PPH occurs between 24 hours and 6 months
postpartum

Explanation:
Secondary PPH 24 hrs- 6 WEEKS postpartum (slide PPH,5 yr)

24. A lady presented with vaginal bleeding 12 days postpartum, all


of the following are true, except:
A. This is secondary postpartum hemorrhage
B. + Antibiotic therapy has no role

Explanation: Infection (2° to RPOC), most common cause of


postnatal morbidity
2nd-10th day. (pdf PH 6 yr)

25. The main risk factor for placental abruption is:


A. Primigravida
B. + PET
C. Maternal age

Explanation: most common factor associated with increased


incidence of abruption is hypertension,whether it is chronic,result of
pre-eclampsia... (pg 68,blueprint textbook)

26. All of the following are true about PET except:


A. There‘s a decrease in plasma volume
B. + There‘s a decrease in peripheral vascular resistance
C. Uric acid elevation is the earliest lab sign
D. Thrombocytopenia is related
E. Elevated liver enzymes is a sign of serious complication

Explanation:

27. All of the following can be avoided with good ANC, except:
A. + Constitutional macrosomia
B. Syphilis
C. Hydrops fetalis
D. Prematurity

Explanation: constitutional synonym; inbred,inherent,built-


in,organic,inborn,ingrained

28. Gestational diabetes is associated with all of the following,


except:
A. + Postmaturity
B. Fetal death
C. Neural tube defects
D. Hypertension

Explanation: postmaturity= not even mentioned in diabetes and


pregnancy lect 5 yr

29. Regarding pre-eclampsia, all are true, except:


A. Recurrence rate is up to 10%
B. + Diazepam is used to prevent further convulsions

Explanation: the best either phenytoin or MgSo4# (hypertensive in


pregnancy lect thn 5)

30. Regarding IUGR, only one of the following is correct


A. + Can be caused by advanced diabetes
B. Can be caused by dieting during pregnancy
C. Occurs at 35 weeks gestation
D. Usually associated with polyhydramnios

Explanation: IUGR stricly related to advance DM (diabetes and


pregnancy lect 5yr)

31. All of the following can be detected by Ultrasound at 12 weeks


gestation, except:
A. Wrong date
B. Chorionicity
C. Triplets
D. + Growth restriction

32. All of the followings are part of Rh genetic subtypes, except:


A. D
B. + d
C. E
D. c
E. C

Explanation: C,D,E,c,e (no small d) (general idea under the topic


rhesus isoimmunization,lect 5 yr,DR laila)
33. Regarding diabetes in pregnancy, only one of the following is
correct:
A. The most common is type II
B. Same incidence of fetal anomalies compared to the population
C. + Glycosuria is unreliable in control assessment
D. Glycosylated hemoglobin level of 7-8 % is considered as
controlled case

Explanation:
50 % of normal healthy pregnant women have glycosuria sometimes

Not all DM,IGT,have persistent glycosuria

(refer lecture note diabetic and pregnancy page 5)

34. A lady who is 22-week gestational age is found to have a uterus


consistent with 26 weeks, all of the following are possible causes,
except:
A. Full bladder
B. Wrong date
C. Ovarian cyst
E. + Gestational diabetes

35. Regarding clomiphene citrate, all of the following are true


except:
A. Can cause headaches, blurred vision and hot flushes
B. Can be used in days 2-6 of menstrual cycle
C. + It is a cyclical steroid estrogen with anti-estrogenic activity
D. Can cause hyperstimulation syndrome

Explanation- clomid is a non-steroidal cyclical estrogen with an anti-


estrogenic activity

Source- lecture note infertility

36. All of the following are progesterone-only contraceptives,


except:
A. + Mifepristone
B. Implanon
C. Mirena
D. Norplant
E. Depo-provera

Explanation- Mifepristone is a low-efficacy partial agonist of


the progesterone receptor used as an abortifacient in the first
months of pregnancy, and in smaller doses as an emergency
contraceptive

Source- https://en.wikipedia.org/wiki/Mifepristone

37. What is the most common cause of spontaneous miscarriage?


A. + Chromosomal abnormalities
B. Infection
C. Cervical incompetence

Explanation- commonest cause of abortion about 50% to 70% of the


first trimester abortions are due to chromosomal abnormalities

Source- lecture note abortion

38. Regarding recurrent abortion, all are true except:


A. + Defined as three consecutive induced abortions
B. Anti-phospholipid is a cause

Explanation- defined as three or more consecutive spontaneous


abortion

Source- lecture notes abortion

39. Regarding threatened abortion, all are true except:


A. U/S is essential for diagnosis
B. Cervix is closed
C. It is the most common type (?)
D. Presents with mild vaginal bleeding
Explanation- Vacuum aspiration (the suction method) is the most
common type of abortion. It is a surgical procedure that involves
anesthesia (mild sedation) and can be performed within the first
trimester of pregnancy.

Source- http://www.pamf.org/teen/sex/pregnancy/abortion.html

40. Regarding menopause, all are true, except:


A. Vaginal and urethral epithelium are estrogen-dependant
B. + Collagen of bones and skin will not be affected by estrogen
deficiency
C. Hot flushes is the most common symptoms
D. Vaginal wall will atrophy and collapse…

Explanation- collagen content of the skin and bones are reduced


by estrogen deficiency

Source- slides menopause

41. Regarding uterine fibroids, all are true, except:


A. HRT is contraindicated in patients with fibroids
B. Nulliparity and obesity are known risk factors
C. Hysterectomy is a modality of treatment
D. Red degeneration typically presents in the second trimester
E. + Hyaline degeneration occurs when the fibroid outgrows its
blood supply

Explanation- hyaline degeneration occurs when muscle and fibers


are replaced by hyaline tissue with further reduction in blood supply

Source- slides fibroid


42. Regarding abortions, all are true, except:
A. Medical treatment is superior in second trimester abortions
B. + Subserosal fibroids are a known cause of abortion
C. Ovulation can occur as early as two weeks following abortion

Explanation- it should be submucous fibroid because it is more likely


to prevent implantation of embryo

Source- lecture note fibroid

43. All of the following surgeries are used to treat genital prolapse
except:
A. Anterior colporrhaphy
B. Manchester
C. Le Fort‘s
D. + Colposuspension
E. Sacrohysteropexy

Explanation- colposuspension is used for treatment urinary


incontinence

Source- lecture note genital prolapse

44. All of the following are used in treatment of PPH except:


A. Ergotamine
B. + MgSO4
C. Oxytocin

Explanation- magnesium sulphate is known as the cause for uterine


atony

Source- slides postpartum hemorrhage


45. Regarding HRT, all of the following are true, except:
A. + It‘s the first line for prevention of osteoporosis in
postmenopausal women
B. Is not considered in the prevention of coronary artery disease

Explanation- Postmenopausal hormone therapy is currently


recommended for the short-term management of moderate to
severe vasomotor flushes. Also in the absence of contraindications ,
it‘s used in urogenital atrophy. Long-term use for prevention of
diseases like cardiovascular diseases and osteoporosis is no longer
recommended.

Source- slides menopause

46. All of the following are raised physiologically in pregnancy,


except:
A. WBCs
B. + Platelets
C. Renal blood flow
D. Cardiac output

Explanation- there is no change in platelet count in pregnancy

Source- slides coagulation disorder

47. Regarding the reduction of incidence of preterm labor, all are


true except:
A. + Smoking cessation has no role

Explanation- smoking is one of the modifiable risk factors. so smoking


cessation has role in reduction incidence of preterm labor

Source- lecture note preterm and prom

48. Which of the following is the best to prevent PPH?


A. + Active management of third stage
B. Correction of anemia before pregnancy
C. Liberal use of blood transfusion
Explanation- active management reduces pph incidence from 15%
to 5%

Source- slides normal labor


49. Regarding preterm labor, all are part of the conservative
management, except:
A. + Frequent vaginal exam to assess cervical dilation
B. CTG to monitor the fetal heart
C. Ultrasound to monitor fetal weight and amount of liquor 10

Explanation-

Source- lecture note preterm and prom


50. Regarding PPROM, all are true, except:
A. Occur in 3% of pregnancies and is responsible for about one third
of preterm births
B. + Antibiotics have no role in treatment

Explanation- If chorioamnionitis is present, delivery is performed


regardless GA and therapeutic broad-spectrum IV antibiotics are
initiated

Source- lecture note preterm and prom

51. Regarding cervical incompetence, all are true except:


A. Ultrasound is essential for diagnosis
B. + Multiple pregnancy is a risk factor
C. Cone biopsy of the cervix is a risk factor
D. Best treated in early second trimester

Explanation- risk factors are congenital, prolonged labor,


instrumental delivery, macrosomic baby, traumatic cervix

Source- slides recurrent miscarriage

52. All of the following are associated with IUGR except:


A. Hypocalcemia
B. Hypoglycemia
C. Meconium aspiration
D. + Anemia

Explanation-

Source- http://www.webmd.com/baby/iugr-intrauterine-growth-
restriction
53. Regarding placenta previa, all are true except:
A. Associated with IUGR
B. Associated with primary PPH
C. Associated with doubling in incidence of fetal abnormalities
D. More in patients with previous uterine surgeries
E. + It can be diagnosed clinically

Explanation- Diagnosis by ultrasound scan showing that the


placenta coming in to the lower segment

Source- slides antepartum hemorrhage

54. All of the following are possible complications for induction of


labor, except:
A. Hyperstimulation
B. Precipitate labor
C. + Postpartum hemorrhage (?)

Explanation- induction of labor can increase the risk of pph

Source- handout induction of labor dr. Fayez Jallad

55. All of the following are indications for labor induction, except:
A. + Genital herpes
B. Chorioamnionitis
C. Rh-isoimmunization

Explanation- genital herpes is absolute contraindication for


induction of labor

Source- handout induction of labor dr. Fayez Jallad

56. All of the following are caused by endometriosis, except:


A. Cyclical hematuria
B. + Primary dysmenorrhea

Explanation- endometriosis cause secondary endometriosis

Source- slides antepartum hemorrhage


57. Regarding Turner syndrome, which of the following is true?
A. Low IQ
B. Karyotype 46 XO
C. +Presents with primary amenorrhea 11

Explanation- turners syndrome cause primary amenorrhea

Source- lecture note amenorrhea


58. All of the following are routine investigations for infertility except:
A. Progesterone at 21 days in a 28-day cycle
B. Semen analysis
C. Tests of tubal patency
D. Day 2 FSH, LH, testosterone, FBS
E. + Day 21 FSH and LH

Explanation- day 21, to detect the secretory changes in the


endometrial glands. We do now serum progesterone not FSH or LH

Source- lecture note infertility

59. Polycystic ovary syndrome patients may have all of the


following, except:
A. Hirsutism
B. + Low body mass index
C. High prolactin level
D. Subfertility
E. Oligomenorrhea

Explanation- it should be high body mass index (obesity)

Source- lecture note infertility

60. Regarding infertility, all are true, except:


A. + Secondary infertility is defined as only failure of pregnancy after
at least one term pregnancy (?)
B. Primary infertility is define as failure of pregnancy with no previous
pregnancies
C. Male factor infertility accounts for 40% of causes
D. Infertility is defined as failure to conceive after 12 months of
regular unprotected intercourse (not same words but it was the
correct definition)

Explanation- secondary infertility is subfertility with a previous


conception

Source- lecture note infertility


61. All of the following are mandatory in a case of urinary
incontinence, except:
A. Pelvic ultrasound
B. Bladder diary
C. Urodynamic studies
D. Careful history
E. Urinalysis

Explanation-

Source- slides urinary incontinence

62. A patient with imperforate hymen, what’s the typical


presentation?
A. + Cyclical lower abdominal pain with amenorrhea.
B. Short stature

Explanation- imperforate hymen cause cyclical lower abdominal


pain as typical presentation

Source- lecture note amenorrhea

63. Regarding the treatment of anemia in pregnancy, all of the


following are true, except:
A. 60 mg of iron is adequate dose in pregnancy
B. + Parenteral iron is used to correct anemia faster
C. 0.4 mg folate is required for all pregnant ladies as a minimum
D. Gastrointestinal side effects are the main cause of
noncompliance to therapy
E. With optimal treatment, average Hb rise is 0.8 g/dl per week 12

Explanation- in severe cases or in intolerable patient to iron tablets


we can give iron injection IV or IM

Source- lecture note anemia in pregnancy


64. Regarding breastfeeding, all are true, except:
A. Colostrum contains antibodies
B. Suckling stimulates oxytocin release
C. + Breastfeeding slows down uterine involution
D. Associated with less allergies in the child
E. Exclusive breastfeeding is essential to have amenorrhea for 6
months

Explanation- uterus involution is accelerated in breastfeeding


women by the effect of oxytocin

Source- slides puerperium

65. Regarding oxytocin, all are true, except:


A. Can cause hypotension
B. Is a nanopeptide
C. + Synthesized in the posterior lobe of pituitary
D. Oxytocin receptors in the uterus are upregulated by the end of
pregnancy
E. Has some antidiuretic activity

Explanation- Oxytocin is produced in the hypothalamus and is


secreted into the bloodstream by the posterior pituitary
gland. Secretion depends on electrical activity of neurons in the
hypothalamus – it is released into the blood when these cells are
excited.

Source- http://www.yourhormones.info/hormones/oxytocin.aspx
66. Regarding anemia in pregnancy, all are true except:
A. Folate deficiency is the second most common
B. + An MCV of 70 fL is diagnostic of iron deficiency anemia

Explanation-
IDA is the most common cause of anemia in pregnancy after
excluding the physiological anemia, because of :

1- depletion of iron stores to supply the fetus


2- decrease intake because of poor appetite
3- absorption delay because of changes in the hormonal levels
4- Chronic blood loss because of menstruation before the
pregnancy
5- Previous recent pregnancy, so there is no enough time to restore
the iron which depleted in the previous pregnancy
6- Conception while lactating

Source- slides anemia in pregnancy

67. What is the most common cause of coagulopathy during


pregnancy?
A. Dilutional coagulopathy
B. + Placental abruption
C. von Willebrand disease
D. Sepsis
E. Amniotic fluid embolism

Explanation- abruptio placenta is the commonest cause of DIC in


the parturient. Bleeding in these patients can largely be concealed
and retroplacental. Up to 30% of patients may develop
coagulopathy. Fluid resuscitation to correct hypovolemia and early
delivery to stop bleeding is essential.

Source- http://www.frca.co.uk/documents/coag_obs.pdf
68. All of the following used in the diagnosis of molar pregnancy
except:
A. Pelvic ultrasound
B. Serial beta hCG
C. Histologic examination
D. + Abdominal/Pelvic CT

Explanation- abdominal/pelvic CT used for metastatic work up

Source- slides gestational trophoblastic neoplasia

69. All of the following are true regarding hydatidifrom moles


except:
A. Complete mole has higher malignant potential than partial mole
B. Complete mole is 46 XX or XY
C. Partial mole is triploidy 69 XXY
D. May present with early PET
E. +70% of women with complete mole have hyperemesis

Explanation- Hyperemesis Gravidarum occurs in 25% of cases

Source- slides gestational trophoblastic neoplasia

70. Regarding gestational trophoblastic disease, all are true, except:


A. Unknown etiology
B. Diet may play a role
C. Results from defective fertilization
D. Most common complete mole is 46XX, partial 69 XXY
E. + Partial moles are more common

Explanation: The majority of hydatidiform moles are complete molar.


Source: lecture note GTN (Dr Esam Lataifeh) , page 2.

71. Which of the following is an epithelial ovarian tumor?


A. Fibroma
B. Endodermal sinus tumor
C. + Brenner

Explanation: Epithelial ovarian tumor:


-serous tumor
-mucinous tumor
-endometroid tumor
-clear cell tumor
-Brenner tumor
-cystadenocarcinoma
Source: lecture note Ovarian Cancer, page 2.

72. Regarding cervical ectropion, all are true except:


A. May present as postcoital bleeding
B. + May present in postmenopausal females
C. Cervical smear is mandatory before treatment

Explanation: Cervical ectropion occurs in the childbearing age, and


most frequently during pregnancy & in COC users, due to high
estrogen levels.
Symptoms: PCB
Cervical smear is mandatory before treatment to exclude cervical
cancer.
Source: slide Abnormal uterine bleeding, 17, (cervical ectropion.

73. Regarding epithelial ovarian tumors, all are true, except:


A. The most important prognostic factor is remaining malignant
tissue after surgery
B. + Hormonal presentation is common
C. Borderline types are treated with surgery with no need for
chemotherapy

Explanation: Hormonal presentation is common in sex cord-stromal


tumor.
Source: lecture note Ovarian cancer, page 18.

74. Regarding postmenopausal bleeding, all are true except:


A. It is always abnormal and requires investigation
B. Endometrial hyperplasia is seen in 5% of cases
C. + Endometrial cancer is seen in 40% of cases
D. Atrophic vaginitis, endometritis are the most common causes
Explanation: Endometrial cancer is seen in 10% of cases.
Source: lecture note Endometrial cancer (Dr Basel Obeidat), page
3.

75. Regarding endometrial cancer, which of the following is true?


A. Same incidence and prognosis worldwide
B. + Most cases present in early stages

Explanation: The commonest symptom is postmenopausal bleeding


(that‘s why mostly endometrial cancer diagnosed at early stage)
Source: lecture note Endometrial cancer (Dr Basel Obeidat), page
3.

76. All of the following are risk factors for endometrial cancer
except:
A. Late menopause
B. Nulliparity
C. + Combined oral contraceptive pills
D. Granulosa cell tumor

Explanation: COCP & progesterone have a protective effect


against endometrial cancer.
Source: lecture note endometrial cancer, page 2.

77. Regarding cervical cancer, all of the following are true except:
A. Abnormal vaginal bleeding is the most common presentation
B. False negative Pap smear is found in up to 50% of patients with
invasive cancer
C. + Radiotherapy is superior to surgery in early stage disease
D. Squamous cell carcinoma is the most common type

Explanation: For early invasive cervical cancer, surgery is the


treatment of choice. In more advanced cases, radiation combined
with chemotherapy is the current standard of care.
Source: http://emedicine.medscape.com/article/253513-treatment

78. A 30 year old with DUB, all are possible treatments except:
A. Mefanemic acid
B. Mirena
C. Combined OCP
D. + Endometrial resection and ablation

Explanation: Endometrial resection is the treatment for women more


than 40 years old.
Source: lecture note DUB (Dr Nael Obeidat), page 6.

79. Regarding menorrhagia, all are true, except:


A. Mostly dysfunctional
B. Commonly caused by adenomyosis in nulliparous women
C. Levonorgesterol-releasing IUD is an effective treatment in all age
groups

Explanation: most probably B, because adenomyosis


characteristically occurs in the grand multiparous woman.

80. All of the following are side effects of progesterone-only pills,


except:
A. Acne
B. Weight gain
C. Breast tenderness
D. + Complex ovarian cysts
E. Irregular bleeding

Explanation: The common side effects of progestogen-only methods


include:
-simple, functional ovarian cyst
-erratic or absent menstrual bleeding
-breast tenderness
-breast tenderness
-acne
Source: Gynaecology Ten Teachers, page 69.

81. All of the following contraceptive methods are matched


correctly to their failure rates, except:
A. COCP --- 1%
B. Depo-provera ---- 1%
C. + Implanon --- 1%
D. Progesterone only --- 2%

Explanation: failure rate:


Implanon- 0.1
COCP- 0.1-1
Progestogen only pill- 1-3
Depo provera- 0.1-2
Source: Gynaecology Ten Teachers, page 63.

82. Regarding UTI in pregnancy, all are true except:


A. E.coli is the most common cause
B. Asymptomatic bactirurea should be treated to decrease risk of
pyelonephritis
C. + Pyelonephritis is more common on the left side
D. Pyelonephritis needs 2-3 weeks of antibiotics
E. A repeat culture is needed to ensure successful treatment

Explanation: Pyelonephritis is more common on the right side.


Source: http://www.hindawi.com/journals/isrn/2012/519321/

83. Regarding sterilization surgeries, all are true except:


A. In Jordan, husband‘s consent is essential for female sterilization
B. + It‘s absolutely contraindicated to do surgery to a lady <28 years
C. Vasectomy won‘t lead to sterility in the first week after surgery

Explanation: It‘s not contraindicated to do surgery to a lady <28


years.

84. All of the following are essential to screen for in ANC except
A. Rubella titer
B. + TORCH infections
C. HBV
D. FBS

Explanation: Routine lab tests in ANC:


-Hb
-platelet
-blood group & Rh
-Blood sugar
-Rubella titer
-Hepatits screening
-Urine analysis
-antibody screening
Source: lecture note ANC (Dr Nael Obeidat)

85. Regarding preinvasive cervical lesions, all are true, except:


A. There‘s a 30% risk of cancer for high grade CIN after 10 years if left
untreated
B. Most low-grade lesions regress spontaneously within a year
C. Always arise from the transformation zone
D. + A pap smear from the ectocervix only is necessary

Explanation: Adequacy of cervical smear:


Adequate- contain cells from the ectocervix and endocervix.
Inadequate- contain 1 type of cell.
-In case of inadequacy, we repeat the smear after 3 months.
Source: clinical skills lab handout (cervical smear)

86. Regarding progesterone-only pills in the treatment of DUB, all are


true except:
A. May cause weight gain
B. The most widely used hormonal treatment
C. + More effective in ovulatory rather than anovulatory cycles
D. Can be used in the second half of the cycle
E. Reduce menstrual flow by 25%

Explanation: It‘s more effective in anovulatory rather than ovulatory


cycle.

87. Regarding ectopic pregnancy, only one of the following is true


A. Most cases are silent
B. Positive pregnancy test and empty uterine cavity on ultrasound
are diagnostic
C. Heavy vaginal bleeding
D. + Patient may have an intrauterine contraceptive device

Explanation: known etiological factor contributing to the risk of


ectopic pregnancy is: use of intrauterine device. In addition, most
cases are subacute & laparoscopy is used to diagnose and treat
ectopic pregnancy.
Source: Gynaecology Ten teachers, page 94.

88. Regarding ectopic pregnancy, all are true except


A. Negative pregnancy test excludes the diagnosis
B. + All types of IUD have similar risk of ectopic pregnancy

Explanation: Pooled data from clinical studies of different types of


IUD showed that the lowest risk of ectopic pregnancy was for users
of copper-bearing IUDs, and the higher risk was for users of
progesterone-releasing IUDs.
Source: http://www.ncbi.nlm.nih.gov/pubmed/3311627

89. Regarding ovarian tumors, all are true, except:


A. + Population screening is beneficial in prevention of ovarian CA
B. Most patients present in advanced stages
C. Most patients have vague, non-specific symptoms

Explanation: Population screening is beneficial in prevention of


cervical cancer.
Why we don‘t screen for ovarian cancer:
-no precancerous stage
-unknown natural course
Source: lecture note Ovarian cancer (Dr Basel Obeidat), page 10.

90. All of the following is true regarding endometriosis, except:


A. + Causes intermenstrual bleeding

91. All of the following are symptoms of overactive bladder, except:


A. + Passage of urine with sneezing

Explanation: Symptoms of overactive bladder


-urgency
-frequency
-large amounts of leakage
-nocturia
*Passage of urine with sneezing occur in stress incontinence.
Source: lecture note urinary incontinence (Dr Lama Al- Mahaisen)

92. All of the following make you suspect premature rupture of


membrane, except:
A. Positive nitrazine test
B. Positive fern test
C. + Uterine contractions on CTG
D. Visualizing the amniotic fluid …

Explanation: Prom is diagnosed by sterile speculum examination


meeting the following criteria:
-pooling positive-clear,watery amniotic fluid seen.
-Nitrazine positive-the fluid turns pH sensitive paper blue.
-Fern positive- the fluid displays a ferning pattern when allowed to
air dry on a microscope glass slide.
Source: Kaplan, page 74.

93. All of the following are contraindications for tocolytics, except:


A. + Preterm breech presentation
B. Chorioamnionitis
C. Uterine atony

Explanation: Contraindications to tocolytic;


In addition to drug-specific contraindications,[4] several general
factors may contraindicate delaying birth with the use of tocolytic
medications.

 Fetus is older than 34 weeks gestation [12]


 Fetus weighs less than 2500 grams or has intrauterine growth
restriction (IUGR)[12] or placental insufficiency[4]
 Lethal congenital or chromosomal abnormalities[4][12]
 Cervical dilation is greater than 4 centimeters[12]
 Chorioamnionitis or intrauterine infection is present[4][12]
 Mother has severe pregnancy-induced
hypertension,[12] eclampsia[12]/preeclampsia,[4] active vaginal
bleeding,[12] placental abruption,[4] a cardiac disease,[12]or
another condition which indicates that the pregnancy should not
continue.[12]
 Other cause of fetal distress or fetal death[12]
Source: https://en.wikipedia.org/wiki/Tocolytic
94. All the following regarding Estimated date is true except
a. + Estimates upto 38 weeks GA
b. Estimated from first day of LMP

Explanation:
- EDD by LMP is calculated by adding 280 days (40 weeks) to the first
day of the last menstrual period.
-Gestational age by LMP is calculated from the first day of the LMP.
Source: http://reference.medscape.com/calculator/estimated-
delivery-date-pregnancy

95. All are true regarding Endometrial CA except


a) + Stage 3 is confined to cervix

Explanation: Staging of endometrial cancer


Stage1- confined to the body of uterus
Stage2- involvement of cervix
Stage3-extension to ovaries and tubes
Stage4- distant mets
Source: lecture note Endometrial cancer, page 4.

96. Tumor secretes AFP


a) Endodermal Sinus tumor

Explanation: Tumor secretes AFP


-endodermal sinus tumor
-embryonal cell carcinoma
-polyembryoma carcinoma
Source: lecture note Ovarian cancer, page 14.

97. All are true regarding Ovarian tumors except


a) Epithelial tumors are characterized by hormone secretion

Explanation: Sex cord-stromal tumors are the one characterized by


hormone secretion. They often present with endocrine effects due
to excess secretion of estrogen & androgen.
Source: Gynaecology Ten Teachers, page 119.

98. Side effect of B sympathomimetics except


A) Tremor
B) + Premature closure of Ductus
Explanation: Premature closure of ductus arteriosus is a side effect
of prostaglandin synthetase inhibitor (indomethacin).
Source: Kaplan, page 73.

99) True regarding secondary dysmenorrhea except


a) Caused by endometriosis
b) + Ovulatory pain (something like that)

Explanation: Ovulatory pain is related with primary dysmenorrhea.

100) woman presented with 2 contractions/10 mins, cervix 4 cm …


a) + Not in labor

Explanation: effective contractions are;


-regular, minimum of 3 contractions per 10 minutes and lasting for 30
secons.
Source: handout clinical skills lab.

101) Woman presents 12 days after delivery with hemorrhage one is


correct
a) Its primary PPH
b) Start Abs immediately (might be this)

Explanation: Secondary postpartum hemorrhage occur between 24


hours and 6 weeks after delivery.
Etiology-retained placental tissue and endometritis.
In addition, Antibiotics should be given if placental tissue is found
even without evidence of overt infection.
Source: Obstetrics Ten teachers, page 261

102) Regarding puerperium all true except


a) peripheral vascular resistance increases right after delivery (not
sure, might be this)
b) vagina may never go back to original structure
c) 6 weeks postpartum

Explanation: The peripheral vascular resistance is usually increases or


it may remain unchanged.
Source:
http://www.kofinasperinatal.org/files/Cardiovascular_adaptational_
changes_in_pregnancy.pdf

Explanation: This period of puerperium is usually considered to be 6


weeks in duration.
The vagina also regresses but it doesn‘t completely return to its
prepregnant sizes.
Source: http://emedicine.medscape.com/article/260187-overview

103) Regarding Ovulation which is correct


A- + Preceded by LH surge

Explanation: Ovulation is preceded by LH surge.


6th Year Final Exam
2013 :
1. All are indications of Anti D except : Abdominal trauma
Fetal death
+ Oligohydramnios
Amniocentesis

Indications for Anti D:

1. Spontaneous or induced abortion


2. Invasive prenatal testing (amniocentesis, chorionic villous sampling)
3. Abdominal trauma
4. Antepartum haemorrhage
5. Ectopic pregnancy
6. External cephalic version.

Source: Rhesus Isoimmunization slides

2. What layer of ovum prevents the penetration of acrosomal enzyme of sperm of


other species?
+ Zona pellucid

As the sperm approaches the zona pellucida of the egg, which is necessary for initiating
the acrosome reaction, the membrane surrounding the acrosome fuses with the
plasma membrane of the oocyte, exposing the contents of the acrosome. The contents
include surface antigens and numerous enzymes which are responsible for breaking
through the egg's tough coating and allowing fertilization to occur.

Source: https://en.wikipedia.org/wiki/Acrosome_reaction

3. Which one is wrong about hyperemesis gravidarum


Tripleoidy
Management by giving fluids
Can lead to wernicke’s encephalopathy
+ Begin in the end of first trimester

Most women have some nausea or vomiting (morning sickness), particularly during the
first 3 months of pregnancy. The cause of nausea and vomiting during pregnancy is
believed to be a rapidly rising blood level of a hormone called human chorionic
gonadotropin (HCG). HCG is released by the placenta.

Some women have extreme nausea and vomiting during pregnancy. This can happen in
any pregnancy, but is a little more likely if you are pregnant with twins (or more
babies), or if you have a hydatidiform mole.

Source: https://www.nlm.nih.gov/medlineplus/ency/article/001499.htm

4. All can occur in infant of diabetic mother except


Caudal regression syndrome
Cardiac defects Renal defects Hypocalcemia
+ Hypoplasia of islets cells of pancreas

Fetal Complications

1. Macrosomia and traumatic delivery


2. Delayed organ maturity (RDS)
3. Congenital malformation
Cardiovascular: Transposition of great artery, ventricular septal defect, aortic
coarction, arterial septal defect.
CNS: Anencephaly, holoprosencephaly, encephalocele
Skeletal n spinal: caudal regression
Genitourinary: Renal agenesis, ureteral duplication
Gastrointestinal: Anal atresia
4. IUFD
5. IUGR

Neonatal complication

1. Hypoglycemia
2. RDS
3. Hypocalcemia
4. Polycythemia

Source: DM n Pregnancy slides


5. FSH affect at which cells?
Ovarian theca
+ Ovarian granulosa

THECA CELLS to convert cholesterol to ANDROGEN.


GRANULOSA CELLS to convert androgen to ESTROGEN
(this mechanism called aromatization), and release inhibin & activin.

Source: Obs & Gyne Morning Sessions

6. All are contraindications for IOL except Contracted pelvis


Transverse lie Previous 2 CS
+ Vaginal prolapsed
Contraindications:
• Is contraindicated when the risk of vaginal delivery is unacceptable i.e. when
delivery by C/S is safer to the mother and/or fetus than the vaginal delivery.
A. Absolute :
1. Placenta praevia
2. Previous 2C/S, previous one due to recurrent cause, previous classical C/S
3. Abnormal antenatal CTG
4. Transverse or oblique lie.
5. Absolute contracted pelvis.
6. Active genital herpes infection.
7. Tumor occupies the pelvis
8. Cervical carcinoma
9. Successful pelvic floor repair and successful surgical treatment of stress
incontinence.
B. Relative:-
1. Severe pre-eclampsia
2. Breech presentation
3. Multiple pregnancy
4. Grand multipara
5. Polyhydramnios.
6. Presenting part above the pelvic inlet.
Source: Induction of labour and prolonged labour slide

7. All are routine investigations in ANC except Hepatitis screen


Rubella titer
+ Toxoplasma IgM

17
Routine lab tests:
1. Hb
2. Platelets
3. Blood group and Rh
4. Antibody screening
5. Urine analysis
6. Rubella titer
7. Hepatitis screening
8. Veneral disease research lab test (VDRL)
9. HIV
10. Blood sugar
11. Pap smear

Source: Antenatal Care Lecture

8. All are increased in AFP except Encephalocele


+Trisomy 21
+Turner’s
+ Trisomy 18

I think the question should be “All are decreased in Afp


except”

In Encephalocele.
-AFP increase in Ntd. Encephalocele is on type of NTD

Alpha fetoprotein:

gestation, fetal demise, liver tumor, NTD, ventral wall


defect.

fetal death, molar pregnancy.

In Turner Syndrome
-AFP decrease
-bHCG increased if hydrops present, decreased if no
hydrops
-Inhibin increasedif hydrops present, absent if no hydrops
In Trisomies 21,18,13,
-AFP decrease

9. All are increased in pregnancy except


Total thyroxin
+Fastingblood sugar
Cortisol
free t3 and T4 increase

Increased during Pregnancy


-Total Thyroxine (t4,t3)
-Cortisol
-TBG
-Post prandial glucose
-Glucose intolerance

Decreased during pregnancy


-Ca
-Albumin
Fasting Blood sugar
-Serum TSH

*You can refer full version in handout ‘Physiological


Changes in Pregnancy’. Mawjood bil package

10. All are true about cardiac diseases in pregnancy except

Pulmonary artery pressure monitoring decrease


Eisenmenger’s Rheumatic disease’s incidence is
decreasing
Commonest congenital heart disease is cushion defects
Eisenmenger’s brings bad prognosis

11. All are contraindications for IUCD insertion except


Uterine anomalies
PID
+ Emergency contraception
Irregular periods
Contraindications for IUCD

Absolute contraindications
 Pregnancy
 Current, recurrent or recent (within 3 M) PID or
sexually transmitted disease
 Puerperal sepsis
 Immediate post septic abortion
 Severely distorted uterine cavity
 Unexplained vaginal bleeding
 Cx or endometrial ca
 Malignant trophoblastic disease
 Copper allergy/Wilson disease, Copper -IUCD
 Breast ca  LNG -IUCD
Relative contraindications
 Risk factor for sexually transmitted diseases or HIV
 Increased susceptibility to infection (eg, those with
leukemia, diabetes, valvular heart disease, or AIDS,-
women on corticosteroid Rx
 48hrs- 4 wks postpartum
 Ovarian ca
 Benign gestational trophoblastic disease
 History of ectopic pregnancy
Source: Contraception slide

12. All are tocolytics drugs except


Ritodrine
Salbutamol
+ Diazepam
Indomethacin
CCB

We all know the Diazepam is not tocolytic drug.


In Ong,Diazepam we use to abort convulsion in Eclampsia
cases

Tocolytic drug;
-B2 agonist (Salbutamol, Terbutaline, Retodrine)
-Calcium Channel Blocker (Nifedipine)-this is the best
-Mgso4 (rarely used)
-Indomethacin
-Oxytoxin antagonist (Atosiban)-(very expensive and rarely
used)

13. All are true about adenomyosis except


Round globular uterine enlargement (Bulky uterus)
Located in myometrium
Common in multiparous women
+ Diagnosed by endometrial biopsy

• Diagnosis often histological on examination of


hysterectomy sample

Source: Endometrial slide


14. All are acute complications of radiotherapy except
Enteritis
+ Vesico-vaginal fistula
Bone marrow suppression
Proctosigmoiditis
Acute cystitis
15. All can cause genital prolapsed except
Forceps delivery
Prolonged labor
Hysterectomy
+ Caesarean section

Aetiology:

 Prolapse is due to weakening of and damage to the supporting structures of the pelvic organs which
may occur as a result of:

1. Childbirth :
 Childbirth is the most important risk factor in the development of prolapse .
 Factors which increase risk of trauma and denervation to the pelvic support include :increasing
parity, prolonged labour , bearing down before full cervical dilatation and difficult instrumental
deliveries.

2. Chronic elevation in intra-abdominal pressure:


 Factors which increase the intra-abdominal pressure include obesity, smoking chronic cough
chronic constipation ,heavy lifting at work , abdominal masses and ascites .

3. Menopause :which leads to weakness of the pelvic support due to the reduction in the amount of
collagen and weakness of the connective tissue

4. Pelvic surgery :
 Vault prolapse –which may occur after abdominal or vaginal hysterectomy
 Rectocele and enterocele –which may occur after colposuspension

5. Congenital prolapse , due to congenital reduction in the amount of collagen and weakness
of connective tissue of the pelvic support .

 Is responsible for the occurrence of prolapse in 2% of nulliparous women .

6. Racial variation .
 Prolapse is common in Caucasian women , less common in Asians , and rare in Blacks .
 This racial variation is explained by the variation in the amount of collagen and connective tissue
in the pelvic support . ie greater in Blacks and lesser in Caucasian

Source: Genital prolapse seminar


16. All indicate severity of PET except
Oliguria
Severe frontal headache
+ Alkaline phosphatase
5g/24 hours proteinuria

Indication for severe PET:


-Severe hypertension (>110/160)
-Heavy protein urea at least 5 g/24 hr urine collection or +3 on dipstick on 2 occasions separated by at
least 4 hours

-Oligurea(< 500 ml / 24 hrs ) or Elevated serum creatinineconcentration (>1.1mg/dL)

-Thrombocytopenia(<100,000/microL)
-Persistentand/orsevere headache

-Visual abnormalities (scotomata, photophobia, blurred vision, or temporary blindness [rare])

-Nausea, vomiting, upper abdominal or epigastricpain


-Elevated liver enzymes
-Dyspnea, retrosternalchestpainor cyanosis, with pulmonary edema signs
-Fetal growth restriction, placental abruption, and oligohydramnios
Source: Management of patients with severe PET and eclampsia slide

17. All are causes of cord prolapsed except


Anencephaly
+ Post-maturity

RISK FACTORS AND ETIOLOGY


**Fetal malpresentation:

Prematurity/lowbirth weight

Multiple gestation

**Rupture of membrane

Multiparty
Polyhydramnios

Iatrogenic causes

Source: Management of obstetric emergencies

18. 30 yr old woman with CIN III by pap smear and currette I think. Next step:
+ LEETZ
Hysterectomy repeat in 6 months

Source: Management of patients with abnormal cervical smear

19. all of this not tocolytic except


+ diazepam
-endomethcin
-salbutamol

Repeated Question. Refer question no 12 for explanation


20. all of this c/i for s/c ex
+ vaginal prolapse "
-contracted pelvic

21. about of turner syndrom all this true ex


+ syctic hymngma "
-45 xo

Im not sure about the answer given.


I think the answer given is wrong.

Cystic Hygroma yes, are seen more often with genetic disease, which include
-Turner Syndrome
-Trisomies
-Noonan Syndrome

Some info on Turner Syndrome

 Short stature
 Lymphedema (swelling) of the hands and feet
 Broad chest (shield chest) and widely spaced nipples
 Low hairline
 Low-set ears
 Reproductive sterility
 Rudimentary ovaries gonadal streak (underdeveloped gonadal structures that later become
fibrotic)
 Amenorrhoea, the absence of a menstrual period
 Increased weight, obesity
 Shortened metacarpal IV
 Small fingernails
 Characteristic facial features
 Webbed neck from cystic hygroma in infancy
 Aortic valve stenosis
 Coarctation of the aorta
 Bicuspid aortic valve
 Horseshoe kidney
 Visual impairments - sclera, cornea, glaucoma, etc.
 Ear infections and hearing loss
 High waist-to-hip ratio (the hips are not much bigger than the waist)
 Attention deficit hyperactivity disorder (problems with concentration, memory, attention with
hyperactivity seen mostly in childhood and adolescence)
 Nonverbal learning disability (problems with math, social skills, and spatial relations)
22. About PCOS all are true except:
+ Progesterone test is -ve
Common in reproductive age
most women have insulin resistance
weight loss is a good treatment

Progesterone test (Prostin Challenge) in Pcos, will give positive result (withdrawal
bleeding)
It suggest Anovulation (PCOS)

23. All are needed to perform forceps delivery except:


+ fetus head palbable 3/5 in abdomen
ROM
fully dilated cervix
adequate pelvis

Fetal head has to be engaged in order to perform forceps deliveries which is 2/5
fetal head palpable abdominally. (Not 3/5. 3/5 and above, considered abdominal,
while 2/5 and below considered pelvic organ-engaged)

Below are things needed for performing assisted deliveries:

Requisitions of assisted vaginal delivery

- Complete cervical dilation & rupturing of membranes


- Well-trained obstetricians
- Baby is alive
- knowing fetal position & presentation which must be cephalic
- Engagement of the presenting part
- Proper analgesia
- Easy access to the operating theatre, or do it in the OR; to do C/S in case of
assisted VD failure.
- Empty urinary bladder

24. All are specified by 1000 except:


+ Maternal mortality
fetal mortality
fetal morbidity
fetomaternal mortality

Maternal mortality is maternal death per 100 000 live births.

According to WHO,
Maternal death is the death of a woman while pregnant or within 42 days of
termination of pregnancy, irrespective of the duration and site of the pregnancy,
from any cause related to or aggravated by the pregnancy or its management but
not from accidental or incidental causes. To facilitate the identification of maternal
deaths in circumstances in which cause of death attribution is inadequate, a new
category has been introduced: Pregnancy-related death is defined as the death of a
woman while pregnant or within 42 days of termination of pregnancy, irrespective
of the cause of death.

Live birth refers to the complete expulsion or extraction from its mother of a
product of conception, irrespective of the duration of the pregnancy, which, after
such separation, breathes or shows any other evidence of life - e.g. beating of the
heart, pulsation of the umbilical cord or definite movement of voluntary muscles -
whether or not the umbilical cord has been cut or the placenta is attached. Each
product of such a birth is considered live born.
25. Most common malignant germ cell tumor in ovaries:
+ dysgerminoma
other stuff

Dysgerminoma
• Is the most common malignant germ cell tumor
• Can arise at any age but the majority of cases (75%) arise in adolescents and
young adults, in whom they account for about one-third of all ovarian
malignant neoplasms.
• Because of their predilection for young women, they are one of the more
common ovarian malignant neoplasms detected during pregnancy
• 75% of women with dysgerminomas present with stage I disease; the
contralateral ovary is involved in 10-15%. Bilateral ovarian disease is more
common with dysgerminoma than with any other malignant OGCN.
(slide Ovarian Tumor)

26. The single most common cause of fetal death:


Asphyxia
IUGR stuff
There was either two options ..1prematurity..2 infections but it was two different
questions,

27. All can be prevented by good ANC except:


+ Placenta previa
Macrosomia
Infection

Things that can be prevented or mini mized by good ANC (from ANC lec note)
-Anemia
-UTI
-Pregnancy Induced HTN
-Preterm labor and delivery
-IUGR
-STDs
-Rh Isoimmunization
-Fetal Macrosomia
-Hypoxia and Fetal death due to post term
-Breech at term

28. All cause nephrotoxicity and bladder toxicity except:


+ bleomycin…it causes lung fibrosis.
cisplatin
cyclophosphomide
ishy-mide
carbo-ishy

The following side effects are common (occurring in greater than 30%) for patients
taking Bleomycin:

-Fever and chills

-Skin reactions: redness, darkening of the skin, stretch marks on the skin, skin
peeling, thickening of the skin, ulceration

-Nail thickening, nail banding

-Hair loss

These side effects are less common side effects (occurring in about 10-29%) of
patients receiving Bleomycin:

Nausea and vomiting.

Poor appetite and weight loss.

Mouth sores.

Lung problems: pneumonitis, rarely pulmonary fibrosis. The incidence of lung


problems increases with age and pre-existing lung conditions. There is a maximum
lifetime dose of Bleomycin. Your health care professional will monitor the amount
of Bleomycin you receive as well as your lung function during treatment.

Occasionally this can cause "radiation recall" effect.

29. About HRT all are true except:


c/i in breast ca
c/i in DVT
first sign that its working is increase in appetite
causes endometrial hyperplasia

Absolute contraindication of HRT


 Suspected pregnancy
 Breast cancer
 Endometrial cancer
 Active liver disease
 Uncontrolled Hypertension
 Known VTE
 Known Thrombophilia.
 Otosclerosis
Relative contraindication of HRT
 Uninvestigated abnormal bleeding
 Large uterine fibroids
 Past history of benign breast disease
 Past history of VTE
 Chronis stable liver disease
 Migraine with aura

30. About CS which is wrong: Always requires general anasthesia


+ vertical and lower segmental are called classical
vertical is more painful than lower segmental

-- Classical incision-- is a longitudinal incision in the anterior fundus, not low vertical or lower
segmental. Cs always require anesthesia but not necessarily Ga, it could be Regional and Local
Anesthesia. Vertical incision in Cs is painful, High risk of bleeding, Rupture, adhesions and other
complications

31. about local anasthesia which is wrong:


Epidural causes headache
+spinal causes hypertension
local infection is a c/i
meningitis is a complication

Spinal anesthesia complication;


-hypotension
-post spinal headache
-urinary retention
-epidural or subarachnoid hematoma
-spinal cord trauma or infection

32. Thyroid disease which is wrong:


Most common pre-exisisting endocrine disorder of pregnancy
fetus need mother thyroxine for brain development
+ maternal total and free t4 is elevated

There is a fall in TSH and a rise in free T4 concentrations in the first trimester of normal pregnancy,
followed by a fall in free T4 concentration with advancing gestation

33. About contraceptive surgery (msh hek kan bs b m3na) all are true except:
+ tubal ligation is absolutely c/i in a woman below 28 yrs
in jordan a husbands consent is needed to do a tubal ligation
vasectomy is ineffective in first 2 weeks
they are reversible

-After vasectomy, remnant sperm remains in the ejaculatory ducts.

The man is not considered sterile until he has produced sperm-free ejaculates as documented by
semen analysis. This usually requires 15-20 ejaculations.

-both female and male sterilization procedures can be reversed surgically, but the surgery is
technically more difficult than the original procedure and may not be successful

34. About physiological anemia in pregnancy which is wrong:


+ diagnosed by MCV < 85

Physiological changes in Pregnancy


 Progressive increase in plasma volume up till 32-34 weeks, (50%)
 Progressive increase in Red cell mass, although the pregnancy, (25%)
 Max physiological anemia occur at 32-34 weeks gestation.
 MCV, MCHC stay constant, i.e. dilutional anemia
 Proressive fall in platelet count, low platelets only if Platelets are < 100 or pathologically
reduced count. 5-10% will be 100-150*109 /l
 There is 2-3 fold increase in iron requirements in pregnancy.
 Hypercoagulable state.

35. A woman comes in 34 weeks pregnant diagnosed with breech, she's stable, whats your next step:
+ Come again in 2 weeks
ECV
C/S
IOL
Nullipara woman usually at 37 weeks the baby will be invertex position. Multipara woman, the baby
can change position to vertex before onset of labor

36. A woman who is 34 k2no weeks pregnant comes with mild vaginal bleeding and stable vitals, on
U/S she has placental abruption, what's your next step:
deliver C/S
tocolytics
fluid support

How to manage this case:

-Admission.

- Blood workout and cross- matching

- Steroids is not indicated unless gestational age less than 34 weeks GA.

- Strict observations may change to severe.


- Twice daily CTG

- Ressuscitation, IV canula ,IV crystalloid

- Cross match blood and FFP.

- Assessment of mother, put fixed catheter, CBC ,KFT (because of early involvement of the kidney,
catheter for urine output measurement Urine for protein, coagulation profile: (thrombocytopenia is
the earliest indicator of DIC ,

plts count < 100,000 / microL= thrompocytopenia.

Plts < 50,000 bleeding from any site of injury.

Plts < 20,000 bleeding from any site.

- Assessment of fetal wellbeing, CTG. If the baby is distressed, aim for immediate delivery.
37. A woman with 8 weeks ammenoria comes with vaginal bleeding and passage of clots and lower
abdominal cramps, on U'S there was a 12 mm empty sac, the ddx can be all of the following except:
+ threatened abortion
Complete abortion
missed abortion
molar pregnancy

Threatened abortion, usually comes with mild vaginal bleeding, with or without abdominal cramps.
On ultrasound, will show presence of fetal heart activity

38. Which is wrong about bacterial vaginosis:


most common bacterial infection
foul fishy discharge
mostly asymptomatic
+ usually theres a local inflammatory process
best treatment is metrinidazole

• Vaginitis (Candidal Vaginitis): significant inflammatory process in vaginal wall.


Accompanied by high number of leukocytes in vaginal fluid. Found with candida and
trichomonas infections.

• Treatment-Vaginal antifungal creams:


- Butaconazole, Clotrimazole, Miconazole

• Bacterial Vaginosis: minimal inflammatory response with few leukocytes in vaginal wall.
Associated with increase in bacterial concentrations.
• Treatment- Metronidazole and Clindamycin (iv and oral)

39. The layer of ovum that prevents fertilization by other species is:
+ Zona pellucida (correct)
- theca interna

Refer question no 2. same


40. During antenatal care which is not a routine investigation
+ toxoplasmosis #
- platelets

- rubella

Routine Test in First anc (refer ANC lec note)

 HB
 Platelets
 Blood Group and Rh
 Antibody Screening:
 Urine analysis:
 Rubella Titer
 Veneral Disease Research Laboratory test (VDRL
 HIV Screening:
 Blood Sugar:
 Pap smear

41. In Endometriosis all are correct except:


- CA 125 can be used for SCREENING (I think this is the answer)

No screening for endometriosis.


Ca 125, we use it in investigation of Endometriosis. It will be elevated
Endometriosis Investigation
• Ca 125 often elevated
• Ultrasonography for ovarian cyst
• MRI

42. About turner syndrome all are true except:


- associated coarctation of the aorta
+increased incidence with increased maternal age
-subnormal intelligence
- female phenotype perhaps m not sure if i mixed it with another question
-May have cytic hygroma

Advanced maternal age is not a risk factor for Turner syndrome

Some info on Turner Syndrome

 Congenital heart disease is a frequent feature of Turner syndrome. Although the most
frequent cardiac lesion is coarctation of the aorta, a spectrum of cardiac defects occurs
which is limited almost exclusively to defects associated with decreased blood flow through
the left heart.
 Short stature
 Lymphedema (swelling) of the hands and feet
 Broad chest (shield chest) and widely spaced nipples
 Low hairline
 Low-set ears
 Reproductive sterility
 Rudimentary ovaries gonadal streak (underdeveloped gonadal structures that later become
fibrotic)
 Amenorrhoea, the absence of a menstrual period
 Increased weight, obesity
 Shortened metacarpal IV
 Small fingernails
 Characteristic facial features
 Webbed neck from cystic hygroma in infancy
 Aortic valve stenosis
 Coarctation of the aorta
 Bicuspid aortic valve
 Horseshoe kidney
 Visual impairments - sclera, cornea, glaucoma, etc.
 Ear infections and hearing loss
 High waist-to-hip ratio (the hips are not much bigger than the waist)
 Attention deficit hyperactivity disorder (problems with concentration, memory, attention
with hyperactivity seen mostly in childhood and adolescence)
 Nonverbal learning disability (problems with math, social skills, and spatial relations)

19

43. Effect of renal disease during pregnancy all is true except:


+ azathioprine is not teratogenic
- can affect pregnancy outcome ? sthg like that

Azathioprine, is teratogenic and cause various birth defects

44. A case of suspected ectopic pregnancy (b-hcg 10000 and no intrauterine sac), patient is stable
no bleeding. What to do?
+ laparascopy
- serial b –hcg

This is the most probable answer because serial b-hcg needs to be done to
confirm ectopic pregnancy

45. Regarding recurrent miscarriage which one is wrong?


- the number of previous miscarriages affect the probability of pregnancy

The probability of pregnancy is not affected by number of previous miscarriages

46. All characteristics of bacterial vaginosis except:


+ Local inflammatory reactions
- fishy odor

Refer question 38
47. All are ACUTE complication of pelvic irradiation except:
- enteritis
-proctitis
+ uretrovaginal fistula

48. Inhibin is secreted from which ovarian tumor?


+ Granulosa cell tumor

Tumor markers used in ovarian cancer include:

CA-125

 Used to help in diagnosis, assessment of response to treatment, and


monitoring for recurrence.

Alpha-fetoprotein (AFP)

 Used in ovarian germ cell tumors.


 Used to assess stage, prognosis, and response to treatment.

Beta-hCG (Beta Human Chorionic Gonadotropin)

 Used in ovarian germ cell tumors.


 Used to assess stage, prognosis, and response to treatment.

HE4
 HE4 can be used to help diagnose ovarian cancer in a woman with
symptoms and may be more sensitive than CA125.
 It can also be used to assess response to treatment and monitor for
recurrence.

Inhibin A & B

 A hormone that is normally produced by ovarian tissue, but may be


elevated in certain types of ovarian cancer (mucinous epithelial
carcinoma, granulosa cell tumors).
 Can be used to assess response to treatment and monitor for
recurrence.

CEA (Carcinoembryonic antigen)

 Can be elevated in ovarian tumors and may be used to evaluate


response to treatment.

49. Androgen insensitivity syndrome which one is wrong: - karyotype is XY


- Female phenotype
+ Normal uterus
-normal testosterone level

AIS , does not have uterus. They lack of androgen receptor function.
Without androgen stimulation, internal wolfian duct structures atrophy.
With testicular Mullerian Inhibitory Factor present, the mullerian duct
derivatives involute. No nuterus formed.

50. All of the following are true about hyperemesis gravidarum except: -
mainstay of treatment is hydration
- in severe cases associated with Wernicke's encephalopathy
+ more during 2nd and 3rd trimester

Its usually during first trimester

51. Which is wrong about recurrent miscarriage:


+ cervical insufficiency happens in 1st trimester
LMW heparin and aspirin might help it happens in 1%
Mostly unknown etiology
Cervical insuffiency is one of the causes of 2nd trimester miscarriage. To manage, we do cervical
cerclage at 14th week

52. Regarding multiple pregnancies, all the following statements are


correct except:
- increase risk of placenta previa
- High order multiples are mainly due to assisted reproductive technologies
- Disseminated intravascular coagulopathy is common in case of single fetal
demise

All are true


53 . A32-year-old woman, gravida 2,para2, comes to the physician for follow-up of an abnormal Pap
test. One month ago, her Pap test showed a high-grade squamous intraepithelial lesion (HGSIL).
Colposcopy demonstrated acetowhite epithelium at 2 o'clock. A biopsy taken of this area
demonstrated HGSIL. Endocervical curettage (ECC) was negative. The patient has no other medical
problems, has never had cervical dysplasia, and takes no medications. Which of the following is the
most appropriate next step in management?
a. Repeat Pap test in I year
b. Repeat Pap test in 6 months
c. Repeat colposcopy in 6 months
d. + Loop electrode excision procedure (LEEP)*
e. Hysterectomy

54. Regarding cervical intraepithelial neoplasia (CIN), All the following are correct except
a. High Risk HPV fpes (16, 18, 31,33,or 35), are associated with high-grade cervical lesions (CIN2, 3)
and cervical cancer
b. Low risk HPV type (6 and i 1), are associated with low grade cervical lesions (CIN1, Ccndylomata
acuminata)
c. + At least 70 %f patients with CIN3 will develop invasive cancer within l0 years*
d. Low grade lesions CIN1 may spontaneously regress
e. Cone biopsy is adequate treatment for high grade cervical lesions CIN3
== About 20% will progress to invasive cancer if left untreated for several years ==

55. Regarding cervical erosion (ectropion), All the following statements are correct except
a. It is due to eversion of the columnar epithelium
b. Best treatment method is cryotherapy
c. + It is an ulcer*
d. Treated conservatively during pregnancy e. It is common in combined pill users

Cervical ectropion (or cervical eversion) is a condition in which the central (endocervical) columnar epithelium
protrudes out through the external os of the cervix and onto the vaginal portion of the cervix, undergoes
squamous metaplasia, and transforms to stratified squamous epithelium.[1] Although not an abnormality, it is
indistinguishable from early cervical cancer; therefore, further diagnostic studies (e.g., Pap smear, biopsy)
must be performed for a differential diagnosis.

== so its not an ulcer but metaplasia ==

56. Regarding warfarin, All the following statements are correct except
a. Is teratogenic
b. Monitored by prothrombin time
c. Crosses the placenta
d. + Contraindicated during lactation*
e. Specific antidote is fresh frozen plasma

57. Complications of post maturity include AII the following except


a. Prolonged labor
b. Oligohydramnios
c. Intrauterine fetal death
d. + Respiratory distress syndrome*
e. Meconium aspiration syndrome

• Complications:
A. Placental insufficiency and hypoxia which leads to:
1. Increased perinatal mortality (PNM): the PNM is doubled for
each week after 42 weeks.
2. Meconium aspiration syndrome.
3. Oligohydramnios and cord compression
B. Increased fetal weight and ossification of skull with decreased moulding, which leads to:
1. Prolonged labour and failure to progress which leads to ↑ incidence of C/S.
2. Shoulder dystocia – with its neonatal & maternal risks.
B. Maternal risks: vaginal & cervical lacerations & rupture uterus.
C. Neonatal risks:
1. neonatal asphyxia & death.
2. cervical cord injury
3. brachial plexus injury: Erb's palsy (injury to C5&6), Klumpk's palsy (injury to
C8&T1) and Phrenic nerve injury (injury to C4)
4. clavicular & humeral fractures.
58. Principles of management of septic abortion include All the following except
a. + Immediate evacuation of the uterus*
b. lntravenous antibiotics
c. Correction of hypovolaemia
d. Cervical swab for culture
e. Blood grouping and cross matched (2 units of blood)

Management :
1- Investigations :
 CBC , blood grouping , XM 2 units of blood .
 Cervical swabs (not vaginal) for culture and sensivity
 Coagulation profile , serum electrolytes & blood culture if pyrexia > 38.5
2- Antibiotics : Cephalosporin I.V + Metronidazole I.V
3- Surgical evacuation of uterus - usually 12 hrs after antibiotic therapy ( until a reasonable
tissue levels of antibiotics have been achieved )
4- Post-abortion management

59. The following chemotherapeutic agents are associated with adverse effects to the kidney
and urinary tract except
a. + Chlorambucil*
b. Cyclophosphamide
c.Ifosfamide
d. Cisplatin
e. Carboplatinum

Side effects
Bone marrow suppression (anemia, neutropenia, thrombocytopenia) is the most commonly
occurring side effect of chlorambucil. Withdrawn from the drug, this side effect is typically
reversible. Like many alkylating agents, chlorambucil has been associated with the development
of other forms of cancer.

Less commonly occurring side effects include:

 Gastrointestinal Distress (nausea, vomiting, diarrhea, and oral ulcerations).


 Central Nervous System: Seizures, tremors, muscular twitching, confusion, agitation, ataxia, and
hallucinations.
 Skin reactions
 Hepatotoxicity
 Infertility
 Hair Loss

60. Regarding follicle-stimulating hormone (FSH), only one of the following statements is
correct
a. + Is responsible for oestradiol production from the granulosa cells*
b. Brings about follicular rupture
c. Is raised in polycystic ovary syndrome == LH is raised with LH:FSH ratio of 3:1 instead of
normal 1:1 ==
d. Is necessary for the initial stages of embryo development
e. Is necessary for maintenance of the corpus luteum == in early pregnancy, corpus luteum is
maintained by b-hcg which is secreted by trophoblast until 12 weeks ==
6th Year Final Exam 2012:
1. organic causes of menorrhagia include all of the following except:
a. PCOS
b. endometrial hyperplasia
c. endometrial cancer
d. copper releasing IUCD
e. fibroid

Explanation :
Organic causes
Organic causes of menorrhagia include infection, bleeding disorders, and
organ dysfunction. Note the following:
 Infections can be of any genitourinary origin. The aforementioned
sexually transmitted diseases are of greater concern in the teenage and
early adult population. Bleeding from the urethra or rectum always must
be considered in the workup, especially in the postmenopausal woman
who has negative findings after a workup for vaginal bleeding.
 Coagulation disorders can evade diagnosis until menarche, when heavy
menstrual bleeding presents as an unrelenting disorder. These include
von Willebrand disease; factor II, V, VII, and IX deficiencies; prothrombin
deficiency; idiopathic thrombocytopenia purpura (ITP); and
thromboasthenia.[20] See more on bleeding disorders below.
 Organ dysfunction causing menorrhagia includes hepatic or renal
failure. Chronic liver disease impairs production of clotting factors and
reduces hormone metabolism (eg, estrogen). Either of these problems
may lead to heavy uterine bleeding.

Endocrine causes
Endocrine causes of menorrhagia include thyroid and adrenal gland
dysfunction, pituitary tumors, anovulatory cycles, PCOS, obesity, and
vasculature imbalance. Note the following:
 Both hypothyroidism and hyperthyroidism result in menorrhagia. Even
subclinical cases of hypothyroidism produce heavy uterine bleeding in
20% of patients. Menorrhagia usually resolves with correction of the
thyroid disorder.[19]
 Prolactin-producing pituitary tumors cause menorrhagia by disrupting
(GnRH) secretion. This leads to decreased LH and FSH levels, which
ultimately cause hypogonadism. Interim stages of menorrhagia result
until hypogonadism manifests.
 The most common etiology of heavy uterine bleeding is anovulatory
cycles. The finding of menorrhagia at irregular intervals without any
known organic etiology confirms the clinical diagnosis. This is most
common in adolescent and perimenopausal populations.
 The hallmarks of PCOS are anovulation, irregular menses, obesity, and
hirsutism. Insulin resistance is common and increases androgen
production by the ovaries.
 Hyperinsulinemia is a direct consequence of obesity. This overproduction
of insulin leads to ovarian production of androgens, as occurs in PCOS.
 Vasculature imbalance is theorized to be the result of a discrepancy
between the vasoconstricting and aggregating actions of prostaglandin
F 2 (alpha) and thromboxane A 2 and the vasodilating actions of
prostaglandin E 2 and prostacyclin on the myometrial and endometrial
vasculature.
Anatomic causes
Anatomic etiologies for menorrhagia include uterine fibroids, endometrial
polyps, endometrial hyperplasia, and pregnancy. Note the following:
 Fibroids and polyps are benign structures that distort the uterine wall
and/or endometrium. Either may be located within the uterine lining, but
fibroids may occur almost anywhere on the uterus.
 The mechanism by which endometrial polyps or fibroids cause
menorrhagia is not well understood. The blood supply to the fibroid or
polyp is different compared to the surrounding endometrium and is
thought to function independently. This blood supply is greater than the
endometrial supply and may have impeded venous return, causing
pooling in the areas of the fibroid. Heavy pooling is thought to weaken
the endometrium in that area, and break-through bleeding ensues.
 Fibroids located within the uterine wall may inhibit muscle contracture,
thereby preventing normal uterine attempts at hemostasis. This also is
why intramural fibroids may cause a significant amount of pain and
cramping. Fibroids may enlarge to the point that they outgrow their
blood supply and undergo necrosis. This also causes a great deal of pain
for patients.
 Endometrial hyperplasia usually results from unopposed estrogen
production, regardless of the etiology. Endometrial hyperplasia can lead
to endometrial cancer in 1-2% of patients with anovulatory bleeding, but
it is a diagnosis of exclusion in postmenopausal bleeding (average age
at menopause is 51 y). If a woman takes unopposed estrogen (without
progesterone), her relative risk of endometrial cancer is 2.8 compared to
nonusers. [21]
Iatrogenic causes
Iatrogenic causes of menorrhagia include IUDs, steroid hormones,
chemotherapy agents, and medications (eg, anticoagulants). Note the
following:
 IUDs can cause increased menstrual bleeding and cramping due to
local irritation effects.
 Steroid hormones and chemotherapy agents disrupt the normal
menstrual cycle, which is restored easily upon cessation of the products.
 Anticoagulants decrease clotting factors needed to cease any normal
blood flow, including menses. This type of menorrhagia also is easily
reversible.

Sources : http://emedicine.medscape.com/article/255540-clinical#b5

2.About Contraception failure rates, all true except :


a. POP 2%
b. COC 1%
c. depo.provera 1%
d. implanon 1%
e. mirena 0.5% Cant recall the % :/:/.. check the lec..

Explanation :
Failure rate of
1. COCP : 1-2%
2. Depot Medroxyprogesterone acetate (Depot-Provera) < 1% (0.5%)
3. Implanon : 1%
4. POP : 0.5% (From Hackers pg 306)
5. Copper IUD : 0.6 (From Hacker pg 306)

Sources : Lecnote Family Planning & Contraception also Hacker textbook


pg 306
3. About lower uterine segment .. one is true:
a. bladder is posterior
b. covered by loose vesciral peritoneum .
c. round ligament from its upper part
d. crossed by the ureters e. inferior epigastric artery are lateral to it

Explanation :
Sources :

4. Monozygotic twins .. all true except :


a. mainly diamniotic . (supposedly monoamniotic)
b. increased risk of C/S
c. more congenital heart disease
d. are 1/3 of all twins

Explanation :
According to the time when division occurs
1. First 74 hours : diamniotic dichrorionic (30%)
2. 4-8 days : diamniotic monochorionic ( 69%)
3. 8-12 days : monoamniotic monochorionic (1%)
4. After day 12 : Conjoined

Sources : Lecnote multiple pregnancu by Dr Omar Hussein

5. About female sterilization .. all are true except :


a. is absolutely contraindicated if <28 yrs old
b. needs a consent from the husband

Explanation : Not contraindicated, only less effective in younger woman


(upto 5.4% failure rate in woman<28years old). Fallopian tube procedure
are the most commonly done procedure.

Sources : Hacker page 312

6. regarding C.section, all of the following are true except :


a. midline incision is more painful than the lower segment transverse
b. bladder injury is more common than uretric injury
Explanation :
Sources :

7. All are fertility sparing treatment for fibroid except:


a. LHRH analogue
b. myomectomy
c. Uterine artery embolization
d. COC
e. mifepristone (RU 486.

Explanation : Uterine artery embolization (UAE) is a procedure where


an interventional radiologist uses a catheter to deliver small particles that
block the blood supply to the uterine body. The procedure is done for the
treatment of uterine fibroidsand adenomyosis

Some adverse effects are like it may cause infertility, loss of ovarian
function and loss of orgasm.

Source :
https://en.wikipedia.org/wiki/Uterine_artery_embolization#Adverse_effect
s

8. female with pap smear the result is LSIL , next step :


a. Colposcopy
b. Referral in 6 months

Explanation :
Referral criteria for colposcopy -
1. Any smear showing borderline nuclear changes or mild dyskaryosis with
high risk HPV
2. Any smear showing moderate or severe dyskaryosis
3. Any smear suggesting malignancy
4. Any smear suggestive of glandular abnormality
5. 3 consecutive inadequate smears
6. Keratinizing cells (underlying CIN?)
7. Post-coital bleeding
8. Abnormal-looking cervix

Source : Oxford Handbook of ObsGyn pg 707


9. Couple, infertility , tall male with small testes. Semen analysis:
oligospermia, next step:
a. Karyotype
b. Fsh level
c. Prolactine

Explanation : In azopermia + oligospermia you should also test for FSH, LH,
prolactin and karyotyping. In case of Klinefelter system, both small testes
and infertility are indicative. The standard diagnostic method is the
analysis of the chromosomes' karyotype on lymphocytes

Source : https://en.wikipedia.org/wiki/Klinefelter_syndrome#Diagnosis

10. About recurrent miscarriages all are true except :


a. screening for toxoplasmosis is recommended
b. screening for lupus anticoagulant is recommended
c. can be associated with PA (I think this is the right answer if PA is
placenta abruption)
d. progesterone is recommended in unexplained cases

Explanation :
A number of maternal infections can lead to a single pregnancy loss,
including listeriosis, toxoplasmosis, and certain viral infections
(rubella, herpes simplex, measles, cytomegalovirus, coxsackie virus).
However, there are no confirmed studies to suggest that specific
infections will lead to recurrent pregnancy loss in humans. Malaria, syphilis
and brucellosis can also cause recurrent miscarriage.[ (from wiki)

Also, PA is not a risk factor for recurrent miscarriages. Most common femal
risk factors for recurrent miscarriage are uterine anomalies while for fetal
risk factors are chromosomal anomalies.

11. All of the following can decrease effect of COCP except:


a. Phenytoin
b. Amoxicillin
c. Gastroenteritis
d. Rifampicin
e. Valproic acid 1

Explanation : Some drugs reduce the effect of the pill and can
cause breakthrough bleeding, or increased chance of pregnancy. These
include drugs such
as rifampicin, barbiturates,phenytoin and carbamazepine. In addition
cautions are given about broad spectrum antibiotics, such
as ampicillin and doxycycline, which may cause problems "by impairing
thebacterial flora responsible for recycling ethinylestradiol from the large
bowel"
(https://en.wikipedia.org/wiki/Combined_oral_contraceptive_pill#Drug_in
teractions)

There are many commonly used medications which can affect the
efficacy of the pill:

 Antibacterials - enzyme inducers only - eg, rifampicin, rifabutin.


 Antidepressants - St John's wort (which can be bought over-the-
counter).
 Anticonvulsants - carbamazepine, oxcarbazepine, eslicarbazepine,
phenobarbital, phenytoin, primidone and topiramate due to their
enzyme-inducing activity. Also lamotrigine which has a specific safety
warning - there is an increased risk of seizure whilst taking the COCP
with lamotrigine, and a risk of toxicity during the pill-free week, when
levels rise. Therefore risk may outweigh benefit (UKMEC Category 3).
 Antiretrovirals - in particular, ritonavir-boosted protease inhibitors.
 Ulipristal acetate (the "morning after pill" ellaOne®).

(http://patient.info/doctor/combined-oral-contraceptive-pill-first-
prescription)

12. GA: 37 wks, asymptomatic placenta previa anterior, 1 cm above


cervix edge, the patient is at risk off all except:
a. PPH
b. NICU admission due to RDS (not sure if this is the correct answer coz PP
can cause both anaemia and rds fetus)
c. Severe neonatal anemia
d. C/S

Explanation :
>>Maternal complications ;
[11] including rebleeding (Planning delivery and control of

hemorrhage is critical in cases of placenta previa as well as placenta


accreta, increta, and percreta.)

[1

-3%); in the US, the maternal mortality rate is 0.03%, the


great majority of which is related to uterine bleeding and the
complication of disseminated intravascular coagulopathy

>>Fetal complications :

Congenital malformations
risk for infant neurodevelopmental delay and sudden infant
death syndrome (SIDS)

Sources : http://emedicine.medscape.com/article/262063-overview#a6

13. Clue cells are seen in :


a. bacterial vaginosis

Explanation :
a) no pus cells -> no infection -> leukorrhea
b) hyphae, spores and pus cells -> candidiasis
c) clue and pus cells -> bacterial vaginosis
d) flagellated parasite and pus cells -> trichomonas vaginitis

Sources : Clinical Skills Lab tafree3‘, under topic DnC by Dr Fayez Jallad

14. Case: ovarian tumor bilateral 5*5 cm , CA 125 = 500. Modality of Rx:
a. Chemotherapy then surgery (interval debulking surgery)
b. Total hystrectomy with bilateral salpingo oopherectomy with
omentectomy. (primary surgery)

Explanation : In primary surgery, we perform midline incision and then


hysterectomy and bilateral salpingeophorectomy, omentectomy, and we
do removal of any gross tumour. Sometimes, it may lead to colonic
resection and 2-3 weeks after that, we refer the patients for 6 cycles of
chemotheraphy.

Sources : Ovarian Tumours lecnote, pg 9

15. About gestational DM .. all are true except:


a. OGTT is not needed after puerperium
b. macrosomia incidence is 10%
c. a fasting glucose of 7.2 at 24 weeks is diagnostic for DM (I think this is
the answer. Supposedly any value>7.8 is suggestive of GDM
d. postprandial glucose level > 8 necessitate start insulin

Explanation :
We have primary and secondary methods of screening.
1. Primary screening
-Done during the first visit at 24-28th week
-We give 50g load of glucose and any value>7.8mmol suggest GDM
-To confirm, we do OGTT (give modified 75g load of glucose) and if the
value<7.8mmol the patient is OK

2. Secondary screening
-We do 2 hours postprandial and plus urine specimen

16. All of the following are relatively contraindicated in pregnancy


except:
a. Tetracycline
b. carbimazole (this is definitely the answer!)
c. Cox.2 inhibitor
d. carbamazepine

Explanation :
>>Tetracyclines - Get deposited in fetal bones and retard their growth,
also affect teeth causing them to be discolored and deformed
>>Carbamazepine – Multiple birth defects
>> Carbimazole - should be used judiciously in pregnancy as it crosses the
placenta. It has (rarely) been associated with congenital defects,
including aplasia cutis of the neonate but is not contra-indicated.
However, it more predictably may cause fetalhypothyroidism so (in
minimal doses) it can be used in order to control
maternal hyperthyroidism. There are reported cases of goiter and choanal
atresia in fetus.[2] Furthermore, breast feeding is possible but only if lowest
effective dose is used and neonatal development is closely monitored.

Sources : Wikipedia and


http://www.medindia.net/patients/patientinfo/drugs-pregnancy.htm
17. All are symptoms of fibroid except:
a. mennoraghia
b. IMB
c. abdominal distension
d. pressure symptoms
e. infertility

Explanation : Most common sx of uterine fibroid

— seven days or more of menstrual


bleeding

Sources : http://www.mayoclinic.org/diseases-conditions/uterine-
fibroids/basics/symptoms/con-20037901

18. one of the following is suggestive of stress incontinence:


a. leakage of urine on sneezing

Explanation : Patient have involuntary leakage of urine with sneezing,


couhhig, laughing, or anytime an increase in intrabadominal pressure
exceeds urethral sphincter closure mechanisms

Source : Urinary Incotinence slide by Dr Rawan Obeidat

19. all are worrying sign in late pregnancy except:


a. white vaginal discharge
b. watery vaginal discharge
c. generalized itching
Explanation :

White vaginal discharge is a sign of leucorrhea which is very common


during pregnancy. This is partly because of increased estrogen production
and greater blood flow to the vaginal area. This discharge is made up of
secretions from the cervix and vagina, old cells from the walls of the
vagina, and normal bacterial flora from the vagina.

Source : http://www.babycenter.com/0_vaginal-discharge-during-
pregnancy_270.bc

20. PCB in all of the following exept:


a. cervicitis
b. cervical polyp
c. cervical carinoma in situ (this one is true!)

Explanation :

Cervical ectropion - especially in those women taking the combined


oral contraceptive pill (COCP).

Cervical or endometrial polyps.

Vaginal cancer.

Cervical cancer - usually apparent on speculum examination.

Sources : http://patient.info/doctor/intermenstrual-and-postcoital-
bleeding

21. ASCUS:
a. 10% progress to invasive disease

Explanation : 6-10% will progress to high grade lesion and 90% to low
grade. For low grade>>repeat the smear after 6 months and if
persisted>>do colposcopy and bx
For high grade>>do HPV and if it is positive do the colposcpoe. If it is
normal repeat the smear in 6 months over 2 years.

22. all are surgery for prolapse except:


a. anterior colporaphy
b. colposusspension
c. colpopexy

Explanation : Surgical options ;


1. Uterine prolapse : vaginal hysterectomy, Manchester or
sacrohysteropexy)
2. Vaginal prolapse : anterior colporrhaphy, posterior
colpoperineorrhaphy, resection of enterocele sac or abdominal
sacroplexy in vault prolapse)
3. Lefort‘s operation

Source : OSCE oriented scriptbook ny Lajnah Tibb page 9

23. all can be detected by US in third trimester except:


a. turner
b. fetal demise
c. fetal anemia (I think this one is correct. It can be detected by Doppler
US)

Explanation : In case of fetal anemia, it is usually associated with fetal


hyrops or can be seen with fetal hepatomegaly/splenomegaly. In hyrdrop
fetalis, there will be fluid accumulation and swelling of the fetal tissues.
Also, US can detect signs of fetal heart failure and increased blood flow at
Mid Cerebral Artery.

24. about ANC ... one is true :


a. booking visit is preferred in 11.14 weeks
b. detailed anomaly scan is done at 22.24 weeks
c. at 36 weeks presentation should be confirmed

Explanation : From a source I found it is stated that detailed anomaly scan


should be done in between week 18-20.
The anomaly scan or ultrasound level two scan is the most common scan
of the second trimester. This scan can:

 show how your baby is growing


 make sure your baby's internal organs are developing well
 detect certain health problems or malformations in your baby
 estimate the amount of amniotic fluid
 check the position of the placenta

Sources : http://www.babycenter.in/a557390/second-trimester-
scans#ixzz3n2FR0tvG

25. heparin .. one is false :


a.monitored by aPTT
b.contraindicated in lactation

Explanation :
Heparin use in lactation. Not excreted in the breast milk; compatible.

Source : http://reference.medscape.com/drug/calciparine-monoparin-
heparin-342169#6

26. laparoscopy is useful in all the following except :


a. endometrial polypectomy
b. subtotal hysterectomy
c. salpengectomy

Explanation : Polyps are removed by means of curettage with the aid of


hysteroscopy

Source : https://en.wikipedia.org/wiki/Endometrial_polyp#Diagnosis
27. about cervical incompetence .. all are true except:
a. cerclage done at 11 weeks
b. can be congenital
c. diagnosed by US

Explanation : According to wiki -> The treatment consists of a


strong suture being inserted into and around the cervix early in the
pregnancy, usually between weeks 12 to 14, and then removed towards
the end of the pregnancy when the greatest risk of miscarriage has
passed.

Source : https://en.wikipedia.org/wiki/Cervical_cerclage

28. First step in shoulder dysticia :


a. Episiotomy
b. McRobert (legs up.
c. suprapubic pressure .

Explanation :
As soon as shoulder dystocia is identified, the mother's hips are flexed
back onto her abdomen. This is called McRobert's Position, and helps the
baby's shoulder slip under the mother's pubic bone by enlarging the
pelvic outlet. Usually at about the same time, a nurse or other assistant
pushes down on the baby's shoulder, behind the pubic bone, helping it
pass under. More than half of all shoulder dystocias are relieved just by
these two maneuvers, which usually take less than a minute. It is important
that the pressure be suprapubic in nature to dislodge the shoulder from
beneath the pubic bone and not fundal, or on the top of the uterus, as
this serves only to further impact the shoulder beneath the pubic bone.

Source : http://www.healthline.com/health/pregnancy/delivery-shoulder-
dystocia#McRobert'sPosition1

29. all are risk factors for cord prolapse except:


a. ECV
b. prematurity
c. oligohydramnios
Explanation :
>>RF for umbilical prolapse

disproportion.

-lying placenta.

 Amniotomy with high presenting part.


 Vaginal manipulation of the fetus with ruptured membranes.
 Insertion of intrauterine pressure catheter.
 Attempted external cephalic or internal podalic version.

30. One is false about ectopic:


a. can be excluded if BHCG test is .ve
b. almost always patients have a period of ammenorhea or irregular
bleeding
c. aria stella is characteristic
d. 50% of pts have +ve urine pregnancy test (I think this is the answer)

Explanation : In ectopic, the means of diagnosis are by doing


ultrasonography and bHCG test. For bHCG, we check the serum bHCG
and the minimum level for bHCG to detect pregnancy by transvaginal US
is 1500 mIU/mL. Ectopic rarely present before this value.

Source : Ectopic Pregnancy lecnote, pg 8


31. At 12 weeks gestation ... a gestational sac found using abdominal US..
CRL = 7mm ... and no heart beat found .. your next step :
a. repeat US after 1 week
b. terminate pregnancy (missed abortion.
c. serial B.HCG

Explanation : Missed abortion is diagnosed by performing 2 US (TV and TA)


with at least 7 days apart which will show embryo of > 7 weeks gestation
(CRL > 6 mm and gestational sac>20mm in diameter with no evidence of
heart activity

Source : Abortions lecnote by Dr Fayez

32. first sign of uterine Dehiscence is :


a. tachycaria
b. hypotension
c. fetal distress
d. abnormal abdominal shape

Explanation : Typicall, rupture is characterized by the sudden onset of


intense abdominal pain and some vaginal bleeding. Impending rupture
maybe heralded by hyperventilation, restlessness, agitation and
tachycardia. The most consistent finding is abnormal fetal heart pattern.

Source : Hacker Textbook, pg 131

33. Amniotic fluid embolus .. all true except:


a. managed by heparin and FFP
b. usually with coagulation disorder
c. very serious condition

Explanation :

34. about STD .. all true except :


a. gonorrhea can cause infertility
b. HIV patient must be deliverd vaginaly
c. primary herpes can be associated with lymphadenopathy
d. syphilis in third trimester should be treated with penicillin
Explanation :
1. Gonorrhea might cause subfertility
2. DOC for syphilis is penicillin
3. Primary herpes I s associated with inguinal adenopathy, dysuria and
acute urinary retention.

35. HRT .. all are true except:


a. increase risk of colon cancer.
b. causes endometrial hyperplasia
c. subdermal patches have less SE
d. local estrogen is more effective for atrophic vaginitis .

Explanatoin : HRT reduces the risk for having colon cancer, osteoporosis.
Stroke, vasomotor symptoms, urogenital symtomps and sexual function.

HRT increases the risk for breast cancer, VTE, and endometrial cancer.

Source : Climateric slides

36. All are matched except :


a. POP . Wt loss
b. danazol . hoarseness of voice
c. COC . headache
d. mirena – spotting

Explanation :
Side effects of POP include:
 Irregular menstrual cycles.
 Spotting or bleeding between menstrual periods.
 Sore breasts.
 Headache.
 Nausea.
 Dizziness.
 Bloating or weight gain, especially with the birth control shot.
 No menstrual periods. Although mini-pill use can stop periods, the shot is
most likely to do so.
Source : http://www.webmd.com/sex/birth-control/progestin-only-
hormonal-methods-mini-pills-shots

37. about endometrial CA.. one is true :

a. serous papillary type is the commonest


b. serous papillary has worse prognosis than endometriod

Explanation : Subtypes of endometrial adenocarcinoma -


a) endometrioid (most common, 90% of cases)
b) clear cell (most malignant)
c) papillary serous (with hereditary types)
-clear cell and papillary serous represent only 10% of endometrial cancer,
but they are responsible for more>50% of death related to endometrial
cancer.

d)adenocanthoma : endometrioid adenocarcinoma with squamous


differentioan (good prognosiss)

e) adenosquamous carcinoma : has both malignant glandular and


malignant squamous components (poor prognosis)

**clear cell and papipllary serous both have poor prognosis

38. Molar .. one is false :


a. grape like vesicles
b. malignant sequele can be avoided
c. 50% chance of persistant GTN .

Explanation : If what is meant by persistent GTN is the conversion of


benign hydatidiform mole into invasive mole, the incidence is about 10-
15%

2-3% will develop into choriocarcinoma

Source : https://en.wikipedia.org/wiki/Molar_pregnancy
39. about HPV vaccine .. one is true :
a. not effective in males
b. can be used in treatment of HPV 16 ,18 related CIN II, III
c. single dose at 12 years old

Explanation :

All preteens (age 11 or 12 years) need an annual flu vaccine and


 1 dose of Tdap vaccine,
 2 doses of meningococcal vaccine, and
 3 doses of HPV vaccine.
I‘m not sure about the dosage used here in Jordan

Source : http://www.cdc.gov/vaccines/who/teens/for-parents.html

40. About Kielland forceps true except:


a. Has cephalic curvature
b. Has pelvic curvature
c. Has an english lock
d. Used for rotation

Explanation : English lock (fixed lock) can be found in traction forceps


such as Simpson and Piper.

Sliding lock can be found in rotational forceps such as Kielland forceps

Source : ObGyn Clinical Skills Lab tafreee3‘ under topic instrumental


delivery

41. All causing 2nd amenorrhea except:


a. Meyer.rokitansky
b. Ashermans syndrome
c. Pcos
Explanation : Meyer rokitansky is the most common cause of primary
amenorrhea characterized by a failure of the Müllerian duct to develop,
resulting in a missing uterus and variable degrees of vaginal hypoplasia of
its upper portion.

Source : https://en.wikipedia.org/wiki/M%C3%BCllerian_agenesis

42. All causing 2nd amenorrhea except:


a. Transverse vaginal septum

Explanation : Transverse vaginal septum is one of the cause of primary


amenorrhea.

Source : Amenorrhea lecnote, pg 2

43. About candidal infection all are true except:


a. vaginal discharge
b. ph less than 5
c. +ve WIFF test

Explanation : +ve Whiff test characterized by appearance of fishy odour is


suggestive of bacterial vaginosis, not candidiasis. It is done by adding a
small amount of 10% potassium hydroxide.

Source : ObGYn Clinical Skills Lab tafree3‘

44. postdate complication except :


a. shoulder dystocia
b. hyperbiluribinemia
c. placenta insufficient

Explanation : Hyperbilirubinemia or fetal jaundice is not a complication of


postdate pregnancy but rather a complication of term or preterm babies.
The following I found from the wiki. ▼▼

Physiological jaundice[edit]
Most infants develop visible jaundice due to elevation of unconjugated
bilirubin concentration during their first week. This common condition is
called physiological jaundice. This pattern of hyperbilirubinemia has been
classified into two functionally distinct periods.
Phase one

1. Term infants - jaundice lasts for about 10 days with a rapid rise of
serum bilirubin up to 204 μmol/l (12 mg/dL).
2. Preterm infants - jaundice lasts for about two weeks, with a rapid rise
of serum bilirubin up to 255 μmol/l (15 mg/dL).
Phase two - bilirubin levels decline to about 34 μmol/l (2 mg/dL) for two
weeks, eventually mimicking adult values.

1. Preterm infants - phase two can last more than one month.
2. Exclusively breastfed infants - phase two can last more than one
month.

Source : https://en.wikipedia.org/wiki/Neonatal_jaundice

45. The definitive treatment for Endometriosis :


a. Hysterectomy
b. Hysterectomy & Bilateral Salpengoopherictomy
c. Myomectomy

Explanation : The most comprehensive surgeryincludes total abdominal


hysterectomy, bilateral salpingo-oophorectomy with destruction of all
peritoneal implants, and dissection of all adhesions.

Sources : Hacker Textbook, pg 302

46. All of these require US to Dx except :


a. Threatened Abortion
b. Missed Abortion
c. Inevitable Abortion

Explanation : Both inevitable and incomplete abortions they are


diagnosed clinically. US is not essential for the diagnosis, just for
confirmatory purpose.

Source : Abortions lecnote, pg 4


47. Unexplained infertility one is false :
a. diagnostic test is ovarian ultrasound .. can‘t remember the rest of the
choices :/

48. all are routinely done pre. operatively except :


a. chest X.ray (unless if you’re thinking about lung cancer or maybe mets
from invasive mole!)
b.CBC

49. all done in case of urinary incontinence except :


a. urine analysis
b.pelvic U/S
c.KFT
d.urodynamic

Explanation : Diagnostic tests that are done in urinary incontinence -


1. Stress test
2. Cotton swab ( Q-tip) test
3. Urethrocystoscopy
4. Cystometrogram
5. Urethral pressure measurements
6. Uroflowmetry
7. Voiding cystourethrogram
8. Ultrasonography
9. Urianalysis, postvoid residual volume test, urodynamic test (from the
slides Urinary Incotinence)

Source : Hacker Textbook, pg 281-283

50. all routinely done in ANC except :


a. PTT

Explanation : Routine lab tests done during ANC –


1. Hb
2. Platelets
3. Blood group and Rh
4. Antibody screening
5. Urine analysis
6. Rubella titer
7. Hepatitis screening
8. Venera; disease research laboratory test (VLDRL)
9. HIV screening
10. Blood sugar
11. Pap smear

Source : Antenatal Care lecnote by Dr Nael, pg 4-5

51. one is associated with fetal acidosis :


a.amniotic fluid index = 5
b.reversed end diastolic flow in umbilical artery Doppler
c.HR of 110 .

Explanation :

52. primi gravida fully dilated for 1 hr , u accept these findings except :

a. semi.flexed
b.ROP
c. station 0
d. caput ( I think this might be the answer )

Explanation : If caput succedaneum is what meant by ‗caput‘ in the


answer option, it happens shortly after delivery and I don‘t think it can
happen as soon as during the first phase of labour.

Caput succedaneum refers to swelling of an infant‘s scalp, which appears


as a lump or bump on the head shortly after delivery. This condition is
harmless and is caused by pressure put on the infant‘s head during
delivery. It does not indicate damage to the brain or the bones of the
cranium.

Source : http://www.healthline.com/health/caput-
succedaneum#Overview1

By Ibnu Afiq
53. about Rh .isoimmunisation one is false :
a. anti D given within 72 hrs ,for Rh positive woman .

Explanation: anti D is given within 72 hrs ,for Rh negative woman

54. Endometriosis can be a cause of all the following except :


a. infertility
b.amenorrhea
c.dymenorrhea

Explanation: endometriosis symptoms are dysmenorrhea, dyschezia and


dyspareunia. Menorrhagia is uncommon.
Source: Hacker pg 300

55. one is true about breech presentation:


a. it carries higher mortality than cephalic whatever the mode of delivery .
b. post.maturity is a known risk factor .

Explanation: even with optimal management, perinatal mortality of


breech fetuses ishigher than nonbreech fetuses. Risk factors that
contribute to perinatal mortality is:
-lethal cong.anomalies, prematurity, birth trauma and asphyxia
Source: Hacker pg 169 & 170

56. Elevated A feto.prtn in all of the following conditions except:


a. down syndrome

Explanation:
1) Low A feto protein is found in Down syndrome and Trisomy 18. Diabetic
pt also.
2) Elevated A feto protein in placental abruption, neural tube
defects including spina bifida and anencephaly, and abdominal wall
defects, endodermal sinus tumor (EST) or another germ cell
tumor containing EST. These tumors can occur in the pregnant woman
(often as an ovarian tumor) or in the fetus.
Source: https://en.wikipedia.org/wiki/Elevated_alpha-
fetoprotein#Maternal_testing_for_fetal_screening
57. True about stored blood:
a. Deficient in 2,3 DPG

Explanation:
The more 2,3DPG in the cell, the more oxygen is delivered to body tissues.
Conversely, the less 2,3-DPG in the cell, theless oxygen is delivered
2,3DPG levels are important in large blood transfusions, because stored blood
quickly loses 2,3-
DPG and its ability to deliver oxygen.After transfusion, the red cells rebuild the
2,3-
DPG, but it takes about 24 hours to regain a normal level of 2,3DPG and hemo
globinfunction.
Source: http://medical-dictionary.thefreedictionary.com/2,3-
diphosphoglycerate+test

58. Not an absolute indication for CS delivery:


a. 2 previous CS.
b. cord prolapse.
c. cord presentation .
d. Second twin mal.presentation .

Explanation: elective cs in twins presentation, but may allow vaginal


delivery
Source: Hacker pg 165

59. About ovarian cancer,one of the folowing is wrong:


a. Dysgerminoma is more common in older age/menopouse woman

Explanation: Germ cell tumor- dysgerminoma in young women, most


common, and highly malignant
Source: Table ovarian tumor

60. About primary amenorrehea, what is wrong:


a. Brain MRI must be done
b. Pelvic u/s is essential
c. karyotyping is a must if FSH and LH are low .
d. TSH can be done
Explanation: Low FSH and LH indicates hypogonadotrophic
hypogonadism
Source: http://www.aafp.org/afp/2006/0415/p1374.html

61. Not detected in first trimester by U/S :


a. microcaphaly

Explanation: In many cases, microcephaly may not be evident by


ultrasound until the third trimester and, therefore, may not be seen on
ultrasounds performed earlier in pregnancy. The diagnosis of
microcephaly may be made at birth or later in infancy.
Source:
https://www.urmc.rochester.edu/encyclopedia/content.aspx?ContentTy
peID=90&ContentID=P02610

62. Not true in fibroid :


a. 2 % will transform into sarcoma

Explanation: Typically, fibroids neither lead to cancer nor do they increase


a woman's chances of developing cancer of uterus. However, studies
indicate that one out of every 1,000 women admitted to the hospital for
fibroid surgery have a leiomyosarcoma, an extremely rare form of
malignant tumor of the uterine muscle.
Source: hacker pg 241

63. A 22 years old primigraveda GA 30 weeks her BP 150/90 one of the


following is true :
a.serum uric acid is elevated
b. you have to repeat BP after 6 hour rest
c. this is pregnancy induced hypertension

Explanation: Gestational hypertension is usually defined as having a blood


pressure higher than 140/90 measured on two separate occasions, more
than 6 hours apart, without the presence of protein in the urine and
diagnosed after 20 weeks of gestation.
Source: https://en.wikipedia.org/wiki/Gestational_hypertension

64. In DIC all increase except :


a. fibrinogen .
b. Aptt .
c. PT .

Explanation:
reduced level= fibrinogen & platelet count,
increased level = aPTT, PT, FDP
Source: Table DIC

65. A 40 years old female has a heavy menses what is the best
contraceptive method :
a. mirena IUCD
b.OCP
c.progesteron minipills.

Explanation: 20-40 years old, use mirena IUCD


Source: table dysfunctional uterine bleeding

66. All of the following are vaginal procedures except :


a. tubal ligation .

Explanation: fallopian tubes are not part of vagina


Source: https://en.wikipedia.org/wiki/Tubal_ligation
67. Something about amenorrhea all of the following is true except :
a. Rokitansky syndrome karyotype is 46 XY .

Explanation: 46 XY is Swyer syndrome(pure gonadal agenesis/ dysgenesis)


-with failure of testicular development
-no androgen or MIF, so Wolfian structures regress and Mulrian structures
persist,
-with normal female phenotype,
-normal or excessive height (delayed epiphyseal closure due to low
androgens or estrogens)
-Menses will occur with estrogens
Source: Table amenorrhea

68. about endometrial CA all are true except :


a. 5 year survival rate in early stages is 30 % .

Explanation: Endometrial cancer usually present in early stage, 5 yr survival


rate is 70%
Source: Table endometrial adenocarcinoma

69. about PAP smear all are true except :


a. HSIL is usually caused by HPV 6 , 11 .

Explanation: HPV of high risk (16, 18) is found in 70% of all cervical cancers
High grade squamous intraepithelial lesion (HSIL) is part of epithelial cell
abnormality with 20% progress to invasive cancer if left untreated for
several years.
Source: slide early pregnancy bleeding 6th year

70. One of the following is not of the acute management of


preeclampsia:
a. hydralazine
b. labetolol
c. nifipedine
d. methyldopa .
Explanation:Safest efficacious drugs in acute control of severe
hypertension complicating pre-eclampsia are labetolol, hydralazine, oral
nifedipine (should be cautious to avoid hypotension) or sodium
nitroprusside (can cause fetal cyanide toxicity with prolonged
administration)
Source: Hacker pg 179

71. Methotrexate is not used if:


a. sac less than 40 mm
b. fetal heart activity is detected

Explanation: Methotrexate is a folic acid antagonist that inhibits DNA


synthesis and cell reproduction for management of early, unruptured
ectopic pregnancy in most cases.
-Indication of use is unruptured mass of less than 3.5 cm and hcG doesn‘t
exceed (6000-15,000 IU/mL)
-contraindications of MTX use are gestational sac more than 3.5 cm and
embryonic cardiac motion
Source: table hacker pg 297

72. about a baby with meconium stained liquor at the onset of labor,
what to do:
a. if the fetal heart rate was active only observe
b. deliver immediately by CS
c. it is an indication for fetal heart monitoring

Explanation: Intrapartum management for meconium stained liquor:

 If significant meconium staining is noted in labour, there should be


continuous electronic fetal monitoring.
 This is defined as dark green or black amniotic fluid that is thick or
tenacious, or any amniotic fluid that contains lumps of meconium.
 Transfer mother to obstetric-led care, if it is safe to do so and delivery is
not imminent.
 If there are signs of fetal distress, a fetal blood sample should be
obtained. If pH is <7.21, there should be emergency delivery.
 Ensure that the advanced resuscitation unit and appropriately trained
staff are available.
 There should be no suction prior to delivery

Source: http://patient.info/doctor/meconium-stained-liquor

73. All of the followings can be used in the treatment of stress


incontinence except:

a. Pelvic exercise
b. Colposuspension
c. TVT (Tension-Free Vaginal Tape)
http://www.webmd.com/urinary-incontinence-oab/tension-free-vaginal-
tape-for-stress-incontinence-in-women
d. Toilet training .

Explanation: bladder or toilet training is for urge incontinence


Source: slide urinary incontinence 6th yr

TVT
6. 5th Year Final Exam 2012 :
1. Which following is oxytocin antagonist:
a. Atosiban
b. Ritodrine

2. All are risk factors for placental previa except:


a. Previous previa
b. Polyhydramnios
c. Previous C/Sec
d. Multiparity

3. All are risk factors for endometriosis except:


a. Sisters have endometriosis too
b. High parity
c. Intfertility

4. Contraindication of ritodrine:
a. Hyperthyroidism
b. Uterine anomalies

5. Which is true about Turner’s Syndrome:


a. 45 XY is the characteristic
b. Associated coartication of aorta
c. Secondary amenorrhea

6. Which is wrong about Granulosa Cells Ovarian Tumor:


a. Bilateral in 50%
b. Malignant in 25%
c. Produce estrogens
d. Associated with endometrial cancer
e. Have Call.Exner bodies

82
7. Commonest complication to fetal on diabetic mother:
a. Spina bifida
b. Anal atresia
c.VSD
d. Renal agenesis
e. Sacral agenesis

8. Spinbarkeit is:
a. Threading of mucus
b. Thickening of mucus
c. Thinning of mucus
d. Crystallization of mucus

9. What is the treatment for Postpartum Blues:


a. Reassurance
b. SSRI
c. Psychological consultation
d. ECT

10. A women with G2 P0 +1 is considered as:


a. Have a stillbirth
b. Have 2 miscarriages
c. Pregnant right now
d. Previous preterm birth

11. Which has the strongest association with recurrent preterm birth:
a. Smoking
b. Previous preterm
c. PET

12. All following is contraindicated in pregnancy except:


a. Ciprofloxacin
b. Erythromycin
c. Doxycycline
d. Vitamin A
83
e. Methrotrexate

13. Most common symptom of Ectopic pregnancy :


a. Abdominal Pain
b. Vaginal bleeding
c. Amenorrhea

14. Which is not a sexual transmitted disease:


a. Toxoplasmosis
b. Chancroid
c. Chlamydia

15. All are uses of clomiphene citrate except:


a. Premature ovarian failure
b. Polycystic ovarian syndrome
c. IVF

16. All are investigations done for hydatidiform mole except:


a. Chest Xray
b. Coagulation profile
c. Pelvic/abdominal CT scan
d. bHCG

17. All are absolute contraindication of COCP except:


a. Liver disease
b. Breast cancer
c. Previous DVT
d. Taking pills in the previous pregnancy

18. All are true about minipills except:


a. Cause functional ovarian cyst
b. Have same effects as COCP in ovulation inhibition

19. The most likely cause of acute abdominal pain for patient who had just have IUCD insertion

84
is:
a. Copper allergy
b. Uterine contraction
c. Uterine rupture

20. Which is wrong about hormone replacement therapy:


a. Give for short duration
b. Only orally
c. Lowest effective dose is recommended
d. Increase breast cancer risk

21. Which investigation done for an infertile couple:


a. Ovarian biopsy
b. Semen analysis
c. Laparoscopy
d. Antisperm antibodies
22. A male patient came to the clinic with asthenozoospermia and teratozoospermia while her
wife is normal. What kind of treatment you’ll choose for them?
a. GIFT
b. ICSI
c. IVF

23. Which is wrong about human milk:


a. Lesser protein than cow’s milk
b. Lesser calories than cow’s milk
c. More lactose than cow’s milk
d. Rich in Vitamin B

24. All are absolute contraindications for induction of Labor except:


a. PET
b. transverse lie
c. Abnormal CTG
d. Previous 2 C/sec
e. Previous pelvic floor repair

85
5th Year Final Exam 2012 :
1. A lady presented with uterine contractions (3 per 10 minutes). The cervix is 4 cm dilated and 1 cm
long, what’s the stage of labor?
A. Active first stage
B.+ Latent phase (the cervix still not fully effaced)
C. Active second stage
Latent phase
D. Third stage
• Time from onset of labor until 3cm dilation of cervix
E. She is not in labor
• By the end of this phase the cervix will be fully effaced.

Active first stage

• From 3cm dilation until 10 cm cervical dilation

2. A lady had her last menstrual period on 27/12/2012. What is her expected date of delivery?
A. 03/10/2013
B. 20/09/2013
C. Cannot be calculated
Note: This Q seems easy and most of us had chosen "A", but the problem is that it didn't mentioned

3. A baby with breech presentation. Hips are flexed and knees are extended, what’s the type of
breech?
A. Complete
B. Incomplete
C. +Frank
4. In vertex presentation, what’s the bony denominator of the presenting part?
A. +Occiput (vertex)
B. Sinciput (brow)
C. Chin (face)

5. The relation of the presenting part of the fetus to maternal ischial spines is called….
A. Lie
B. Position Fetal Station
C. +Station  the relation of the presenting part of the fetus to the ischial
spines of the mother's pelvis
D. Presentation

6. All of the following are correctly matched, except:


A. Vertex --- Suboccipito-bregmatic
B. +Brow --- Occipito-frontal (supraoccipitomental/sinciput)
C. Face --- Submento-bregmatic
7. Regarding normal labor, all of the following are true, except:
A. Second stage starts with full cervical dilation and ends with delivery of the baby
B. +True uterine contractions can be suppressed by epidural anesthesia
C. Third stage is delivery of placenta and the membranes
D. Second stage takes longer in primigravida compared to multiparous
E. False contractions don’t cause cervical dilation

8. Regarding the mechanism of normal labor, all of the following are true, except:
A. Fetus enters the pelvis in the transverse diameter
B. +Internal rotation is for fetal shoulders (for fetal head not the shoulder)
C. The anterior shoulder is delivered before the posterior
D. The head is delivered in extension

9. All of the following are classes of tocolytics, except:


A. Calcium channel blockers
B. +Anticholinergics
C. Oxytocin antagonists
D. Prostaglandin inhibitors

10. All of the following are correctly matched, except:


A. Face --- presentation
B. Flexed --- Attitude
C. Transverse ---- Lie
D. + Vertex --- Position (presentation)

11. Which of the following is considered an abnormal finding in CTG?


A. Early deceleration in second stage (physiological)
B. +Late deceleration (placental insufficiency)
C. Baseline heart rate of 120 (normal :110-150)
D. Absence of accelerations (absence of accelerations with an otherwise normal CTG is of uncertain
significance)

13. Which of the following is NOT part of the routine partogram?


A. Vital signs
B. +Pelvimetry
C. Cervical dilation
D. Strength of contractions

14. Regarding fetal skull in relation to maternal pelvis, all are true, except:
A. Platypeloid pelvis is likely to cause obstructed labor
B. Android is the typical male pelvis
C. +Gynecoid pelvis forces the baby into an occipito-posterior position (occipito-anterior)

15. All of the following are true regarding spinal anesthesia, except:
A. + Can cause hypertension (hypotension)
B. Meningitis is a possible complication
C. Contraindicated in case of coagulopathy
Complication of spinal anesthesia
 Hypotension, bradycardia if block
reachs T2-T4.
D. Contraindicated if skin is infected at the site of needle
 HIGH SPINAL BLOCKADE.
 Post spinal headache (post dural)
 Failure of technique .
 Epidural or subarachnoid hematoma
 Spinal cord trauma or infection
 Urinary retention
16. Regarding the use of pethidine in labor, all are true,
 Rarely, convulsions and blindness
except:
A. Can cause neonatal respiratory depression
B. Can cause neonatal bradycardia
C. Easily administered
D. Naloxone is the antidote
E. + Inhibits uterine contractions

Meperidine (pethidine)
 synthetic phenylpiperidine derivative which is commonly administered
intramuscularly (IM) in a dose of 1mg /kg.
 Analgesia is maximal 45 minutes post injection
 delays gastric emptying,
 increase gastric volumes in labor.
 causes sedation
 dose-dependant respiratory depression
 its active metabolite (nor-meperidine) has convulsant properties.
 crosses the placenta and its effects on the fetus are dependant on dose
and timing of administration
17. A baby in breech presentation, where would you listen to the fetal heart?
A. + Above the umbilicus on the side of the back
B. Above the umbilicus opposite to the back
C. Below the umbilicus on the side of the back
D. Below the umbilicus opposite to the back
E. At the umbulicus

18. All of the following are risk factors for vaginal candidiasis, except:
A. + Thyrotoxicosis
Risk factors of Vaginal Candidiasis:
B. Oral contraceptive use
1. Altered immune status (eg. Using systemic steroids)
C. Pregnancy 2. Increased glucose levels (undiagnosed or uncontrolled DM)
3. Decreased lactobacilli concentrations (using antibiotics)
4. Contraceptive practices (influence pH)
5. Tight clothing, panty hose, and bathing suits (yeast thrives in a
dark, warm, moist environment)
D. Diabetes

19. Regarding Trichomonas vaginalis infection, only one of the following is correct
A. Commonly causes salpingitis
B. + It’s a parasitic STD that causes a pruritic discharge.
C. Clue cells are characteristic (bacterial vaginosis)

20. Regarding vulvovaginal candiasis, only one of the following is correct:


A. + can be microscopically diagnosed by using KOH
B. Muco-purulent vaginal discharge
PID:

 Acute/ Chronic clinical syndrome


21. By definition, PID is:
 Spectrum disease involve cx, uterus, tubes,
A. Infection of the vagina and ovaries
B. Infection of the cervix  • Ascending spread of infection from the
C. + Endometritis and salpingo-oophoritis vagina and endocervix to the endometrium,
fallopian tubes, ovaries, &/ or adjoining
structures
 Upper genital tract infection, salpingitis
endometritis, parametritis, tubo-ovarian
abscess & pelvic peritonitis

22. The main postpartum complication of placental abruption is:


A. + Post partum hemorrhage?
B. Sheehan syndrome
C. Renal tubular necrosis
D. Renal cortical necrosis
E. Endomyometritis
Maternal Foetal
 Disseminated intravascular coagulopathy  Premature labour
 Hypovolemic shock  IUGR in chronic abruption
 Amnionic fluid embolism  Hypoxic ischemic encepalopathy and
 Renal tubular necrosis and acute renal cerebral paulsy
failure  Foetal death
 Post partum haemorrhage
 Sensitization of Rh(-) mother for foetal
blood
 Sheehan’s syndrome
 Maternal death

23. Placenta previa is associated with increased risk of all of the following except:
A. IUGR
B. Primary PPH
C. + Prolonged pregnancy (Premature labour)

24. Regarding postpartum hemorrhage, all of the following are true, except:
A. Atony is the most common cause of primary PPH
B. Infection of RPOC is the most common cause of secondary PPH
C. + Secondary PPH occurs between 24 hours and 6 months postpartum

Secondary PPH

 Loss of MORE than or EQUAL to 500mL blood from the genital tract between
24 hours and 12 weeks post delivery
25. Which is true about stages of labor:

a. Descent, Engagement & Flexion

Cardinal movements of labor, the typical sequence of


positions assumed by the fetus as it descends through the
pelvis during labor and delivery. The positions are usually
designated as engagement, flexion, descent, internal rotation,
extension, external rotation or restitution, and expulsion.

(http://medical-
dictionary.thefreedictionary.com/cardinal+movements+of+la
bor)

26. All are initial investigations in ANC except: (the keyword is initial)

+a. Blood glucose (ANSWER)

At 26–28 weeks gestation, a second round of blood tests,


commonly referred to as the “second antenatal” tests, is
advised for pregnant women. In most cases the LMC will
arrange these tests. The second antenatal screen includes:

-50 g glucose tolerance test (the ”polycose” test)

-CBC

-Blood group antibodies

b. Hematocrit

+c. Liver transaminases (ANSWER ALSO)

During the last three months of pregnancy and in the


immediate post partum period a large number of liver diseases
can cause elevated transaminase levels, depending upon the
clinical presentation. So, it’s not important in the initial
investigation.
(http://www.ncbi.nlm.nih.gov/pubmed/8022721)

d. Antibody screening

Tests included in the first antenatal screen include:

-Complete blood count

-Blood group and antibody screen

-Rubella antibody status

-Syphilis serology

-Hepatitis B serology

-HIV

(http://www.bpac.org.nz/BT/2011/July/pregnancy.aspx)

27. About thyroid disease in pregnancy, which is false:

a. Hashimoto‘s thyroiditis is the commonest cause of


hypothyroidism

+b. High total thyroxin

Serum total T4 and total T3 steadily increase during pregnancy


to approximately 1.5 times the non-pregnant level by mid
second trimester. So, It’s normal to have high total thyroxin
level. But, if it’s above the "normal range for pregnancy"
(approximately 9-18 mg/dl, 120-240 nmol/L), hyperthyroidism.

(http://www.obfocus.com/high-risk/Thyroidtests.htm)

c. Low thyroid binding protein

TBG is increased during pregnancy because the high estrogen


level induce increased hepatic synthesis. (Hacker and Moore’s
page 62)
d. Beta blockers are used for initial treatment in hyperthyroidism

Beta blockers ameliorate(improve) the symptoms of


hyperthyroidism that are caused by increased beta-
adrenergic tone. These include palpitations, tachycardia,
tremulousness, anxiety, and heat intolerance. Thus, a beta-
blocker should be started (assuming there are no
contraindications to its use) in most patients as soon as the
diagnosis of hyperthyroidism is made, even before determining
the cause of the hyperthyroidism. They should be continued
until resolution of hyperthyroidism.

28. All are causes of meconium in utero except:

a. Bowel peristalsis

b. Hypoxia

+c. Intrauterine cystic fibrosis (meconium ileus is the first sign


intrauterine CF. It‘s not related to meconium aspiration)

MAS is often related to fetal stress. This can be caused by


problems in the womb, such as infections, or by difficulties
during the birth. A distressed baby may have hypoxia
(decreased oxygen), which can make the baby's intestinal
activity increase and cause relaxation of the anal sphincter
(the muscular valve that controls the passage of feces out of
the anus). This relaxation then moves meconium into the
amniotic fluid that envelops the baby.
(http://kidshealth.org/parent/medical/lungs/meconium.html#)
29. Which following patient is given Anti D: ???

a. Rh. -ve female with Rh+ve male

b. Rh. -ve female who had amniocentesis

+c. Rh. -ve female with threatened miscarriage at 6 weeks

30. Which of following can cause preterm birth?

+a. Gardnarella vaginalis

In women in spontaneous preterm labor with intact


membranes, the most commonly identified bacteria are
Ureaplasma urealyticum, Mycoplasma hominis, Gardnerella
vaginalis, peptostreptococci, and bacteroides species

(http://www.nejm.org/doi/full/10.1056/NEJM200005183422007)

b. Lactobacillus

c. Trichomonas

31. All are true about lower uterine segment except:

a. Develop 28-32 weeks of pregnancy

b. Poor contractile

c. Less vascular than upper segment


+d. 1cm before pregnancy

e. Stretch during labor

32. Which is an inappropriate match:

+a. Station 1 cm above ischial spine

b. Position occipito posterior

c. Engagement 3/5th palpable

d. Lie Oblique

e. Presentation Shoulder

33. Which has the largest diameter:

+a. Brow

b. Occipitoposterior

c. Occipitotransverse

d. Occipitoanterior

34. Causes of heart failure include all except: (all are true)

a. corticosteroids
b. Underweight

c. Tocolytics

35. About follicular phase of menstrual cycle which one is true:

a. Decrease aromatization (increases aromatization of


androgens to estrogens)

b. decrease estrogen

c. progesterone predominance

(follicles >10mm are estrogen predominant and smaller ones


are androgen predominant)

d. Fixed 8 days period

+e. endometrial proliferation

36. About testicular feminization, all correct except:

+a. male phenotype

b. due to androgen insensitivity

37. about breast milk the options were:

a. has less proteins than cow's milk (true)

b. has less calories than cow's milk (false)

c. has more lactose than cow's milk (no idea)


38. about lower uterine segment, the wrong answer among these is:

a. develops between 28-32 weeks

b. has less blood supply

c. less muscular

39. normal physiology during pregnancy:

a. splitting

b. loud S1 and S2

+c. 95% have diastolic murmurs that disappear after delivery

d. S2 has no constant pattern or something like that

40. Regarding fontanels:

a. ant closed in the first 6 month (closed at 2nd year)

+b. vertex most common presentation

c. vertex btw ant and posterior fontanel and parietal bone

d. molding is normal finding (molding is abnormal)

41. about preeclampsia:

a. seizure can be presented after delivery for the first time


b. intravascular pressure increase

+c. HELLP syndrome can be presented without hypertension

HELLP can occur without clinical manifestations of DIC and is a


sign of severe preeclampsia even if blood pressures are normal
or only minimally elevated (Hacker page 177)

d. chronic hypertensive patient may not need medication in mid


trimester

42. lady that 10 weeks GA... discovered to have mass 6*6. What is your
action?

A reassurance

+b. reevaluate at 14-15 weeks

c. immediately surgery

43. most common presentation for endometrial CA:

+a. post-menopausal bleeding

44. Regarding Epidural Anesthesia in labor, all are wrong except :

a. less effective in pain relief than other methods. (not the


answer)

45. All the following cause stress incontinence except:


a. Pregnancy and childbirth

b. COPD (leads to chronic coughing) and obesity

c. smoking

+d. connective tissue diseases

46. DUB treatment , all except : (all are true)

a. COCP

b. progestin only pill

c. Mirena

d. GnRH analogue (rarely used)

47. what is not a hematological change in pregnancy:

a. decrease blood count.

+b. decrease ESR .


‫اخزٍف اًٌٍٍ َاٌىٍبس‪ ،‬رعبلت اٌىُس َاٌظٍم ‪ ،‬رُاًٌ اإلششاق َاٌعزمخ ‪،‬‬
‫رٌه‬
‫اٌفجش َرٌه اٌغشَة‪ ،‬وم اٌزضد ٌٍب مشبعش‪َ ،‬وم َجفذ ٌٍب لٍُة ‪َ .‬وم‬
‫وبوذ‬
‫أعجُثخ مه األعبجٍت ‪ ،‬ثم فمذ اإلوسبن ٌٍَزٍب َسَعزٍب مع اٌزىشاس‪ ،‬إيً‬
‫اٌمٍت‬
‫اٌمؤمه اٌزي رزجذد فً حسً ٌزي اٌمشبٌذ ‪ٌَ ،‬ظً أثذاً ٌزوش ٌذ هللا فٍٍب ‪،‬‬
‫فٍزٍمبٌب فً وً مشح ثشَعخ اٌخٍك اٌجذٌذ‪...‬‬
6th Year Final Exam 2011 :
1. True about ectopic: a. Better prognosis if implanted in the distal portion
of the fallopian tube

Because ampulla is larger and more difficult for the ectopic to


rupture. Most common site is the right ampulla

2.True about GDM: a. Maternal Insulin does not cross the placenta.

3. Not associated with Diabetes : a. Associated with folic acid deficiency

Kita bagi folic acid preconception, so ada kaitan dgn folic , kalau
hba1c >6 , 25% bleh jadi conginetal malformation.

4. When applying forceps:


a. It is important to know position of head (or something like that..

ENGAGEMENT IS A MUST IN INSTRUMENTAL DELIVERY EXCEPT


VACUUM IN SECOND TWIN

5. 21Y/O Amenorrhea, negative PCT, what is the next best hormone to


diagnose her: a. FSH.
Amenda PCT???

6. Correct parameter:

a. Biparietal=9.5, other form had a different answer.

7. Face presentation is: a. Submentobregmatic.

8. About maternal Bony pelvis:

a. it is long but if I remember correctly it was something about ischial


tuberosity.
Tinggal past year ni buat soalan kompem betul soalan , kompem betul
jawapan ja laa.ten teacher assessment pun ada, board review q&a pun
ada. Jangan buang masa.

9. 9 weeks amenorrhea, by Abdominal US shows 20 mm gestational sac:


what's your management : a. Serial B HCG...important to realize here that
US is abdominal not vaginal.

Any mass, bleeding pain in reproductive age RULE OUT PREGNANCY

10. Treatment of Ovarian CA: a. 6 cycles of carbo+pacitaxel.

Management of Ovarian Malignancies: Surgery

• Primary surgery: TAH + BSO + 6 cycles of chemotherapy.

11. Ovarian Cancer secreting hormone: a. Choriocarcinoma, other form I


think it was dysgerminoma

Since choriocarcinomas include syncytiotrophoblasts (beta-HCG


producing cells), they cause elevated blood levels of beta-human
chorionic gonadotropin. (wiki)

12. Associated with decrease risk of PET: a. Smoking.

+placenta previa + consanguinity (more paternal antigen exposure)

13. Regarding HF mole, which is wrong?

+a. OCP is safe and effective during post treatment management.

b. prominent theca lutein ovarian cyst > 6 cm should be surgically


managed.

JANGAN PAKAI COC TIME TGH FOLLOW UP, PAKAI KONDOM

14. All of the following are true about lactation except:

a. can be continued until 4 years of age...

b. Breast milk is poor in protein compared to cows'


+c. Placental lactogen hormone maintains milk production.

d. prolactin stimulates lactation.

Its structure and function is similar to that of human growth


hormone. (WIKI)

15. Which is wrong about bacterial vaginosis ?

+a. Caused by lactobacillus

b. Cause 2nd trimester abortion .

BV jadi dari banyak jenis bakteria, it cause 2nd trisem abortion, screening
cari bantang ni timee 16 week

16. Lactational mother comes after 7 weeks of amenorrhea, complaining


of abdominal pain, wt's the most imp invx

: a. U/S

+b. B.HCG (to exclude ectopic.

c.urine analysis and culture

d.endocervical swab

jangan ingat sbb dia lactating dia xleh beranak, lactation bkn 100%
infertility

17. Contraindicated drug in Pregnancy:

a. ampicillin

b. cephalosporin

c. Zidovudine

+d. tetracyclin

Teeth discolouration

18.What's the risk of complex atypical endometrial hyperplasia to progress


into endometrial Ca : a.1%
+b.25 %

19. Not a risk factor for genital prolapse: a. Nullparity .

El3aks

20. We don't use clomiphene citrate in : a. Kallman's syndrome

Kallmann syndrome occurs when the hypothalamic neurons that are


responsible for releasing gonadotropin-releasing hormone (GnRH neurons)
fail to migrate into the hypothalamus during embryonic development

21. Not a relative contraindication for uterine embolization:

a. intramural fibroid

b. future pregnancy

c. previous internal iliac ligation

Explanation : Contraindications of UAE

1. Malignancy
2. Pregnancy
3. Menopause
4. Extensive adenomyosis
5. Preserve fertility
6. Submucosal fibroids (important)
Source : 6th yr slide ―Management of Pelvic Masses‖

22. False about fibroid:

a. Something related to Ca and parathyroid hormone

b. polycythemia
Explanation : Fibroids is an estrogen sensitive tumor, thus anything that
cause increase in estrogen will lead to increase risk of fibroids such as
hyperthyroidism (not hyperparathyroidism). Fibroids also may lead to
polycythemia due to production of erythropoietin.

Source : 5th yr lect note ―Uterine Fibroids & website


http://www.hindawi.com/journals/ogi/2012/539365/ (polycythemia)

23.Can't be treated with laparoscope :

a. Submucosal fibroid.

Explanation : Treatment of fibroids

1. Hysterectomy
2. Myomectomy
3. Endometrial ablation (for submucosa) --- this is done by
hysteroscopy
4. Resection (for pedunculated)
Source : 6th yr slide ―Management of Pelvic Masses‖

24. Can‘t be treated vaginally:

a. Ovarian (or adnexial . mass)

Explanation : One of the options for treatment of ovarian cancer is total


abdominal hysterectomy (not vaginal hysterectomy) + BSO. Most of
ovarian cancer are treated by TAH while cervical and endometrial are
treated by vaginal hysterectomy.
Source : 6th yr slide ―Management of Pelvic Masses‖ & website
http://www.mayoclinic.org/tests-procedures/vaginal-
hysterectomy/basics/why-its-done/prc-20020565

25. Not associated with bacterial vaginosis:

a. Inflammed vulva

Explanation : Symptoms of BV

1. Offensive fishy smelling discharge (thin, homogenous,


adherant to the walls of the vagina,white/yellow)
2. Smell is noticeable around the time of menses, or
following intercourse.
Source : 5th yr slide ―Vaginal Discharge‖

26. Primi, 38 weeks comes with breech …. ANC is otherwise normal…wt's


your next step:

a. ECV

b. positional excersie.

Explanation : Majority of causes of breech presentation is due to


prematurity, and most of them will turn to vertex by the time the GA
reaches 36 weeks, thus ECV is done at 36th week.

Source : 5th year slide ―Malpresentation‖

27. 30 y old pregnant comes with blurred vision and +3 proteinuria , BP 150
/100…. your next step :

a. Arrange for delivery .


b. Give hydralazine

c. Give steroid then go for delivery .

Explanation : Management of severe PET is delivery regardless of GA

Source : 6th yr slide ―Management of Patient with Severe PET & Eclampsia‖

28. False about RH isoimmunization :

a. ABO incompatipility increase the risk of RH isoimmunization

b. father has to be RH positive

c. mother should be RH.

Explanation : ABO incompatibility is protective against Rh iso-immunization


because the RBCs are immediately agglutinated and hemolyzed so that
the Rh antigen is not available to provide antigenetic stimulus.

Source :
http://www.ncbi.nlm.nih.gov/pmc/articles/PMC1932164/?page=1

29. Multiple pregnancies is not associated with:

a. fetal distress

b. shoulder dystocia

c. fetal anomalies

Explanation : Complications of multiple pregnancy:

1. Miscarriage: in singleton pregnancies the chance is only


1% while in dichorionic twins it's 2% and in monochorionic
it's raise to reach 12%.
2. Preterm delivery: in singleton again it's only 1%, In
dichorionic it's 5% and in monochorionic it reaches 10%.
3. Intra uterine growth restriction: in singleton 5%, in
dichorionic 20% and in monochorionic is double of
dichorionic chances so 40%.
4. Fetal abnormalities and defects: in dichorionic it's twice
as high as the singleton chances to have and in
monochorionic it's 4 times higher.
Source : 5th yr lec note ―Multiple Pregnancies‖

30. Not indication for anti.D?

a. Preterm labor .

b. Manual removal of placenta

Explanation : When the feto-maternal leakage happens?

1. Cesarean section C.S


2. Ectopic pregnancy
3. Abortion whether its spontaneous or therapeutic
4. In case of abruption placenta
5. Iatrogenic with the choriocentesis or amniocentesis
6. Sever preeclampsia
7. Silent cases and it is the worst type because we cant
predict it and after a time we do indirect coombs test
and we find anti D
Source : 5th yr lec note ―Rh Iso-immunization‖

31. Wrong about cervical cancer:

a. It's always coming from preinvasive cervical lesion.


Explanation : I‘m sorry coz I couldn‘t find the answer for this question 

32. Correct about cervical cancer:

a. Most common type is squamous cell ca.

Explanation : Histopathology

1. Squamous cell carcinoma – 69%


2. Adenocarcinoma (including adenosquamous) – 25%
3. Other histology (e.g. small cell carcinoma, lymphoma,
sarcoma,…etc.) – 6%
Source : 5th yr slide ―Cervical Cancer‖

33. Q about HM ……wrong about it:

a. Patient can be pregnant after 3 months.

Explanation : Patients need to use effective contraception during entire


interval of follow-up i.e weekly until normal B-hCG for 3 consecutive weeks
then monthly until normal B-hCG for 6 consecutive months

Source : 5th yr slide ―Gestational Trophoblastic Neoplasia‖

34. True about Down syndrome caused by translocation:

a. constitutes more than 90% of cases

b. increased in the frequency if the mother has balanced translocation

Explanation : I think the answer is wrong. Here what I found from the
website ―People with translocation Down syndrome can inherit the
condition from an unaffected parent. The parent carries a rearrangement
of genetic material between chromosome 21 and another chromosome.
This rearrangement is called a balanced translocation. No genetic
material is gained or lost in a balanced translocation, so these
chromosomal changes usually do not cause any health problems.
However, as this translocation is passed to the next generation, it can
become unbalanced. People who inherit an unbalanced translocation
involving chromosome 21 may have extra genetic material from
chromosome 21, which causes Down syndrome.‖

Etiology of Down‘s Syndrome

1. 95% is due to meiotic nondisjunction.


2. 4% is due to unbalanced translocation.
3. 1% have mosaic type
Source : http://ghr.nlm.nih.gov/condition/down-syndrome & 6th yr slide
―Approach to fetal Anomalies‖

35. Wrong about recurrent miscarriage:

a. Most cases happen in 2nd trimester

Explanation : Risk is higher with older maternal age and very early
gestational age

(e.g, less than 6 weeks of gestation the risk of miscarriage is 22-57% versus
15% at 6-10 weeks & 2-3% after 10 weeks)

Source : 6th yr slide ―Thrombophilia & Recurrent Miscarriage in Pregnancy‖

36.First thing to do in shoulder dytocia after calling for help is :

a. to flex her knees and hips

b. do episiotomy and do head traction


Explanation : Management of shoulder dystocia ―ALARMER‖

A – Ask for help

L – Leg hyperflexion (McRobert‘s maneuver)

A – Anterior shoulder disimpaction

R – Rubin Maneuver

M – Manual delivery of posterior arm

E – Eposiotomy

R – Roll over on all fours

Source : https://en.wikipedia.org/wiki/Shoulder_dystocia

37. Can't use forceps when:

a. Membrane is rupture and vertex presentation but cervix is not fully


dilated

Explanation : Preequisites for instrumental delivery

1. Engagement
2. Fully dilated cervix
3. Fetal maturity
4. Raptured membranes
5. Fetal position and presentation
6. Empty bladder
7. Presence of effective contractions
Source :6th yr lec note ―Obs & Gyne Clinical Skill Lab‖
38. Wrong about STD:

a. Secondary herpes comes with multiple painful vesicles (primary does..

Explanation : see table below

Source : 5th yr slide ―STD‖

Primary GH Recurrent GH

occurrence No HSV-1 or HSV-2 + HSV antibodies to


antibodies the same serotype

lesions Multiple, bilateral, Fewer, unilateral, less


painful vesicles painful vesicles

Systemic symptoms Usually present rare

healing In 14 – 21 days In 5 – 7 days

scaring No No

Complications in Increase risk for Rare


pregnancy obstetric & neonatal
complications

Vertically transmission 50% of infants (in 4% of infants(in


vaginal delivery) vaginal delivery)
39. Diabetes type one is associated with all the following except:

a. macrosomia .

Explanation : I think the answer is wrong because this is what I found from
the website ―Pregestational and gestational diabetes result in fetal
macrosomia in as many as 50% of pregnancies complicated by
gestational diabetes and in 40% of those complicated by type 1 diabetes
mellitus‖

Source : http://emedicine.medscape.com/article/262679-clinical

40. About Endometriosis all are correct, except:

a. tubal factor most common cause of infertility

b. symptoms usually at luteal phase .

Explanation : I didn‘t find the most common cause of infertility in


endometriosis but in Ten Teachers(Gynecology) page 107 there is a table
about the mechanism of infertility in endometriosis. I put it in the next
page.

Ovarian function Luteolysis caused by Prostaglandin


F2
Oocyte maturation defecrs
Endocrinopathies
Luteinized unruptured follicle
syndrome
Altered prolactin release
Anovulation
Tubal function Impaired fimbrial oocyte pick-up
Altered tubal mobility
Coital function Deep dyspareunia – reduced
coital frequency
Sperm function Antibodies causing inactivation
Macrophage phagocytosis of
spermatozoa
Early pregnancy failure Prostaglandin induced
Immune reaction
Luteal phase deficiency

5th Year Final Exam 2011 :


1. All of the following decrease vaginal bleeding except:

a. Copper IUCD

b. LHRH analogue

c. NSAIDs

One of the S/E of Cu IUCD is menorrhagia, which opposite with Mirena –


can be used in TT of menorrhagia. (Gyne 10teacher page 72)

LHRH analogue  -ve feedback  menopause symptom (also TT of


menorrhagia)

NSAIDs reduce prostaglandin  reduce menorrhagia (but less effective


than other drugs that used in TT of menorrhagia)

2. Which of the following is not sexually transmitted?

a. Bacterial vaginosis
b. Trichomoniasis

c. Chancroid

Bacterial vaginosis – overgrowth of polymicrobial in vagina. It is vaginal


infection not STDs.

Trichomoniasis – flagellated protozoa

Chancroid – bacterial infection by Hemophilus ducreyi

3. All of the followings concerning PET are correct except:

a. Being Primigravida is most important risk factor

b. Hypotensive agents stops disease progression

c. Most important risk to fetus is prematurity

PET is a progressive disease, the definitive TT is termination of pregnancy


(cause prematurity). It is disease of primigravida, can be in multipara with
history of PET. Aim of hypotensive agent is to reduce risk of maternal
cerebrovascular accident (CVA).

4. A pimigravida presented to you at 34 week gestation with blurring of


vision. Her blood pressure was 150/100, how would u manage this
patient?

a. Arrange for delivery

b. Give IV Hydralazine & reevaluate symptoms

c. Give IV steroids & wait 48hr then deliver her

It is eclampsia (with neurological symptom). Definitive TT is termination of


pregnancy. Severe PET & eclampsia, we deliver the baby after
stabilization, regardless the gestational age. (Hacker, page 177)

But in mild-PET, if <34 weeks, give steroids and monitor. If 34-37 weeks, wait
and monitor. If >37weeks, we deliver. (lecture note)
5. All of the following are complications of pregnancy preceded by DM
except?

a. Neural tube defect

b. Fetal death

c. Post maturity

Gestational diabetes can cause prematurity (not post maturity). Other,


disproportional macrosomia, delayed organ maturity, congenital
malformation, sudden IUFD, IUGR and increase abortion. (Fetal
complication)

6. All are done in prenatal testing except:

a. Urine analysis for protein

b. Rubella

c. Uric acid

Uric acid – we don’t do it. Rubella can cause miscarriage, stillbirth or birth
defects in fetus. That’s why we screen, non-immune woman should avoid
infectious contact and take vaccine after pregnancy. We also do CBC,
blood group (rhesus), urinalysis, hep B, HIV, syphilis, hemoglobinopathies.
(Gyne 10Teacher page 55,56)

7. Which of the following is incorrect regarding DM in pregnancy?

a. There’s an increase in fetal anomalies in gestational diabetes

b. Fetal death can still occur even when DM is well controlled

c. Macrosomia can occur even when glucose levels are well controlled
Well controlled DM by diet can reduced DM complication except fetal
macrosomia & maternal HXN (lecture note), but maybe fetal death
also???

Gestational diabetes gradually develop and is least pronounced during


1st & early 2nd trimester (embro/organogenesis). Birth defects usually
develop in 1st trimester. Unlike pre-existing DM, which increase fetal
anomalies. (wiki, complication GDM)

8. In preterm prelabor rupture of membranes, all are suggestive of


chorioamnionitis except:

a. Bloody vaginal discharge

b. Greenish vaginal discharge

c. A small for date uterus

d. Tachycardia

Chorioamnionitis diagnosed clinically, maternal fever + at least 2 of


following: (maternal leukocytosis/ maternal tachycardia/ fetal
tachycardia/ uterine tenderness/ foul odor amniotic fluid)

Maybe small date uterus indicates ROM by all causes.

9. Which of the followings is correct about endometrial adenocarcinoma?

a. More common in PCOS

b. Physical exam can be diagnostic

c. In advanced stages, surgery is main stay of treatment

One of the endometrial CA risk factor is unopposed estrogen. PCOS


secrete androgen (which converted to estrogen in adipose tissue).

Definitive diagnosis is by endometrial biopsy. Surgery is the main TT.


10. All concerning CIN are correct except:

a. D & C is enough in CIN III

b. HPV (6, 11) cause condyloma & CIN I

c. HPV (16, 18) cause CINII, III

d. 35% of CIN III progress to invasive CA in 10 years

e. CIN I rarely progress to invasive CA

Low grade (CIN 1) & high grade (CIN 2 & 3). Of course D&C in not
enough. TT for CIN – large loop excision of transformation zone LLETZ,
cryotherapy, “cold coagulation” using hot prob.

11. Prostaglandin E1 causes all the following except:

a. Flushing

b. Pyrexia

c. Hypertension

d. Seizure

e. Apnea

PG E1 (misoprostol) is used in missed abortion (terminate pregnancy).


Common S/E: nausea, vomiting, fever. It causes hypotension, not
hypertension.

12. A 25 year old female found to have a 5 *3 cm ovarian cyst, X ray


showed calcification, which is the most likely diagnosis:

a. Mature cystic teratoma

b. Dysgerminoma
c. Yolk sac tumor

Maybe because of calcification? Mature cystic teratoma contain tissue


from 3 embroyonic layers mesoderm, endoderm & ectoderm which may
contain teeth, bone, hair etc.

13. Which of the following is normal in semen analysis?

a. 35 million sperm/ ml

b. 2% normal motility

c. Liquification in 2 hours

d. Normal shape 3%

Normal parameter : Volume >2mL / pH >7.2 / sperm conc. >20million per


mL / sperm num. >40million per ejaculate / motility >50% grade A & B /
morphology >30% normal forms (Gyne 10teacher, page 90)

14. Which of the following indicate ovulation if done at mid cycle?

a. LH surge

b. Serum Estriol

c. Progesterone

d. B. HCG

e. FSH
LH surge indicate ovulation.

15. A 35 year old female presented with history of 6 month duration


amenorrhea, all can be a cause except:

a. Testicular feminization syndrome

b. Pregnancy

c. Lactation

d. Menopause

Testicular feminization syndrome is also known as Androgen insensitivity


syndrome. It can come with female external genitalia without internal
genitalia (tubes/uterus). So, it is primary amenorrhea. The case is about
secondary amenorrhea.

16. An asymptomatic primi-gravida presented to you at 10 week


gestation, urine culture showed 100000 colonies of E.coli, how to manage
this patient?

a. Admit & give IV antibiotic


b. Give oral antibiotic

c. Repeat culture

d. Reassurance

The case is about asymptomatic bacteriuria. Asymptomatic bacteriuria


and acute cystitis can be treated as outpatient with antibiotic. But acute
pyelonephritis should be admitted. (Hacker, UTI page 211,212)

17. All increase in pregnancy except:

a. WBC

b. Heart rate

c. Cardiac output

d. Peripheral vascular resistance

e. RBC mass

Blood pressure in pregnant woman is slightly decreases because of


reduced peripheral resistance. Towards end of pregnancy it increases.
(Hacker page 56,57)

18. All are found in fetal distress management, except:

a. put the patient in supine position

b. stop oxytocin injection

c. oxygen mask

d. rapid infusion (dehydrated)

Actually, put patient into a left lateral position. Supine position makes
venocaval compression and reduced uterine blood flow. Others are true.

19. The most common presentation of endometriosis is:


a. menorrhagia

b. dysmenorhea

c. intermentstrual bleeding

Classical clinical features – severe cyclical non-colicky pelvic pain


restricted to around the time of mens. Sometimes associated with heavy
mens loss. (Gyne 10teacher, page 105) It is 2ndary dysmenorrhea.

20. The most common presentation of PID is:

a. dysparonea

b. menorrhagia

c. vaginal discharge

d. abdominal pain

Most common is abdominal pain. Deep dysparunea, menorrhagia and


vaginal discharge also can be presented.

21. 35 gestation week and FSH is 31 cm >> all could be the cause, except:

a. oligohydraminous

b. growth restriction

c. rupture of membrane

d. preterm labor

Maybe it’s not FSH. Maybe it’s actually fundal height. All can cause small
for gestational age except preterm labor.

22. All are treatment of genital prolapsed, except:


a. vaginalal hysterectomy

b. anterior wall colporaphy

c. hysteroplexy

All of them are in the TT of prolapsed. (lecture note) ????

23. All are associated with PCOS, Except:

a. obesity

b. endometrial hyperplasia

c. amenorrhea

d. high FSH:LH ratio

It is actually high LH:FSH ratio. LH will be elevated, FSH maybe normal or


low in PCOS. All other answers are true.

24. All to be done normally to a 25y/o married with menorrhagea, EXCEPT:

a. histology study to a biopsy (obtained from endometrium)

b. ultrasound

c. hormone study

d. laparoscopy

Endometrial biopsy is taken when the menorrhagic patient is older than 40


y/o.

25. all are symptoms for urinary ** in women, except:

a. straining to void

b. urgency

c. urge incontinence
d. stress incontinence

Straining to void mostly in men??? BPH??

26. Regarding bacterial vaginosis, all are true, except:

a. spouse shouldn‘t be treated

b. metronidazole is the DOC

c. foul fishy smell.

d. None of above.

All of other answers are true.

27. Not a symptom of cervical CA:

a. vaginal discharge

b. abnormal vaginal bleeding

c. weight loss

d. Pelvic pain

Weight loss maybe present in advanced cases. But not specific kot?? All
others are true.

28. All are complications of twin pregnancy, Except:

a. DM

b. preterm labor

c. postpartum bleeding

d. anemia
DM is not complication of multiple pregnancy. All other answer are true.
Other complications are miscarriage, perinatal mortality, death of one of
the fetus, fetal growth restriction, fetal abnormalities and chromosomal
defect. And unique complications of monochorionic are TTTS and a-
cardiac.

29. About postpartum hemorrhage one is incorrect:

a. atony : placenta previa

b. atony: twin pregnancy

c. atony: breech presentation

d. secondary hemorrhage: retained particles

Breech presentation doesn’t cause uterine atony. Maybe can cause


trauma by forceps. All others are true.

30. Treatment of postpartum blues is:

a. reassurance

b. psychological counseling

c. drug (something)

d. ECT

Postpartum blues is usually short-lived (3-5 days after delivery), it is benign


condition, no pharmacotherapy is needed. Provide support and
education has been shown to have a +ve effect.

31. About ectopic pregnancy only one is true

a. most of the time discovered incidentally during ANC


b. negative urine pregnancy test rule out ectopic pregnancy .

c. show with pelvic mass

Ectopic pregnancy shows positive pregnancy test because of B-HCG


elevated. Others are not true.

32. All cause post partum pyrexia, except:


a. DVT
b. chorioamnionitis
c. mastitis
d. sepsis
Ans :

Puerperal pyrexia is defined as a temperature of 38C or


higher on any 2 of first 10 days postpartum, exclusive of the
first 24h measured orally by a standard technique.
Causes :
1. Genital tract infection
2. UTI
3. Breast infection ( Mastitis)
4. URTI/LRTI
5. Thrombophlebitis and DVT
6. Wound infections ; Anemia
7. Non puerperal causes ; appendicitis

33. All of the following is a risk for PET, except :


a. smoking .
b. previous PET
c. genetic
d. obesity
Ans:
Risk factors Protective factors
1. Previous PET 1. Smoking
2. Nulliparity ( 8x more frequent) 2. Placenta Previa
3. Family Hx of PET 3. Exposure to Paternal Antigen
4. Extreme ages > 40y
5. Pre-existing medical
condition- DM/chronic
HTN/chronic renal
disease/SLE/antiphospholipid
Ab/
6. BMI ≥ 26.1 ( Obesity )
7. Obstetric complications-
Multiple
gestation/Hydatidiform
mole/Polyhydramnios/Hydrop
s Fetalis/
8. Low socioeconomic
9. Race ; Black
34. one of the following can be done for Rh testing:
a. indirect coomb's from cord
b. fetal bilirubin
c. maternal bilirubin
d. father rhesus antigens

35. PID causes all of the following, except:


a. infertility
b. ectopic pregnancy
c. cervical stenosis

Complication of PID :
1. Tuboovarian abscess
2. Endometrial and tubal damage – Infertility, ectopic
pregnancy
3. Chronic pelvic pain, menorrhagia, deep dyspareunia,
2’ dysmenorrhea
4. Adhesion
5. Recurrence
6. Fitz-Hugh-Curtis yndrome
36. ovulation occurs at:
a. day 14 of menstrual cycle
b. 14 days before menses
c. 7 days after menses
d. 21 days after menses
Ans;

Menstrual cycle : 28 +-7 days


1. Ovarian Cycle
i) Follicular phase
ii) Ovulation
iii) Luteal phase ( 14 days fixed)
2. Uterine Cycle
i)Proliferative
ii) Secretary
iii) Menstruation
Day of ovulation= Period length-14 (luteal
phase)

37. All of the following increase risk of Factors increase risk of folate
folate anemia, except : deficiency :
a. anticonvoulsant drugs 1. Anticonvulsant drugs
b. hemolytic anemia 2. Hemolytic anemia
c. trait thalasemia 3. Thalassemia
d. spherocytosis 4. Spherocytosis
5. Previous pregnancy
6. Low socioeconomic status
7. Vegetarian
IRON DEFICIENCY ANEMIA
1. Hemoglobin >10g/dL
2. MCV < 80
3. RDW > 15

Treatment
1. Oral supplement
- needs at least 30mg iron
per day
2. IM/IV iron
ANS : All of the above are true 3. Blood transfusion
- Prior delivery

38. About iron def anemia all true


except :
a. blood transfusion may be needed
during last trimister
b. iron supplement 10mg per day is
not enough .
c. None of above .

39. All true about cardiovascular changes during pregnancy, except:


a. stroke volume increase
b. volume increase 20% in singleton pregnancy .
Ans;

CVS changes during pregnancy


1. Increase
-CO, HR, SV
-plasma volume
-venous pressure ( varicose vein, hemorrhoids,LL edema)
2. Decrease
-PVR, BP
-Venous return
3. Other changes
-Hyperdynamic circulation
-Compression of IVS & pelvic v.
-Peripheral vasodilation
* Plasma volume increases up to 50% with significant in 1st trimester. Max
increase by 30th wk. This increases even greater with multiple fetuses.
(Kaplan USMLE Step 2)
Causes of fetal tachycardia
1. Maternal
- Fever
40. all raise heart rate of fetus, except: - Chorioamnionitis
a. mother taking pethidine . - Drugs
b. mother fever - Hyperthyroidism
c. chorioaminitis 2. Fetal
- Tachyarrhythmia
- Hypoxia
ANS : Pethidine causes poor variability
- Anemia
on CTG
- Infection/sepsis

41. Most common abnormal presentation is:


a. complete breech
b. frank breech .
c. brow presentation
Ans :

Malpresentation is presentation other than cephalic


vertex (95%).
1. Breech (3-4%)
- Frank, commonest
- Incomplete
- Footling
2. Brow
3. Face
4. Shoulder

42. All are absolute indication for C section, except:


a. previous uterine scar in midline incision (or something like that)
b. previous scar in uterine body
c. transverse presentation during labor
d. prolapsed cord with cervical dilatation 7cm and a live fetus
43. All of these causes growth restriction, Except:
a. age
b. race .
c. diabetes
d. dieting during pregnancy

CAUSES IUGR
FETAL MATERNAL PLACENTAL
1. Chromosome 1. Undernutrition, 1. Reduced
defects; trisomy 18, eg.poverty, eating uteroplacental
triploidy disorder perfusion
2. Single gene defect; 2. Maternal hypoxia, 2. Reduced feto-
Seckle‘s syndrome eg altitude, CHD placental perfusion
3. Structural 3. Drugs,smoking, 3. Infarction, abruption,
abnormalities; renal alcohol, cocaine placental tumors TTTS,
agenesis 4. Advanced diabetes velamentous cord
4. Incfections; CMV, 5. Uterine malformation insertion
toxoplasmosis
5. Multiple gestation

44. All are used for diagnosis of fibroid, except :


a. laparoscopy Diagnosis of fibroid :
b. hysteroscopy 1. History taking
c. hormonal level 2. Pelvic examination
Ans; 3. Ultrasound
4. Hysteroscopy
45. All are used for treatment of
5. Laparoscopy
endometriosis, except :
a. GNRH analouge
b. hysterectomy
c. danazol
d. NSAID
Treatment of endometriosis
1. Medical
- NSAIDS
- Induce pseudo-pregnancy ( COCP/synthetic
progestogen/Levonorgestrel)
- Induce pseudo-menopause ( Danazol/Gestrinone/GnRH
analogue)
2. Surgical
- Conservative
- Radical ( Hysterectomy + Oophorectomy )

*All can be given but : danazol not given anymore bcz of its side effects
,hysterectomy is done with oophorectomy (radical surgery), continous
progestrone is giving with estrogen

STD
46. All are STDs, except : 1. Chlamydia
a. toxoplasmosis 2. Gonorrhea
b. trachomanlis vaginalis ( 2nd most 3. Syphylis
common STD ) 4. Genital herpes
c. genital warts . ( HPV) 5. HPV
6. Trichomonas vaginalis
ANS : Toxoplasmosis is a parasitic disease
caused by Toxoplasma gondii

47. All are risk for endometrial Ca, except :


a. Obesity
b. Lynch syndrome
c. Smoking
Ans:

Risk factors : Protective factors ;


1. Nulliparity 1. Smoking
2. Overweight/obesity 2. Use of oral
3. Early menarche contraceptive
4. Late menopause 3. Use of
5. Associated factors- progesterone
DM/HTN/Thyroid/Fibroids/PCOS/Infertility/
Use of Tamoxifen/
6. Family Hx of breast,ovarian CA
7. Previous pelvic irradiation
Factors increase multiple pregnancy
48. All are risks for dizygotic 1. Race ; more in black, low in far east
twins, except : Asians.
a. Elderly 2. Maternal age ; older women more
b. GnRH likely
c. HMG 3. Parity
4. Family history
5. Assisted reproduction technique ;
IVF, IUI
49. Only one true about ectopic Ectopic
6. Human pregnancygonadotropin
menopausal triad : (
: hMG) 1. Secondary amenorrhea
a. May present as vaginal 2. Unilateral
7. Clomifene citrate
bleeding abdominal/pelvic pain
b. Amenorrhea essential for diagnosis 3. Vaginal bleeding
Treatment of Menorrhagia
1. Mefenamic acid ( Ponstan )
50. All can reduce menstrual 2. Antifibrinolytic
bleeding, except : 3. Progestogens
a. Low dose aspirin 4. COCP
b. COCP 5. Danazol
c. GnRH 6. GnRH analogue
7. Mirena
8. Surgery ( Resection and
Hysterectomy )
51. All are diagnostic for ovulation,
except :
a. level of E2 in luteal phase
b. endometrial biopsy before end of cycle

Diagnostic for ovulation


1. Basal body temperature
2. Cervix mucus alteration
3. Hormonal assay
- Serum progesterone at day 21st
- Serum prolactin
- Serum FSH/LH at first 2 days of
period
4. Endometrial biosy – day 21st
5. Ultrasound
6. Laparoscopy & Laparatomy
7. LH peak
8. Pregnancy

52. All disrupt bonding to the mother, except :


a. using forceps in delivery
b. Low socioeconomic
c. NICU

53. All cause abdominal pain in 20 wks gestation, except :


a. Ectopic pregnancy
b. Spontaneous miscarriage
c. Appendicitis
ANS :

All of the above can cause abdominal pain in 20 wks gestation

54. Only one is true about GDM:


a. GDM is risk for getting DM later on
b. Insulin used in almost all cases
* Yes GDM is one of risk factor for DM

55. About hot flushes , all are true except :


a. HRT used in mild case
b. Most resolve 1.5 yrs
c. Due to defect in thermoregulation at the level of hypothalamus

* Hot flushes are self-limited, resolved without treatment


within 1-5 years.
* HRT not used in mild cases due to side effect.

56. Most common ovarian tumor in pregnancy :


a. Corpus luteum cyst
b.Teratoma
c. Serous cystadenoma

* Functional cyst is the most common adnexal mass


in pregnancy.
* Most common ovarian tumor is serous
cystadenoma

57. About ovarian tumor , all are true, except:


a. Dysgerminoma associated with increase in alpha fetoprotein .
b. CA 125 marker for serous type

- Dysgerminoma – increase
LDH
- Choriocarcinoma – bhcG
- Yolk sac tumor – alpha
fetoprotein

58. Only one is true about stages of labor:


a. second stage is shorter in multiparus than in primigravida woman.
Stages of Labor
1. First stage
- Latent phase (<20 hrs in nuliparity/ <14 hrs in
multipara )
- Active phase (2-6hrs)
2. Second stage - < 2hrs
3. Third stage - < 30 mins
4. Fourth stage – up to 8 hrs
*Multipara always shorter than primigravida in any
stages.

59. Not a contraindication for OCP uses:


a. 35 and smoker
B. PB 140.160/90.120 (or something like that)
Contraindication for OCP
1. Smoker >35yo
2. Pregnancy
3. Circulatory diseases like IHD, CVA, Stroke, DVT,
uncontrolled HTN
4. Acute or severe liver disease
5. Focal migraine
6. Estrogen-dependant neoplasm
7. DM with vascular disease
8. Thrombophilia

60. in candidal infection, all true EXEPT:


a. metronidazole is the treatment of choice

Treatment of choice in candida is antifungal, not


antibiotics :
1. Topical antifungal
2. Oral antifungal
- Fuluconazole
- Ketoconazole

61. About abortion all are true, except :

a. U/S essential for incomplete abortion

Ans:

Incomplete abortion
1. History - heavy vaginal bleeding + POC + severe
abdominal pain
2. Examination – Bad general condition, dilated cervix, uterus
small for date
3. U/S – fetal heart absent.

*mainly depends on clinical diagnosis.

U/S essential for diagnosis in


1. Threatened abortion
2. Missed Abortion

62. About delivery all true, EXCEPT:


a. fetus head flexion increase during delivery

 Couldn’t find the answer, but head flexion is involved


in mechanism of labour.

63. Ideal or best treatment of eclampsia :


a. termination
b. bed rest & lying on the side

Management
1. Stabilization of the mother
- Protect airway and tongue.
- MgSO4 – prevent or treat
convulsion.
- Lower diastolic BP to 90-100 mm
Hg.
2. Prompt delivery

The Guinness World Records. 2003. p. 142 said :


” Islam is the world’s fastest-growing religion by number of conversions each year
, Although the religion began in Arabia , by 2002 80% of all believers in Islam lived
outside the Arab world . In the period 1990-2000, approximately 12.5 million
more people converted to Islam than to other Religions “ .
6th Year Final Exam 2010 :
1. One of the following is NOT found in a post.term newborn:

b. Hypoglycemia

- Infants at risk of developing symptomatic hypoglycemia:


o Infants with fetal growth restrictions.
o Infants of diabetics mothers
o Preterm infants
o Infants who have experienced significant hypoxia in labor
o Infants who are large for dates – possibility of undiagnosed
maternal gestational diabetes.

2. One of the following is NOT correct about active management of labor


:

a. Epistiotomy is routinely done

- Active management of labor has been developed to the extent


that an assurance is given to every woman who attends this hospital
that her first baby will be born within 12 hours.
o Protocols included:
 Admission only when labor was established
 Painful contractions with:
 Spontaneous rupture of membranes
 100% effacement, or
 Passage of blood stained mucus
 ARM on diagnosis of labor
 Aggrassive oxytocin augmentation for labor progress of
less than 1 cm/h with high duse oxytocin: 6 mlU/min initial
dose, increased by 6 mlU/min every 15 minutes to a
maximum of 40 miU/min.
o Effect decreased labor duration by 1.7 to 2.7 hours
 But increased cost
o Most important factor is delaying admission until active labor

- Episiotomy:
o It is a surgical incision through the perineum to enlarge the
vagina and assist childbirth.
o It is performed under local anesthesia and is closed (sutured)
after delivery.
o Episiotomy will only assist the birth if the head passed the
pelvic cavity
o Indications:
 Rigid muscles of the perineum
 Previous pelvic floor surgery.
 Complicated vaginal delivery (breech, shoulder
dystocia).
 Large baby.
 Fetal distress.
 Fetal malposition,eg. Occipito-posterior.

- Active management of third stage of labor reduces the incidence


of postpartum hemorrhage from 15% to 5%.
o Modern active management of the third stage is a package
of care which includes the following:
 IM injection of 10 IU of oxytocin, given as the anterior
shoulder of the baby is delivered, or immediately after
delivery of the baby;
 Early clamping and cutting of the umbilical cord
 Controlled cord traction.
3. One of the following is NOT correct about multiple pregnancy :

a. Clomiphine citrate increases the risk of monozygotic twins

- Clomiphine citrate increases the risk of dizygotic twins, not


monozygotic twins
- Factors that increase incidence for dizygotic twins:
o Race (blacks > whites)
o Parity
o Maternal age
o Family history (mother being part of a twin)
o Ovulation induction
 10% in clomifene citrate or human menopausal
gonadotropin (hMG)
 20% in IVF
- Incidence is constant for monozygotic twins

- Clomifene Citrate (Clomid):


o is a non-steroidal cyclical estrogen with an anti-estrogenic
activity.
o It occupies the estrogen receptors in the hypothalamus giving
false impression of lack of estrogen , increasing gonadotropins
and stimulating the ovary to produce more follicles.
o Used in hypothalamic pituitary dysfunction causes (e.g. PCOS)
o BMI greater than 27.2 or greater than 30.6 could cause
resistance
o Contraindicated in cases of ovarian cysts and liver disease.
o Risk : multiple pregnancies , OHSS.
4. The most important cause of fetal risk in multiple pregnancy is:

a. Prematurity

- Causes of perinatal morbidity & mortality in twins:


o Prematurity
o Congenital anomalies
o Stillbirths
o RDS
o Birth trauma
o Cerebral hemorrhage
o Birth asphyxia
o Birth anoxia

- Multiple pregnancy increased rate of all pregnancy complications


except post-term delivery:
o Increased risk of perinatal morbidity & mortality
o Preterm delivery, prematurity & PROM
o Low birth weight (50% < 2500 g) & IUGR (40%)
o Malpresentations & increased risk for operative deliveries
o APH
o Mortality rate is 2-3x higher in monochorionic
o Conjoined twins
o Interplacental vascular anastomoses
o Twin-twin transfusion syndrome
o Fetal malformations
o Umbilical cord abnormalities (prolapsed)
5. One of the following is an indication for CS:

a. Tranverse lie in the 1st twin

- Common indications fo CS:


6. One of the following is NOT correct about warfarin:

a. It's contraindicated in lactation.


b. It causes structural abnormalities in the fetus
c. Monitered by INR

- Warfarin:
o Warfarin is a vitamin K antagonist, which crosses the placenta,
carries the risks for fetal hemorrhage and teratogenesis and,
with few exceptions, should only be used in the postpartum
period.
o The dose is 2.5-5 mg/twice daily.
o Action: inhibition of synthesis of Vit-K dependent factors (II, VII,
IX & X).
o The International Normalized Ratio (INR) is commonly used to
measure the effects of warfarin, and the target INR is 2.5
(range, 2.0 to 3.0).
- Disadvantages of Warfarin:
o Bleeding
o Teratogenic effects (condrodysplasia punctata, cerebral
hemorrhage, calcification & microcephaly) if given in the 1st
trimester
o Fetal & neonatal cerebral hemorrhage if given after 36 weeks
- Effect of warfarin is not easily or rapidly reversed, because warfarin
takes 3 days to disappear from the circulation
- Anti-dote : FFP
- Comparisons between heparin and warfarin:
Heparin Warfarin
Cross placenta Does not Do
Causing Not is small dose
Haemorrhage
Reversed Easily & rapidly Not
Antidote Protamine Sulphate FFP
Excreted in breast Not Not
Disadvantages Osteoporosis & Bleeding, teratogenic
thrombocytopenia effects

7. True about ectopic pregnancy:


a. it can present without missed period.

The classic clinical triad of ectopic pregnancy is pain, amenorrhea, and


vaginal bleeding; unfortunately, only about 50% of patients present with
all 3 symptoms. About 40-50% of patients with an ectopic pregnancy
present with vaginal bleeding, 50% have a palpable adnexal mass, and
75% may have abdominal tenderness. In one case series of ectopic
pregnancies, abdominal pain presented in 98.6% of patients, amenorrhea
in 74.1% of them, and irregular vaginal bleeding in 56.4% of patients.

8. The stage of embryology where the embryo is a solid mass of 16 cells is


called :

a. Morula

- Zygote → 2-cell → 4-cell → 8-cell → Morula → Blastocyst

9. One of the following statements about FSH is correct:

a. It works on granulosa cells to secrete estradiol

- Activity of FSH:
o In both males and females, FSH stimulates the maturation
of germ cells.
o In males, FSH induces Sertoli cells to secrete androgen-binding
proteins (ABPs), regulated by inhibin's negative
feedback mechanism on the anterior pituitary.
o In females, FSH initiates follicular growth, specifically
affecting granulosa cells. With the concomitant rise in inhibin B,
FSH levels then decline in the late follicular phase. This seems to
be critical in selecting only the most advanced follicle to
proceed to ovulation. At the end of the luteal phase, there is a
slight rise in FSH that seems to be of importance to start the
next ovulatory cycle.
- When the follicle matures and reaches 8–10 mm in diameter it starts
to secrete significant amounts of estradiol. Normally in humans only
one follicle becomes dominant and survives to grow to 18–30 mm in
size and ovulate, the remaining follicles in the cohort undergo
atresia. The sharp increase in estradiol production by the dominant
follicle (possibly along with a decrease in gonadotrophin surge-
attenuating factor) cause a positive effect on the hypothalamus
and pituitary and rapid GnRH pulses occur and an LH surge results.
- The increase in serum estradiol levels cause a decrease in FSH
production by inhibiting GnRH production in the hypothalamus
10. The most common cause of uterine rupture in labor is:

a. Previous CS
b. Congenital uterine abnormalities
c. Obstructed labor
d. Induced labor

- Etiology of Uterine rupture:


o Incidence is 0.5%
o The most important predisposing factors are
1- Previous C-Section
 Site
• Lower segment  less than 1 %
• Classical  4-7 %
 Number
• Single  up to 0.5 %
• Double  rise up to 2%
2- Instrumental delivery
3- Trauma
11. One of following statements is NOT correct about fetal distress in labor:

a. CS immediately performed
b. Detected by CTG
c. Confirmed by fetal scalp blood sampling

- Fetal Distress Treatment


o The primary treatment used for non-reassuring fetal status is
intrauterine resuscitation. This will help prevent any
unnecessary procedures. Some means of intrauterine
resuscitation include:
 Changing the mother‘s position
 Ensuring the mother is well-hydrated
 Ensuring the mother has adequate oxygen
 Amnioinfusion (the insertion of fluid into the amniotic
cavity to alleviate compression of the umbilical cord)
 Tocolysis (a therapy used to delay preterm labor by
temporarily stopping contractions)
 Intravenous hypertonic dextrose
o Nonetheless, there are cases in which an emergency
cesarean section is necessary. However, due to the over-
diagnosis of fetal distress and potential misinterpretation of the
fetal heart rate, it is recommended to confirm a potential fetal
distress diagnosis with a fetal blood acid base study. Overall,
this condition points to the importance of prenatal care and
proper monitoring of the mother and fetus throughout
pregnancy.
12. One of the following is NOT correct about intrahepatic cholestasis of
pregnancy:

a. Intense itching
b. Preterm labor
c. Intrauterine fetal death
d. Characterestic skin rash

- Intrahepatic cholestasis of pregnancy


- Most women with this condition present in third trimester with itching
without a rash. Typically, the itching is localized to the palms of the
hands and soles of the feet but can be anywhere on the body.
- Hallmarks of ICP include the following symptoms:
o Most common:
 Itching, in particular but not limited to that of the palms
of the hands and soles of the feet, without presence of a
rash
 Itching that increases in the evening
 Itching that does not respond favorably to anti-
histamines or other anti-itch remedies
 Often, elevated LFT results as well as serum bile acid
counts
o Less common:
 Darker urine
 Lighter stools
 Increased clotting time (due to possibly associated
vitamin K deficiency)
 Fatigue
 Increased nausea
 Decrease in appetite
 Jaundice
 Upper right quadrant pain
o It is important to note that not all ICP sufferers have all of the
above symptoms. For example, Jaundice only occurs in
relatively small subset of cases, and in some cases abnormal
lab results were not seen until 15 weeks or more after the onset
of symptoms.
- Risk if untreated:
o Maternal consequences include the following:
 Itching, which can become intense and debilitating
 Premature labor
 Deranged clotting, which requires Vitamin K
o Fetal consequences include:
 Fetal distress
 Meconium ingestion
 Meconium aspiration syndrome
 Stillbirth
o Delivery has been recommended in the 38th week when lung
maturity has been established.

13. One of the following is NOT true about pregnant patients on


anti.epileptics:

a. it's not contraindicated in lactation


b. Patients should take folic acid supplements
c. The probability of congenital abnormalities in the fetus is 3%
d. it's more risky not to take the medications to the mother and fetus than
to take them.

- Uncontrolled epilepsy in a pregnant woman is a serious and


potentially life-threatening condition for both mother and child. Most
pregnant women with epilepsy will need to take at least one
antiepileptic drug. The goal for all concerned is a healthy, seizure-
free mother and an undamaged child. The following somewhat
contradictory issues need to be considered concurrently.
o The optimum treatment of the mother's epilepsy requires that
the most appropriate antiepileptic drug be used in effective
doses throughout pregnancy. This requires knowledge of
specific epileptic syndromes and also antiepileptic drug
pharmacokinetics before, during and after pregnancy.
o Any adverse effect that the antiepileptic drug could have on
the developing child needs to be avoided or minimised during
pregnancy and lactation.
- Fetal abnormality
o Women with epilepsy taking antiepileptic drugs have a
greater (2-3 times) risk than other women of having a baby
with a fetal abnormality. Taking more than one antiepileptic
drug carries a higher risk than monotherapy. Major
malformations, such as congenital heart disease, neural tube
defects, urogenital defects and cleft lips or palates, occur in
about 3-7% of women with epilepsy who take antiepileptic
drugs, although a substantially higher risk is attributed to high
doses of valproate (greater than 1400 mg/day).

Prescription of antiepileptic drugs (AEDs) to women with epilepsy during


the reproductive years requires maintaining a precarious balance
between controlling seizures and minimizing fetal exposure to harmful
effects of the medications. Epilepsy is the most common neurologic
disorder that requires continuous treatment during pregnancy, and AEDs
are among the most frequently administered prolonged teratogen
exposures. Fetal anticonvulsant syndrome consists of various combinations
of the following features: minor anomalies, major congenital
malformations, intrauterine growth retardation, cognitive dysfunction,
microcephaly, and infant mortality.

Certain general treatment paradigms to improve both maternal and fetal


outcomes are commonly accepted, although many of the assumptions
on which the paradigms are based are backed up by little objective
evidence. Treatment standards include maintaining the best possible
seizure control for the mother, or at a minimum, control of generalized
tonic–clonic seizures (GTCSs). GTCSs can cause maternal or fetal hypoxia
and acidosis; in addition, GTCSs have been associated with fetal
intracranial hemorrhages, miscarriages, and stillbirths. The effects of
nonconvulsive seizures on the developing fetus are not fully understood
but may include fetal bradycardia and the consequences of trauma.
Determining the direct effects on the developing fetus of exposure to
AEDs versus exposure to seizures obviously does not lend itself to a
randomized trial. Nevertheless, most women will need to be maintained
on an AED regimen during pregnancy. Selection of the best regimen for a
particular patient must take into account the types of seizures and
epilepsy syndrome, previous response to medications, family history of
birth defects, side effects, and potential teratogenicity.

Another treatment paradigm that is generally accepted as being


important is the use of supplemental folic acid before conception and
during pregnancy . Proven benefits of supplemental folic acid, however,
are based on studies of women without epilepsy in the general
population or women at high risk for neural tube defects, with a family
history of the defects. Studies specifically designed to determine effects of
fetal AED exposure have failed to show a protective effect against major
malformations with folic acid administration. These findings could be due
either to folic acid's inability to affect AED teratogenic mechanisms or,
possibly, to the prescription of inadequate dosage levels of folic acid.
Some more recent publications include recommendations that folic acid
supplementation be administered at higher dosages of 4 mg to 5 mg per
day when prescribing AEDs or, at least, when prescribing carbamazepine
(CBZ) or valproate (VPA).

An additional issue that must be considered when treating women with


epilepsy during their reproductive years is that enzyme-inducing AEDs
must be used cautiously with hormonal contraceptive medications. Given
the importance of planned pregnancies, woman need to be informed of
the lower efficacy of hormonal contraceptives taken in conjunction with
enzyme-inducing AEDs and encouraged to use backup barrier methods.
14. All of the following increase the risk of thromboembolism in pregnancy
except:

a. Thrombocytopenia.

Risk factor of thromboembolism:


1. Pregnancy.
2. Maternal age > 35
3. CS
4. Obesity.
5. Parity >4
6. Prolonged hospital stay.
7. Previous history of thromboembolism
8. Family hx
9. Thrombophilia
10. Anti-cardiolipin syndrome
11. Cardiac disease.
12. Sickle cell disease
13. Gross varicose viens
14. Blood group other than O
15. A patient who's 7 weeks gestation presented with abdominal pain and
mild vaginal bleeding. US shows a 30 mm empty gestational sac. The dx is:

a. Missed abortion
b. Threatened abortion
c. Complete abortion

Early pregnancy
B-hcG

EP
No IU sac
B-hcG

Complete Abortion

IU sac Incomplete Ab

Passed tissue
EP
Decidual sac
histo

U/S
Threatened Ab
No tissue passed
counseling

No fetal heart 
> 6 mm
Missed Ab
CRL
No fetal Heart
=< 6mm repeat after 7
days
16. US is mandatory in the dx of all of the following except:

a. Inevitable abortion
b. Threatened abortion
c. Missed abortion

Inevitable & incomplete abortions:


- History:
o Heavy vaginal bleeding with:
 No passage of products conception (inevitable)
 Passage of products of conception (incomplete)
o Severe lower abdominal pain which follows the bleeding
- Examinations:
o Poor general condition (maybe shock).
o Dilated cervix & products of conception may be passing
through the os
o The uterus size:
 Correct for date (inevitable)
 Small for date (incomplete)
- U/S (not essential for diagnoses): fetal heart activity may or may not
present.
- Management:
1. CBC, blood grouping, XM 2 units of blood
2. Resuscitation → large IV line, fluids & blood transfusion
3. Oxytoxic drugs → Ergometrine 0.5 mg IM + Oxytocin infusion
(20-40 units in 500 cc saline) or PG to encourage the uterine
contraction & minimize the bleeding.
4. Evacuation & curettage.
5. Post-abortion management.
17. One of the following is wrong about HRT in menopause:

a. Tibilone is an estrogenic HRT


b. Breast tenderness is one of the start up symptoms
c. Abnormal vaginal bleeding is a contraindication
d. Uterine fibroid is a relative contraindication

Tibolone (INN, USAN, BAN) (brand name Livial, Tibofem) is a synthetic


steroid drug with estrogenic, progestogenic, and weak androgenic
actions which was introduced in 1988 and is used widely in Europe, Asia,
Australasia, and, with the exception of the United States (where it is not
available), the rest of the world. It is used mainly for treatment of
endometriosis, as well as hormone replacement therapy in post-
menopausal women. Tibolone has similar or greater efficacy compared to
older hormone replacement drugs, but shares a similar side effect profile.
It has also been investigated as a possible treatment for female sexual
dysfunction.

HRT Contraindications:
Absolute Suspected pregnancy
Breast cancer
Endometrial cancer
Active liver disease
Uncontrolled Hypertension
Known VTE
Known Thrombophilia.
Otosclerosis
Relative Uninvestigated abnormal
bleeding
Large uterine fibroids
Past history of benign breast
disease
Past history of VTE
Chronis stable liver disease
Migraine with aura
18. The age of menopause is:

a. Genetically determined
b. Affected by OCP use
c. Smoking has no effect
d. Affected by the age of last pregnancy

Menopause Age:

- The Top Factor:


o There are a number of factors that affect a woman‘s age at
menopause, but one is more important than any other: the
age her mother experienced menopause.
o ―Menopause is strongly genetically linked, so you‘re very likely
to fall within a few years either way of the age your mother
was at menopause,‖ says Nanette Santoro, MD, director of the
division of reproductive endocrinology and infertility at the
University of Colorado-Denver School of Medicine and a
member of the board of directors of the American Society for
Reproductive Medicine.
o This isn‘t always true, of course. Some women reach
menopause at an unusually early age -- before 45 or so -- with
no known cause, which could be the result of an inherited
issue or a one-time genetic mutation. ―These can be random
events, but can also be passed on," says Howard Zacur, MD,
PhD, who directs the reproductive endocrinology and infertility
division at Johns Hopkins School of Medicine.
o So if your mother reached menopause at 40, but her sisters
and your grandmother were all around the average age of
50, it‘s unclear whether you‘ll follow her path or theirs.
o But if most of the women in your family, your mother included,
reach menopause early, late, or somewhere in the middle,
you can eye your calendar with some degree of confidence.

- Menopause Age: 4 More Influences


o Your mother's age at menopause is a key factor, but not the
only one. Here are four others to consider:
- Smoking . No other lifestyle factor does more damage to
your ovaries than smoking. So if you smoke and your mother
didn‘t, you‘ll probably reach menopause earlier than she
did. If she smoked and you don‘t, you probably reach
menopause later than she did.
- Chemotherapy . Most forms of chemotherapy used in
younger women are at least mildly toxic to the ovaries.
Many women go through temporary menopause while
undergoing chemotherapy; if cycles do return (they don‘t
always), you can still expect to reach regular menopause a
couple of years earlier than you otherwise would have.
- Ovarian surgery. ―The more you operate on the ovaries, the
more healthy tissue gets damaged,‖ says Marcelle Cedars,
MD, director of the division of reproductive endocrinology
at the University of California, San Francisco, School of
Medicine. So if you‘ve had diagnostic surgery for
endometriosis, for example, Cedars recommends using
medical options (such as hormonal suppression) to treat the
condition in order to avoid repetitive surgeries.
- Ethnicity. ―Certain ethnic groups may have menopause at
slightly different ages,‖ says Santoro. ―Hispanic and African-
American women reach menopause a little earlier, and
Chinese and Japanese women a little later, than the
average Caucasian woman, who reaches menopause at
about age 51.5.‖ Those are averages; every woman is
different.

- Not a Factor:
o Here are three things you might think would influence
menopause age, but don‘t:
 Age at first period. Although the average age of menarche
(onset of first menstrual period) has been getting younger in
U.S. women, there hasn‘t been a corresponding shift in the
average age at menopause. The average age at
menarche is now about 12.4 years old, down from 13.3 in
women born prior to the 1920s, but the average age at
menopause has been around 51.5 for decades. ―You
would assume that a woman only has so many cycles in her
life and if she menstruates later, she‘ll reach menopause
later, but that doesn‘t seem to be true,‖ Cedars says.
 Pregnancy and breastfeeding. These have no impact on
menopause age.
 Use of hormonal birth control methods. ―Even if you‘re using
a birth control method that stops ovulation, it doesn‘t stop
the loss of follicles, the constant process of the ovary taking
them from the resting pool of eggs,‖ Cedars says. ―All the
follicles available in the cohort that month die away, even if
you‘re not ovulating, so birth control doesn‘t appear to
delay menopause."
19. One of the following is TRUE about Assisted Reproductive Techniques
ART:

a. Clomiphine citrate has an adverse effect on cervical mucous


b. Multiple HCG shots are given to induce ovulation

- Clomid side effects:


o Clomid treatment has some potential for adverse effects. Side
effects are definite "cons" of clomiphene use. Adverse effects
are seen in some, but not all women using the drug:
 Mood swings, psychological / emotional side effects
 Hot flashes
 Abdominal discomfort
 Visual disturbances
 Ovarian cyst formation
 Nausea
 Thinning of the uterine endometrial lining
 Reduced production of cervical mucous - this can lower fertility
(bypassed by insemination)

20. One of the following is NOT true about IVF:

a. The optimal number of ova transfered into the uterus is 3.4 .

- Two to three days after the eggs are fertilised, the best quality
embryos are selected to be transferred to your womb.
o If you are under the age of 40, one or two embryos can be
replaced.
o If you are 40 or over, a maximum of three embryos can be
used (unless you are using donated eggs, when the maximum
is two because these eggs will be from donors who are not
older than 35).
o If you have good quality embryos, those that are not
transferred can be frozen. Some clinics may also offer
blastocyst transfer, where embryos are transferred five to six
days after fertilisation.
21. In sperm preparation, capacitation is:

a. When the sperm acquires acrosomal enzymes


b. When the sperm is able to penetrate the zonna pellucida
c. When the sperm is able to fertilize the ovum

Capacitation involves the destabilisation of the acrosomal sperm head


membrane allowing greater binding between sperm and oocyte. This
change is facilitated by the removal of sterols (e.g. cholesterol) and non-
covalently bound epididymal/seminal glycoproteins. The result is a more
fluid membrane with an increased permeability to Ca2+.

22. One of the following is a normal value in semen analysis:

a. A progressive motility of 25%


b. A count of 35 million/ml
c. A normal morghology of 20%

Normal values of the analysis:


• Count > 15 million
• Motility; sperms are classified due to their motility into 4 grades (a,b,c
and d):
A: rapid progressing sperms
B: slow progressing sperms
C: immotile sperms
D: dead sperms
For normal semen motility a and b must be > 32%
• Liquefaction time; should be within 20 minutes
• Volume; should be > 1.5 ml
• Morphology; normal morphology > 4 %
• Wiki; Regarding sperm morphology, the WHO criteria as
described in 2010 state that a sample is normal (samples from
men whose partners had a pregnancy in the last 12 months) if
4% (or 5th centile) or more of the observed sperm have normal
morphology.
• Previously it was >50%
• So, the answer c is also correct.
23. One of the following is NOT correct about ovarian hyperstimulation:

a. Drainage of the enlarged ovaries is one modality of treatment


b. It's worse in a conception cycle
c. Low albumin level can be found

OHSS:
- Is a systemic disease resulting from vasoactive products (Vascular
endothelial growth factor VEGF) released by hyperstimulated
ovaries.
- The pathophysiology of OHSS is characterised by increased
capillary permeability, leading to leakage of fluid from the
vascular compartment, with third space fluid accumulation and
intravascular dehydration.
- Severe manifestations include a tendency to develop thrombosis,
renal and liver dysfunction and acute respiratory distress syndrome
(ARDS), causing serious morbidity.

• Patients with mild and moderate OHSS :


 Treated at home with bedrest and strict control of fluid intake
 Analgesia using paracetamol (NSAIDs should not be used)
 Continue luteal support with progesterone ( not hCG)
• Patients with severe and critical OHSS :
 Hospitalized , confined to bed , strict control of fluid intake,
Intravenous colloid replacement until hemodilution is achieved .
 Ultrasound guided paracentesis if significant discomfort or
respiratory difficulty due to severe abdominal distension, or if
oliguria persists despite adequate volume replacement.
 Heparin is recommended to avoid thrombosis .

24. One of the following is true concerning a patient with Sheehan


syndrome:

a. HMG is used for ovulation induction

The treatment of Sheehan's syndrome lies in the replacement of deficient


hormones. This woman was given pretreatment with cyclic oestrogen and
progesterone to improve the uterine size. Other authors, too, have
reported better ovulation rates in women with hypopituitarism when
pretreated with oestrogen and progesterone.5 Our patient was put on
oestrogen therapy along with HMG injections. Other authors have
witnessed successful outcome after cotreatment in the seventh cycle with
conjugated equine oestrogen in a woman of craniopharyngoma who
had taken six cycles of follicular development with HMG therapy alone
25. One of the following is NOT correct about gynecological surgeries:

a. LFT is routinely done before surgery

 Laboratory investigations should be ordered only when indicated by


the patient‘s medical status, drug therapy, or the nature of the
procedure and not on a routine basis.
 A clinician should consider the risk-benefit ratio of any ordered lab
test.
 CBC: Hb, WBC‘s, platelets, blood group (should be done in all pt)
 Blood glucose: DM, vascular disease
 Urinalysis: DM, UTI
 LFT: liver disease, alcoholism, hepatitis
 KFT: major surgery, hypotension is expected, nephrotoxic drugs will
be used, age above 50
 Coagulation screen: Hx of coagulation disorder, alcohol abuse, liver
dx, anticoagulants
 CXR: smokers, >40, provide a valuable preoperative baseline, Hx of
CVS disease, pulmonary disease, thyroid enlargement, lung tumor
26. One of the following is NOT a side effect of Paclitaxil:

a. Diarrhea
b. Arthralgia
c. Neuropathy

Side effects of Paclitaxil:

Common side effects include nausea and vomiting, loss of appetite,


change in taste, thinned or brittle hair, pain in the joints of the arms or legs
lasting two to three days, changes in the color of the nails, and tingling in
the hands or toes. More serious side effects such as unusual bruising or
bleeding, pain/redness/swelling at the injection site, Hand-foot syndrome,
change in normal bowel habits for more than two days, fever, chills,
cough, sore throat, difficulty swallowing, dizziness, shortness of breath,
severe exhaustion, skin rash, facial flushing, female infertility by ovarian
damage and chest pain can also occur.

A number of these side effects are associated with the excipient used,
Cremophor EL, a polyoxyethylated castor oil. Allergies to drugs such as
cyclosporine, teniposide and drugs containing polyoxyethylated castor oil
may indicate increased risk of adverse reactions to paclitaxel.
Dexamethasone is given prior to beginning paclitaxel treatment to
mitigate some of the side effects.[citation needed] Leuprolide, a GnRH
analog may prevent ovarian damage, according to mice studies.
27. One of the following is NOT a cause of delayed puberty:

a. Cold climate
b. Hypothyroidism
c. Prolactinoma

Possible causes of delayed puberty:


 Variation of normal (constitutional delay)
 In females, prolonged high level of physical exertion, e.g. from being
an athlete
 Systemic disease, e.g. Inflammatory bowel disease, chronic renal
failure
 Undernutrition e.g. anorexia nervosa, zinc deficiency
 Hypothalamic defects and diseases e.g. Prader-Willi syndrome,
Kallmann syndrome
 Pituitary defects and diseases e.g. hypopituitarism
 Gonadal defects and diseases e.g. Turner syndrome, Klinefelter
syndrome, Testicular failure due to mumps orchitis, Coxsackievirus B,
irradiation, chemotherapy, or trauma. Testicular failure is treated
with testosterone replacement, Ovarian failure.
 Absence or unresponsiveness of target organs e.g. androgen
insensitivity syndrome, Müllerian agenesis
 Other hormone deficiencies and imbalances, Endocrine disorders.
e.g. hypothyroidism, Cushing's syndrome
 Cystic fibrosis
 Mutations in FSHB
 Frasier syndrome
 Various forms of congenital adrenal hyperplasia.
 Gonadotropin deficiency, resulting from a number of congenital
and acquired abnormalities of the central nervous system
 Biedl-Bardet syndrome
 Brain tumors e.g. craniopharyngioma, prolactinoma, germinoma,
glioma; diseases of hypothalamus, irradiation and trauma.
28. One of the following is NOT true about ovarian cancer:

a. Patients with early stage are discovered at younger age than those with
advanced stage .

- Presentation of ovarian ca usually at advanced stage because most of


them are asymptomatic.
29. A 21 year old patient underwent a right ovarian cystectomy. The
specimen was found to contain teeth, cartilage, intestinal glands, and
hair. The most likely dx is:

a. Thyroid cancer
b. Benign cystic teratoma

Germ cell tumors:


De-differentiate into the primordial germ cells (totipotential germ cell),
differentiate towards:

1- Embryo-like neoplasms :
- teratomas : composed of multiple cell types derived from one or
more of the 3 germ layers ; ectoderm, endoderm , mesoderm.
Cells differentiate along various germ lines, essentially
recapitulating any tissue of the body. Examples include hair,
teeth, fat, skin, muscle, and endocrine tissue .
- dysgerminomas : is composed of primitive, undifferentiated
germ cells .
2- Extraembryonic fetal-derived (placenta-like) cell populations
3- Mixture of both

30. One of the following is true about genital prolapse:

a. Pelvic floor facsia supports the vagina

Pelvic support:

1) Uterus:
Uterus that placed in pelvis, imagine it as suspended in air, it is
supported in its place by three groups of ligament, each group
considered as a pair of ligaments to prevent prolapse:
a) The transverse cervical ligament.
b) The uterosacral ligament.
c) The pubocervical ligament.

2) Vagina: it is supported by fascia and muscles ‗‘levator ani‘‘ and


perineal muscles .
31. One of the following is NOT done initially in the investigations of urine
incontinence:

a. Post void US
b. Urodynamic studies
c. Clinical assesment

Investigations:
• Urine analysis
• Postvoid residual volume (PVR) — In general, a PVR of < 50 mL is
considered adequate emptying, and a PVR > 200 mL is considered
inadequate and suggestive of either detrusor weakness or bladder
outlet obstruction
• Urodynamic testing
• Uroflowmetry
• Videocystourethrography
• Intravenous urography: If suspected ureterovaginal fistula.
• Ultrasound : For Post-micturition urine residual estimation , Urethral
cysts and diverticula.
• Magnetic resonance imaging : show detailed anatomy .
• Cystourethroscopy

32. One of the following is NOT true about β.HCG:

a. Used for Dx and follow up of benign and malignant molar pregnancy


b. Used as a tumor marker in non.gestational choriocarcinoma
c. it's a good marker for placental site trophoblastic tumor

Placental Site Trophoblastic Tumor PSTT:


• PSTT is a very rare and unique form of GTN, represent a neoplastic
transformation of intermediate trophoblastic cells.
• PSTT can occur after a normal pregnancy, abortion, term delivery,
ectopic pregnancy or molar pregnancy
• Characterized by low B-hCG levels, expression of HPL is increased on
histologic section and as well as in the serum
• Diagnosis is confirmed by dilatation and curettage and
hysterectomy
• Most cases are confined to the uterus but mets has been reported
• Surgery is the primary treatment of choice
• Good prognosis is anticipated in cases localized to the uterus, with
distant mets or delayed treatment the outcome is dismal.

33. A patient who's 6 weeks amenorrhea presented with abdominal pain


and vaginal bleeding. Her β.HCG level was 3000U/L. Her TVUS shows no
gestational sac but no other abnormality. The best next step is:

a. Repeat β.HCG in 48 hours


b. Give systemic methotroxate
c. Laparatomy (there was no laparascopy choice)

+GS IUP, confirm viability

>2,000 IU/L TVUS

-GS Manage as an EP

+GS IUP, confirm viability

B-hCG 1,500-2,000 IU/L TVUS


Inconclusive, repeat
-GS
US when >2,000

If 1,500 – 2,000 IU/L TVUS


Follow up every 2-3
<1,500 IU/L
days
Physician
Increasing slowly consultation
referral

- It is a case of suspected ectopic pregnancy. So, laparoscopy should be


done to visualize and locate the location (or laparotomy if inadequate
visualization or exposure with laparoscope)
34. One of the following is true about ectopic pregnancy:

a. Accounts for 10% of maternal mortality


b. β.HCG and US findings going with ectopic are diagnostic

Prognosis of Ectopic Pregnancy:

• Mortality (7-10%) because of Wrong diagnosis, Undiagnosed or


Resuscitation attempts
• Shock (10%) at time of diagnosis
• 60% of ectopic have >500ml free blood in abdomen at time of
laparatomy.

35. 60 years old patient presented with postmenopausal bleeding with


normal pelvic examination. The next step is:

a. TVUS

- Suspected case of endometrial ca.

Diagnosis:

• Cervical smear
• Trans-vaginal US (endometrial thickness <4mm in postmenopausal,
but if >4mm take a biopsy to R/O Ca).
• Endometrial biopsy (either in the clinic, by hysteroscopy or D&C)
• If biopsy confirmed Ca, do staging
• CBC,KFT,LFT,CXR & pelvic MRI.

36) One of the following is true about endometrial cancer:


a. +Clear cell is worse prognosis than endometrioid.

 Both are closely associated with endometriosis


 Borderline endometrioid tumors are rare & have good
prognosis
 Clear cell (mesonephroid) tumors have been suggested to
have poorer prognosis than other ovarian carcinomas,
including high-grade serous carcinomas
 Germ cell tumors are chemosensitive

37) One of the following is NOT a cause of menorrhagia:


a. +Ectropion

 Cervical ectropion (eversion): endocervical columnar


epithelium protrudes out of external os & undergoes
squamous metaplasia; transforms into stratified squamous
epithelium
 It causes post-coital bleeding & excessive non-purulent
vaginal discharge
 Causes of
menorrhagia: 6. Pregnancy complications
1. Hormone 7. Cancer
imbalance 8. Inherited bleeding disorders
2. Ovaries 9. Anti-inflammatory drugs,
dysfunction anticoagulants
3. Uterine fibroids, 10. PID, thyroid, liver & kidney
polyps problems
4. Adenomyosis,
endometriosis
5. IUD

38) One of the following is the most likely to have endometriosis:


a. +A 30 years old female with secondary infertility for 6 years
and dyspareunia
b. A 15 years old with lower abdominal pain in the first 2 days of
period fading with menses

 Risk factors:  Symptoms:


1. 25-40 of age 1. Dysmenorrhea
2. Nulliparous 2. Dyspareunia
3. Positive family history 3. Infertility
4. Early menarche, 4. Menorrhagia,
irregular menses menometrorrhagia, inter-
menses
5. Discomfort with bowel
movement & urination

39) A 17 years old complaining of dysmenorrhea was found to


have pelvic endometriosis. The initial treatment:
a. +NSAIDs
b. GnRH for 9 months

 Medical:  Surgical:
1. First-line: NSAIDs, low 1. Definitive: TAH-BSO
dose oral 2. Fertility preserving:
contraceptives (OCs), Laparoscopic or
progestins for 3-6 laparotomy (Note: >3cm
months diameter should be
2. Second-line: Higher removed surgically)
doses progestins, GnRH
analogues, danazol

40) One of the following is NOT true about DUB:


a. +Mefenamic acid increases PG

 Mefenamic acid, ibuprofen & naproxen sodium (NSAIDs)


decrease PG production by enzyme inhibition. They are the
treatment for dysmenorrhea not for DUB
 DUB treatment:
1. Conjugated estrogens
2. Estrogen-progestin combination
3. Cyclic progestin (MPA: medroxyprogesterone acetate)
4. Mirena with Levonorgestrel-releasing IUD
5. Endometrial ablation
6. Vaginal hysterectomy

41) A 40 year-old patient with heavy menses and hypertension.


The best contraceptive method for her is:
a. COCP
b. +Mirena

 OCs shouldn‘t be used for women in their 40s who are


smokers, because of high estrogen concentration & its
association with thrombophlebitis, MI. It has hypertensive
side effect.

42) A patient was referred because of unfound IUCD thread. The


next step:
a. +Pelvic US
b. Abdominal X-ray

 The IUCD probably entered the uterus which located in


pelvic region & US is readily available

43) IUCD is contraindicated in all of the following except:


a. +Previous CS
b. Previous PID
c. Previous ectopic (may suggest uterine abnormalities)
d. Uterine abnormalities

 CI due to unacceptable  CI due to risks outweigh


risk: advantages:
1. Pregnant lady 1. Postpartum 48hrs-4weeks
2. Malignant GTN 2. Benign GTN
3. Cervical, uterine cancer 3. Ovarian cancer
4. Septic abortion 4. AIDs
5. STIs 5. Likelihood of exposure to
6. PID in past 3 months STIs
7. Pelvic tuberculosis
8. Unexplained abnormal
vaginal bleeding
44) One of the following doesn’t cause hirsutism:
a. +Cyproterone acetate

 Cyproterone acetate, spironolactone & finasteride are


androgen blocker used for hirsutism.
 Causes of
hirsutism: 7. Adrenal gland cancer, Von Hippel-
1. PCOS Lindau disease
2. CAH 8. Insulin resistance
3. Cushing‘s 9. Stromal hyperthecosis in
4. Excess GH menopause
5. Ovarian tumors 10. Obesity
6. Porphyria 11. Drugs: danazol, minoxidil,
cutanea tarda phenytoin, cyclosporine,
glucocorticoids, anabolic steroids

45) One of the following is correct about PCOS:


a. +Ovarian morphology found in 20% of females

 6-10% of women of reproductive age have some form of


PCOS
 Clinically, the most common signs of PCOS are hirsutism
(90%), menstrual irregularity (90%), and infertility (75%)
 About 20% of hormonally normal women may also have
polycystic-appearing ovaries
 In about 60% to 70% of patients with PCOS, insulin sensitivity
is decreased, leading to insulin hypersecretion
 60% associated with obesity

46) A 30 years old female presented with menorrhagia was


found to have a 10*12 cm fibroid. You would counsel her
that:
a. Mirena is best for her
b. Uterine artery embolization is optimal if she wants to preserve
fertility
c. +Myomectomy is advisable

 Fibroids are only treated when it is >12 weeks gestational


size (+-12cm fundal height) or implicated as a cause of
infertility
 Treat the menorrhagia medically
 For preservation of fertility, myomectomy is an option if the
number & size of fibroid is limited
 For preservation of uterine, endometrium ablation to
manage menorrhagia
 For preservation of both, uterine artery embolization (UAE)
only when few small to moderate sized. After UAE
pregnancy is still possible but is higher risk
 Hysterectomy is definitive therapy

47) One of the following is NOT true about uterine artery


embolization:
a. +Submucosal fibroid is not a contraindication
b. Premature ovarian failure is a complication
c. PID is a contraindication

 Contraindications:  Side effects:


1. Current or recent 1. Embolism
infections in the genital 2. Septicemia
tracts 3. Endometritis
2. Serious diagnosis doubt 4. POF, infertility, loss of
3. Asymptomatic fibroids orgasm
4. Pregnancy or planning 5. Hysterectomy
5. Patient refuse 6. Post-embolisation
6. Current use of GnRH Syndrome (PES):
agonist acute/chronic pain, fever,
7. Submucosal fibroids malaise, N&V, night sweats
8. Extensive adenomyosis 7. Hematoma
9. Previous internal iliac 8. Vaginal discharge &
artery ligation bleeding

48) One of the following is NOT a presentation of fibroid:


a. Dyspareunia and IMB

 Fibroid presentations:
1. Mostly asymptomatic
2. Pelvic pressure, congestion, bloating
3. Heaviness or mass in the lower abdomen, lower back
pain
4. Menorrhagia
5. Severe pain associated with red degeneration (acute
infarction)
6. Intermenstrual bleeding is not characteristic but may
occasionally occur with submucous myomas ulcerating
through the endometrial lining
7. Dyspareunia is also common if the uterus is incarcerated
within the pelvis
8. Secondary dysmenorrhea
9. Submucosal leiomyomas may be associated with an
increased incidence of infertility

49) A patient with severe acute PID. One of the following is


appropriate in her management:
a. Laparoscopy after 48 hours of appropriate IV antibiotics with
no improvement

 Laparoscopy is no longer considered clinically necessary


for diagnosis of PID
 Patients should be reevaluated by pelvic examination 3 to
4 weeks after treatment
 Failed oral antibiotics require hospitalization to administer IV
 Antibiotics are to complete for 14 days
 Treat partners for the past 60 days
 Inpatient (IV-14 days):  Outpatient (IM-14 days):
1. Regimen A: 1. A: Ceftriaxone +
Cefoxitin/cefotetan + doxycycline +/-
doxycycline metronidazole
2. Regimen B: 2. B: Cefoxitin + doxycycline
Clindamycin + +/- metronidazole
gentamycin

50) One of the following is NOT correct about STDs:


a. +Cervical warts (condylomata acuminata) mandate a
cervical smear
b. Primary syphilis presents as painless ulcer (chancre)
c. Serology is useful in Herpes diagnosis
 Laboratory tests for Herpes: viral culture, PCR & serologic
test
 Biopsies of atypical or persistent lesions (warts) are needed
to rule out neoplastic disease

51) One of the following is NOT correct about candidal


vulvovaginitis:
a. +pH >6
b. Intense itching
c. Common in pregnancy

 Risk factors: high-dose OCs, diaphragm use with


spermicide, DM, antibiotic, pregnancy, immunosuppression
& tight occlusive clothing
 Presentation: vaginal itching, burning, irritation &
postvoiding dysuria
 The discharge is odorless, pH <4.7 & thick/curdy with the
appearance of cottage cheese
 Examination: vulvovaginal erythema, with acute/chronic
excoriation
 Budding yeast cells, pseudohyphae or mycelial tangles in
50-70% microscopically

52) One of the following is NOT correct about ectropion:


a. +Mainly present with IMB

 Cervical ectropion present with increased vaginal


secretions and postcoital spotting
 Nabothian cysts (mucus retention cyst) are common with
ectropion due to trapped columnar epithelium by
squamous epithelium
 The columnar cells produce mucus and are more
vulnerable to trauma and infection with chlamydia

53) HPV DNA testing is indicated in:


a. +ASCUS
b. Atypical squamous cell cannot exclude high grade lesion
(HSIL)(ASC-H)
c. SCC
 Abnormal pap smears: ASCUS, LSIL (CIN I, II), HSIL (CIN II, III,
CIS), ASC-H, AGC, AIS, SCC
 Management for ASCUS: repeat pap smear in 6m, or; HPV
test, or; immediate referral to colposcopy & biopsy
 Management for ASC-H, LSIL, HSIL, AGC, AIS, SCC & other
high grade lesions: immediate referral to colposcopy &
biopsy

54) A patient whose pap smear showed high grade squamous


intraepithelial lesion. Next step is:
a. +Colposcopy
b. Repeat pap smear in 6 months
c. HPV DNA testing

55) One of the following is NOT correct about cervical cancer:


a. +Chemotherapy can replace surgery in early stages

 Stage Ia1: abdominal/vaginal hysterectomy is appropriate


 Stage Ia2: radical hysterectomy + pelvic
lymphadenectomy
 Stage Ib1: primary surgery is treatment of choice (radical
hysterectomy + bilateral pelvic lymphadenectomy)
 Stage Ib2: primary chemoradiation theraphy often chosen
 Stage IIa: radical surgery or chemoradiation
 Stage IIb: combination of external-beam chemoradiation &
intracavitary brachytherapy
 Stage IIIa & IIIb: chemoradiation
 Stage IVa: chemoradiation +/- ―salvage‖ pelvic
exenteration
 Stage IVb: palliative chemoradiation

56) One of the following is NOT correct about labor:


a. +Prolonged labor due to hypotonic uterine contraction is
usually associated with fetal distress

 Fetal distress is usually associated with obstructed labor


 Prolonged labor can lead to maternal exhaustion &
dehydration
 Obstructed labor is the arrest of fetus vaginal delivery due
to mechanical obstruction despite of strong uterine
contraction
 Prolonged labor is the prolongation of the 1st stage of labor
due to inefficient uterine contraction

57) One of the following is correct about 3rd stage of labor:


a. +Cord traction with no uterine contraction increase the risk
of uterine inversion

 3rd stage of labor is the delivery of placenta


 Signs of placental separation: (1) a fresh show of blood
from the vagina (2) the umbilical cord lengthens outside
the vagina (3) the fundus of the uterus rises up (4) the uterus
becomes firm and globular
 Traction on the cord is only done after these signs have
appeared
 Uterine contraction reduces bleeding & risk of uterine
inversion
 The contraction may be hastened by uterine massage or
oxytocin infusion
 It is routine to add 20 U of oxytocin to the intravenous
infusion after the baby has been delivered

58) One of the following is correct about cephalohematoma:


a. Ossification can occur and may lead to asymmetrical skull

 It is a hemorrhage of blood between the skull and the


periosteum
 Usually due to prolonged second stage of labor or
instrumental delivery
 If severe the child may develop jaundice, anemia or
hypotension
 Risk of an infection leading to osteomyelitis or meningitis
 The swelling takes weeks to calcify & resolves
spontaneously
 Ossification of the sutures that leads to asymmetrical skull is
craniosynostosis, which caused by maternal smoking,
valproic acid, genetic & biomechanical factors

59) One of the following is correct about vacuum delivery:


a. +Can cause cephalohematoma

 Ventouse (VE) is the commonest cause of


cephalohematoma
 It is contraindicated in preterm delivery, breech & face
presentation (only vertex)
 Flexion of the fetal head must be maintained to provide the
smallest diameter to the maternal pelvis by placing the
posterior edge of the suction cup 3 cm from the anterior
fontanelle over the sagittal suture
 If detachment of the suction cup from the fetal head (pop-
off) occurred twice or still no progress down the birth canal,
CS should be done
 Indications: (same as forceps)
1. Prolonged second stage of labor: 1hr multipara, 2hrs
nullipara (+1 with anesthesia)
2. Suspicion of immediate or impending fetal compromise
3. To stabilize the aftercoming head during a breech
delivery
4. To shorten the second stage of labor for maternal
benefit: maternal HTN, cardiac, pulmonary diseases

60) A primigravida in labor with cord prolapse and


asymptomatic. The fetus presentation is vertex. The best next
step:
a. +Emergent CS
b. Deliver vaginally

 Management of cord prolapse: (1) administer oxygen to


the mother (2) Relieve cord compression (3) Immediate
delivery
 CS is the recommended mode of delivery
 Vaginal delivery only if: (1) cervix is fully dilated & quick
delivery is desired (2) fetal demise (3) fetal anomalies
incompatible with life
61) One of the following is NOT a complication of induction of
labor:
a. +Precipitate labor
b. Fetal distress
c. Uterine rupture

 Complications:
1. Excessive oxytocin infusion rate: hyperstimulation thereby
causing fetal distress from ischemia & also uterine rupture
2. Intrinsic antidiuretic effect of oxytocin: increase water
reabsorption will cause severe water intoxication with
convulsions and coma can occur
3. Prolonged infusion of oxytocin: can result in uterine
muscle fatigue (nonresponsiveness) & postdelivery
uterine atony (hypotonus), which can increase the risk for
PPH

62) One of the following is NOT a prerequisite for instrumental


delivery:
a. +Epidural analgesia
b. Full cervical dilatation

 Appropriate consent from the patient about potential risks


& benefits should be obtained
 The cervix must be fully dilated, membranes ruptured, and
the fetal head engaged into the pelvis
 Clinical assessment to determine the level of the presenting
part, estimation of the fetal size, and adequacy of the
maternal pelvis is mandatory
 Anesthesia must be adequate by either pudendal nerve
block with local infiltration (for outlet forceps only) or
regional anesthesia
 The bladder should be emptied
63) One of the following is NOT correct about regional
anesthesia:
a. CI in epilepsy
b. Can lead to postural headache
c. Can cause severe fetal bradycardia
d. +All are true
 Local Anesthesia includes: (1) local infiltration (2) nerve
block (3) regional anesthesia
 Regional: (i) Spinal – into CSF (ii) Epidural – into the space
between spinal membranes
 Immediate maternal  Delayed maternal
complications: complications:
1. Hypotension & may cause 1. Postural puncture
fetal bradycardia (distress) headache
2. Complete motor & sensory 2. Backache
paralysis 3. Urinary retention
3. Convulsions 4. Tingling in hands/fingers
4. Cardiac arrest 5. Epidural abscess (rare)

64) The most consistent sign in severe abruptio placenta is:


a. +Oliguria
b. Ecchymosis
c. Fetal distress

 Hypovolemic shock & acute renal failure due to massive


hemorrhage may be seen with a severe abruption if
hypovolemia is left uncorrected
 Placental abruption is the most common cause of DIC in
pregnancy
 Abruption places the fetus at significant risk for hypoxia
and, ultimately, death
 Sheehan‘s syndrome may be a delayed complication

65) One of the following is NOT a late complication of CS:


a. +Wound dehiscence

 Early post-op  Late post-op complications:


complications: 1. Uterine dehiscence and/or
1. Endomyometritis (most rupture
common) 2. Repeat CS
2. Hemorrhage 3. Placenta Accreta
3. Wound infection 4. Cesarean hysterectomy
4. UTI
5. GI complication
6. Thromboembolic
complication
7. Septic thrombophlebitis
8. Wound dehiscence

66) A 34 weeks pregnant patient presented to the ER with history


of heavy vaginal bleeding. She's hemodynamically stable
and dx to have placenta previa. The best next step is:
a. Resuscitate and deliver by CS
b. Induction of labor
c. +Give steroid and deliver in 48 hours
d. Blood transfusion and conservative management till maturity
is achieved

 With a preterm pregnancy (<37w), the goal is attempt to


obtain fetal maturation without compromising the mother‘s
health
 If bleeding is excessive, delivery must be accomplished by
cesarean birth regardless of gestational age
 Use steroids for lung maturation: 2 doses of betamethasone
IM 24 hours apart (48hrs), or 4 doses of dexamethasone IM
12 hours apart (48hrs).

67) A 32 weeks pregnant patient whose dx to have PP presented


with mild bleeding. The best next step:
a. +Establish an IV line & admit her

1. Proper assessment of maternal condition & resuscitation


(IV line)
2. In severe bleeding, emergency cesarean delivery
irrespective of gestational age
3. If bleeding after 36-37 weeks deliver
4. If bleeding not severe and early pregnancy, expectant
management, attempting to reach fetal maturity (36-38
weeks) without risking maternal health

68) One of the following is a sign of severe PET:


a. +Hyperreflexia

1. Severe hypertension (sys: ≥160 mmHg, or dia: ≥ 110 mm


Hg) at rest on 2 occasions 6 hr apart
2. Heavy proteinuria (≥5 g in a 24-hr collection or 3+ in urine
samples collected 4 hr apart)
3. Oliguria (<500 mL in 24 hr)
4. Cerebral or visual disturbances: blurred vision, spots,
scotomata, sudden loss of vision (cortical blindness),
new-onset headache & hyperreflexia are extremely
concerning signs
5. Pulmonary edema or cyanosis
6. Epigastric or right upper quadrant pain
7. Impaired liver function (elevated liver enzymes)
8. Thrombocytopenia
9. Fetal growth restriction

69) A 28 weeks pregnant patient whose BP is 140/95 and


excretes 3g/24 hours of protein in urine. She's asymptomatic.
This is:
a. +Mild PET

 Mild PET: (1) GA >20weeks (2) sustained BP ≥140/90 (3)


Proteinuria 300mg/24hrs or 1-2+
 Asymptomatic

70) A pregnant patient who was diagnosed to have PET


presented with seizures. What NOT to do:
a. Put her in left lateral position
b. Supplemental oxygen
c. +Biophysical profile
d. MgSO4

1. Call for help


2. Avoid aspiration by positioning the mother on lateral
decubitus
3. Administer oxygen to prevent hypoxia
4. Administer MgSO4
5. Manage the BP with hydralazine, nifedipine, labetolol or
IV sodium nitroprusside
6. Fluid balance management
7. A careful fetal evaluation
8. Pregnancy termination if appropriate

71) A primigravida in labor and she's at term. Her ANC was


uneventful. The most appropriate for her fetal monitoring is:
a. +Fetal heart auscultation every 15-30 minutes
b. Continuous CTG

 Auscultation of the fetal heart is performed every 15 mins


after a uterine contraction during the first stage of labor
 At least every 5 mins in the second stage of labor

End of Zaki Fikri’s part: Questions 36-71. I’m happy for any correction. All the
best wishes to you & good luck.
Sources: Hacker & Moore’s, Gynecologic Oncology, lecture & seminar’s slides,
Wikipedia, UpToDate & Mayoclinic.

72. One of the following is NOT in biophysical profile:

a. CTG

b. Fetal weight

c. Fetal gross movement

d. Fetal breathing .

From Hacker and Moore(pg. 220):

Biophysical profile:

1. Fetal breathing: 30 secs of rhythmic movement of fetal thorax


2. Fetal movement: at least 3 movement of fetal body or limb
3. Fetal tone: one extension or flexion of limb joint
4. Amniotic fluid: single deepest vertical pocket of amniotic fluid >2 cm
5. In conjunction with a 30 minutes Nonstress test
73. A pregnant patient who's dx to have PET presents at term with blurred
vision and BP of 150/100. The best next step is:

a. IV hydralazine and watch for symptoms (

b. Give steroids and deliver in 48 hours

c. Arrange for delivery

Management of Pre-eclampsia

Hacker and Moore (pg. 177)

3 things to look for:

1. Mild or Severe? Criteria of severe preeclampsia box 14-2 (pg. 174)


# Hypertension (systolic >160 / diastolic > 110 mmHg at 2 occasions
at least 6 hours apart)

# Heavy proteinuria (at least 5g in 24 hours collection or qualitative


value of 3+ in urine samples collected 4 hours apart)

# Oliguria (<500mL in 25Hr)

# Cerebral or visual disturbance

# Pulmonary edema or cyanosis

# Epigastic or right upper quadrant pain

# Impaired liver function (elevated >40 AST and ALT)

# Thrombocytopenia
# Fetal growth restriction

2. Evidence of fetal compromise (ie., Growth restriction,


oligohydramnios or fetal heart abnormalities)
3. Maturity of fetus
# A woman with mild preeclampsia, without evidence of fetal
compromise, whose disease doesn‘t appear to be progressing, will
generally not delivered unless the gestational age is 37 weeks or more.

# A woman with severe preeclampsia or eclampsia should usually


delivered after period of stabilization, regardless of GA

# A woman with mild disease and there‘s no evidence of fetal


compromise, management consist of rest and observation

# A woman should be delivered before 38 weeks, if she develop signs and


symptoms of worsening disease / no evidence of fetal compromise

# A woman with initial evaluation is consistent with diagnosis of severe


eclampsia, she should be hospitalized for remainder of pregnancy

# A woman with severe HTN is stabilized by bed rest + medical control +


corticosteroid

# Deterioration in clinical status (eg., uncontrollable HTN, deteriorating


renal function, pulmonary edema, evidence of HELLP or coagulopathy,
CNS symptoms, abruption, or abnomail fetal testing) requires delivery

69

74. A patient with chronic HTN. What not to do in ANC:

a. LFT

b. KFT

c. Urinalysis

d. Hepatits B screen

Chronic hypertension Hacker and Moore (pg. 174)


1 of the following:

# known HTN before pregnancy

# development of hypertension before 20 weeks of pregnancy or in case


of first recognition during pregnancy

# persistence of elevation of elevated blood pressure greater than 12


weeks postpartum

May caused by secondary hypertension from renal, vascular, endocarine


(eg. Hyperthyroidism) or methamphetamine or cocaine usage

75. One of the following is correct about GDM:

a. You can decrease the risk of fetal anomalies to values closer to those of
non.diabetics by strict glycemic control

Diabetes Mellitus during pregnancy Hacker and Moore (pg. 191)

# Screening should be done in the first prenatal visit for woman with risks
factors (>25 years old mother, previous macrosomic infant, previous
unexplained fetal demise, previous pregnancy with GDM, family history of
DM, history of PCOS and obesity.

# Screening is usually done between 24-28 weeks with 50 g 1 Hour OGCT;


without regard to last oral intake; if more than 200 mg/dl, no more OGCT
needed, may dangerously elevate blood glucose values

# Abnormal screening should be followed with a diagnostic 100 g 3 Hour


OGCT, if 2 or more abnormal values, patient is diagnosed with GDM

# If overt sign and symptoms of diabetes are present, a fetal scalp blood
sample test should be undertaken first
# White‘s Classification of diabetes in pregnancy ; class A B C D F R H
(table 16-1, pg. 192)

# Class A B C, fetal macrosomia is common while class D F R, fetal growth


restriction occurs more commonly

# Maternal and Fetal complications of diabetes mellitus (table 16-2 pg.


193)

# In pregestational diabetic patients, detailed obstetric ultrasonic study,


fetal echo, and maternal serum alpha fetoprotein should be obtained at
16-20 weeks to check for congenital malformations.

# HbA1c >8.5% should be particularly targeted for careful ultrasonic


assessment for congenital anomalies.

# When to intervene? Unstable maternal state, blood glucose not in


euglycemic range, fetal studies doesn‘t show healthy baby.

# C-sec may be elected for fetus >4250 or >4500g with maternal diabetes
or >5000g for non diabetic mothers. (ACOG); due to increased risks of
shoulder dystocia, labor dystocia, and genital trauma.

1. Management to achieve euglycemia include diet, therapy ( Insulin ;


may include both lipro as short acting and regular / NPH insulin as
intermediate-acting insulin, certain OHA such as glyburide can be
used after first trimester)
2. Plasma glucose should be maintained between 80 to 120 mg/dL +
Fetal heart monitoring
3. Postpartum exogenous insulin management may not needed up to
first 48-72 Hours because of removal of placenta that contain insulin
antagonist + postpartum fasting blood glucose or 75 g OGCT
performed at 6-12 weeks postpartum

76. One of the following is INCORRECT about GDM:

a. Retinopathy is the 1st to improve with insulin therapy


77. α.fetoprotien is elevated in all of the following except:

a. NTD

Hacker and Moore (pg. 81)

# NTD is a multifactorial disorder, 1/1000 birth

# A woman who have had an infant with a neural tube defect should
take vitamins plus 4 mg of folic acid daily before conception.

# Neural tube closure occurs at 28-30 days postconception

# Alpha fetoprotein is measured at 16-20 weeks gestation incombination


of hCG and UE3 estriol

# Amniotic fluid alpha fetoprotein and AChE are frequently elevated in


blood samples of women carrying fetuses affected with neural tube
defects, we can also test for NTD by MSAFP; maternal serum AFP

# Elevated MSAFP also associated with gastroschisis and omphalocele;


false-positive elevation of MSAFP in multiple gestation, fetal demise, or
inaccurate gestational age

# Down syndrome: triple screen 1) <MSAFP 2) >hCG 3) <UE3 + (inhibin A


for quadruple screen)

# If all is < in triple screen: trisomy 18

b. Down's syndrome

78. One of the following isn't a cause of large for dates:

a. Down's syndrome

b. Anencephaly

c. Cleft palate
79. The most common cause of neonatal mortality is:

a. Prematurity

b. Congenital abnormalities

80. One of the following is NOT true about congenital abnormalities:

a. A major anomaly is present in 30 out of one thousand newborns

b. Some of them may not present at birth

c. The most common cause is genetic

Its unknown

81. One of the following is NOT presented per 1000:

a. MMR (maternal mortality rate)

82. One of the following is correct about IUGR:

a. Associated with oligohydramnios in 80%

IUGR Hacker and Moore (pg.153)


# Occurs when birth weight of a newborn infant is below the 10 th
percentile for given GA

# Prone to have meconium aspiration, asphyxia, polycythemia,


hypoglycemia, and mental retardation + adult onset conditions such as
HTN, DM, and atherosclerosis

#Etilogy:

1. Maternal: Poor nutritional intake, smoking, drug abuse, alchoholism,


cyanotic heart disease and pulmonary insufficiency. Some new
studies indicates antiphospholipids antibodies such as lupus-like
anticoagulant and anticardiolipin antibodies may take part. Others
as hereditary thrombophilia may have been associated with greater
risk of IUGR, abruption and preeclampsia. These conditions result in
vascular lesions in spinal arteries supplying the placenta. Treatment
with low dose heparin and aspirin have shown to reduce risk for
IUGR
2. Placental: inadequate substrate transfer because of placental
insufficiency leading to essential HTN, chronic renal disease and
pregnancy-induced HTN. If pregnancy-induced HTN occurs in late
pregnancy, and is not accompanied by chronic vascular or renal
disease, significant IUGR is unlikely to occur. Some IUGR case may
be attributed to placental or cord abnormalities such as
velamentous cord insertion
3. Fetal: Inadequate or altered substrate is present such as in
intrauterine infection; TORCH and congenital anomalies
#Symmetrical VS Asymmetrical

1. Symmetrical: Both head and body is inadequate, but head-to-


abdominal ratio maybe normal but the absolute growth rate is
decreased (most from fetal etiology;TORCH and congenital
anomalies)
2. Asymmetrical: Brain is spared, larger head, liver and pancreas
undergo the most changes. This may lead to adult onset conditions
such as obesity and diabetes.
#Screening: Serial uterine fundal height measurement, more thorough
sonographic assessment is needed when fundal height lags more than
3cm behind a well-established GA or the mother has high risk condition
such as preexisting HTN, chronic renal disease, advanced diabetes with
vascular involvement, preeclampsia, viral disease, addiction to nicotine,
alcohol, hard drugs, or presence of serum lupus anticoagulant or
antiphospholipid antibodies.

#Risks for IUGR on diagnosis: abnormal triple screen + no abnormalities on


ultrasound and amniocentesis, increase resistance of umbilical artery and
uterine artery Dopler assessment at mid-pregnancy

#U/S parameters: BPD, head circumference, abdominal


circumference(single most effective), head-to-abdominal circumference
ratio, femoral length, femoral length-to-abdominal circumference ratio,
amniotic fluid volume, calculated fetal weight, umbilical and uterine
artery Dopler

#At 34 weeks, head-to-abdominal circumference ratio is 1

83. One of the following is NOT correct about Rh.isoimmunization:

a. 1 ml is needed to induce a maternal response

Rhesus Isoimmunization Hacker and Moore (pg. 183)

# 2 exposures to the Rh antigen are required to produce any significant


sensitization, unless the first one is massive

# Cause profound anemia of fetus, neonatal kernicterus

# In the first trimester, the most common cause of transplacental


hemorrhage are spontaneous or induced abortion.

# In the second and third trimester transplacental hemorrhage after


invasive procedures such as amniocentesis or chorionic villus sampling,
after abdominal trauma, or after external cephalic version.
# All pregnant Rh-negative who are not sensitized to the D antigen should
routinely receive prophylactic Rh immunoglobulin ( RhoGaM) at 28 weeks
GA, within 72 hours of delivery of an Rh-positive fetus or any problem
associated with transplacental hemorrhage.

# Detection by

1. Kleihauer-Betke test

2. maternal Rh-antibody titer

3. amniocentesis with flow cytometry

4. U/S dopler of peak velocity in MCA of fetus;1.5 MoM above median


is considered moderate or severe anemia

5. U/S for hydrops; ascites, pleural effusion, pericardial effusion or skin


edema

6. Amniotic fluid spectrophotometry in starting from 27 weeks with Liley


chart (pg. 187) ; source of bilirubin from tracheal and pulmonary efflux
with some transudate from the umbilical and placental vessels. If ∆OD 450
enters zone 3 before 34 weeks, percutaneous umbilical blood sampling is
performed for determination of fetal hemoglobin followed by intrauterine
transfusion if indicated.

# Other methods of therapy; maternal plasmapheresis and phenobarbital

84. One of the following is NOT a cause of delayed uterine involution:

a. Retained pieces of placenta

b. Non breastfeeding

c. Uterine fibroid

d. Full bladder

*Note: actually none of the answers is the answer soo...Wallah mani


3arfeh!!
Subinvolution

# Mainly caused by grand multiparity, overdistension from multiple


gestation and hydromnios, C-sec, uterine prolapse, retroversion of uterus,
uterine fibroid, full bladder, persistent lochia, anesthesia, difficult delivery

# Others may increase the incidence such as RPOC, uterine


sepsis/endometritis

# Treatment may involve antibiotics, exploration, and ergometrine to


reduce blood flow to the uterine

85. One of the following is NOT a contraindication for breast feeding:

a. Mastitis

86. One of the following is NOT a cause of 2nd trimester miscariage:

a. Placenta previa

Placenta Previa Hacker and Moore (pg. 129)

# Painless vaginal bleeding

# 4 Types

# Detection by transabdominal ultrasound during second trimester

# Goal is to attempt to obtain fetal maturation without compromising the


mother‘s health.

# If bleeding is excessive, delivery must be accomplished by C-sec


regardless of GA
# If the patients is at 36 GA, fetal lung maturity should be determined by
amniocentesis and the patient need delivery by elective C-sec, because
of high risk of hemorrhage and fetal hypovolemia and anemia.

# Complications:

1. Maternal death mainly from C-sec or uncontrolled bleeding from


placental site or even from DIC
2. Placenta accrete
3. Uterine atony
4. Preterm delivery
5. Malpresentation

87. One of the following is INCORRECT about recurrent miscarriage:

a. Steroids are effective in cases of undefined etiology

Recurrent spontaneous abortion Hacker and Moore (pg. 75/76/77)

# 3 successive spontaneous abortions usually occur before a patient is


considered as recurrent aborter and need genetic counselling

# Important to rule out DM, SLE and thyroid disease (hypothyroidism more
often associated with ovulation disorder) . Lupus anticoagulant.

# Hysteroscopy or hysterography should be performed to evaluate uterine


cavity anomalies such as cervical incompetence or congenital anomalies
of fundus as in exposure do diethylstilbestrol or acquired abnormalities or
submucosa fibroids or intrauterine adhesions

# No prospective study has been able to demonstrate unequivocally that


a normal pregnancy can be lost as a result of abnormal hormone
production by either the corpus lutheum or placenta; no trial of
exogenous hormones have been able to show benefits and indeed
exogenous sex steroids be teratogenic
# Its appropriate to rule out the presence of Mycoplasma, listeria,
toxoplasma, treponema, CMV and brucella

88. One of the following is INCORRECT about recurrent miscarriage:

a. IVIG is effective in cases of undefined etiology .

89. One of the following is INCORRECT about preterm labor:

a. Cervical length of 2.5 cm is a risk factor

Preterm Labor Hacker and Moore (pg. 146-150)

# After 20 and before 37 GA

# Prematurity is leading cause of infant mortality

# Black patients have higher proportions of deliveries due to PPROM

# Genetic thrombophilias leading to uteroplacental abnormalities which


is leading to IUGR and preeclampsia; early IOL to avoid fetal death

# More women are postponing child birth as lifestyle choice; infertily and
ART usage cause multiple gestation and early labor

# Socioeconomic factors

# Previous preterm labor and second trimester abortions carry an


increased risk

# Bleeding in the first trimester, UTI, multiple gestation, uterine anomalies,


polyhydromnious and incompetent cervix, maternal physical activities,
nutritional status, stress and anxiety
# Diagnosis: uterine contractions (4 per 20 minutes) and cervical changes;
effacement 80% and dilatation of 2 cm or more

# Labor and delivery has a lower limit of 24 weeks or 500g in the presence
of expertise of NICU, vertex= vaginal, breech at less 1500g= C-sec

90. One of the following is not a significant finding in a pregnant at term:

a. Ankle edema

Hacker and Moore‘s (pg.58)

# Venous compression by the gravid uterus elevates pressure in veins that


drain the legs and pelvic organs, thereby exacerbating varicose veins in
the legs and vulva and causing hemorrhoids

# Hypovolumia and increased venous pressure cause lower limb edema

# Various effect on caval compression are somewhat mitigated by the


development of paravertebral collateral circulation that permits blood
from the lower body to bypass the occluded IVC

# Posiero effect, compression of aorta and its major branch, causing low
pressure on femoral artery

91. One of the following is INCORRECT about Ir defeciency anemia in


pregnancy:

a. IV Ir can cause severe anaphylaxis

b. MCV <85 fL is diagnostic

IDA in pregnancy
92. One of the following is NOT part of your counselling for a pregnant
lady travelling on plane:

a. Decrease fluid intake .

Hacker and Moore Travel (pg.86)

93. One of the following is NOT a part of routine booking visit


investigations:

a. X.ray pelvimetry

Prenatal Care Hacker and Moore(pg.71-90)

94. A preganant patient is found to have asymtomatic bacteruria during


ANC. Management is:

a. Oral A.B.s

95. Which of the following is not suitable for a trial of labor:

a. A patient with previous classical CS

TOLAC/VBC Hacker and Moore (pg.226)

# Common indications for classical C-sec include preterm breech with an


undeveloped lower uterine segment, transverse back-down fetal position,
poor access to the lower segment due to myomas or adhesions, or
planned C-sec hysterectomy.
# Trail of labor maybe offered if one or 2 previous Low transverse
cesarean delivery (LTCD) were performed, the uterine incision did not
extend into the cervix or uterine upper segment, and there is no history of
prior uterine rupture.

96. A patient who's G2P1 presents at 35 weeks gestation with breech


presentation. NO abnormalities detected. She's not in labor. Management
is:

a. Visit in 2 weeks

b. X.ray pelvimetry

Breech Presentation Hacker and Moore (pg. 166-170)

# Before 28 weeks, about 25% of fetuses are in breech position, by 34


weeks, they are assumed to be vertex. Overall incidence is 4%

# Complete, Frank, Footling

# Diagnosis by Leopold maneuvers

# Management:

1. Exclude fetal and uterine anomalies of present after 34 weeks to


exclude myomas, Mullerian anomalies, or fetal structural anomalies
by U/S
2. ECV: considered at term before labor. Not done before 36/37 weeks
because of tendency of premature fetus to concert to a breech
presentation. However, it carry risks of paclental abruption and cord
compression
3. Vaginal delivery: criteria (pg.167 box 13-4)
4. Assisted breech delivery
5. C-sec
97. The presenting diameter in a flexed occipito anterior fetus is:

a. Suboccipitobregmatic

98. The risk of PPH is highest in:

a. G3P2 with a 3.9 kg fetus

b. A primigravida with a twin pregnancy

99. One of the following is INCORRECT about labor:

a. X.ray pelvimetry can accurately predict the need for CS

100. One of the following is TRUE about IUGR:

a. 60% can be detected by serial fundal pubic hight measurement

101. One of the following is NOT correct about bacterial vaginosis:

a. It's treated with pinicillin

102. One of the following is a sex cord cell tumor:

a. Granulosa cell tumor

Sex Cord tumors Hacker and Moore (pg.251)

# Fibromas, granulosa-theca cell tumors, sertoli-leydig cell tumors


103. A 37 weeks gestation pregnant presents to the ER with mild vaginal
bleeding. On US she's dx to have PP and the placenta is reaching the
cervical os. Next step is:

a. Deliver by CS

Placenta Previa Hacker and Moore (pg. 129)

#Marginal posterior and above grade, must do C-sec

104. One of the following is correct about ovarian epithelial tumors:

a. Treated mainly by surgery

Epithelial Ovarian Carcinomas Hacker and Moore (pg.415)

105. One of the following is NOT correct about gynecological surgeries:

a. Vaginal hysterectomy is indicated for large fibroids.

Vaginal Hysterectomy Hacker and Moore (pg.340)

# Is elected for benign disease when uterus is mobile, iss less than 12 GA
weeks in size, is characterized by some pelvic relaxation, and expected to
contain few or no adhesions
# Most commonly performed in association with the correction uterine
prolapse, cystocele, rectocele, enterocele in postmenopausal women

106. A primigravida presents with fully dilated cervix for 60 minutes,


effective uterine cintractions, head station is (.1., and severe caput
succidenum.Next step is:

a. Deliver by CS

b. Wait another 30 minutes

107. One of the following statements about epidural anesthesia is


INCORRECT:

a. Foley's catheter is optional

b. IV infusion of crystalooid prior to induction to insertion of epidural


anesthesia .
5th Year Final Exam 2010 :
1. Confirmative dx of ectopic pregnancy is:
a. laparoscope .
b. hysteroscopy
c. transvaginal exam
d. urinary b.HCG .

2. Common presentation of ectopic pregnancy is:


a. vaginal bleeding
b. lower abdominal pain

3. lab investigation that indicate ovulation:


a. progesterone level mid luteal*
b. LH level
c. FSH level
d. estrogen

4. regarding PCOS, one true:


a. most common cause of premature menopause
b. is present ultrasonagraphically in 20% of population* .
c. associated with increase insulin sensitivity
d. associated with (.. progesterone challenge test
e. no familial tendency .

5. one true about clomiphine citrate:


a. hot flushes
b. hirsutism
c. compete with progesterone at high centers
d. should be administered for 12 moths
e. reduce pituitary gonadtrophin secretion .

6. regarding bromocriptine, all true except:


a. can prevent lactation
b. its ergot alkaloid derivative .

7. most common cause of PPH:


a. uterine atony .
b. infections
c. coagulation defect
d. trauma .
8. all about uterine atony true, except:
a. full bladder is a cause
b. uterine massage cause lasting contraction .

9. polyhydromnios caused by all, except:


a. parvovirus infx*
b. renal agenesis
c. deudonal atresia
d. GDM

10. about active management of 3rd stage of labor, all true except:
a. oxytoxic drugs cause placental retention*
b. oxytocin reduces its duration
c. ergometrine C/I in heart disease
d. oxytocin cause hypotension .

11. all true about genital prolapsed, except:


a.1st degree uterine prolapsed descent of cervix within vagina
b. rectocele is prolapse of 2/3 of post vaginal wall &rectum
c. enterocele is true hernia
d. vault prolapsed commonly occur after vaginal hysterectomy

12. all can cause prolapsed, except:


a. repetitive c.section*
b. prolonged labor
c. instrumental delivery
d. chronic constipation .

13. all true about peuriperuim breast abscess, except:


a. staph is most common
b. diagnosis by mammogram*
c. need antibiotics
d. need surgical incision .

14. all true about bacterial vaginosis, except:


a. caused by lactobacillus*
b. cause of 2nd trimester abortion
c. ph >5

15. all are STD, except:


a. HSV .
b. BV .
c. trichomonas
d. gonorrhea
16. all cause vaginal candiasis, except:
a. DM
b. IUCD*
c. pregnancy
d. vaginal douches

17. all true, except:


a. syphilis primary lesion painless
b. recurrent HSV painless
c. genital warts C/I for vaginal delivery* .

18. all true about induction, except:


a. indicated in 15% of deliveries
b. oxytocin used for unfavorable cervix*
c. most common indication is postdate
d. ARM used for favorable cervix .

19. all C/I for induction, except:


a. antenatal bad CTG
b. transverse lie
c. growth restriction*
d. recent pelvic repair

20. about opioid pain relief during labor, all true, except :
a. cause of neonatal respiratory depression
b. cross placental barrier
c. mostly effective in 1st stage
d. morphine commonly used (pethidine) .

21. about epidural analgesia, all true except:


a. coagulation dis is a C/I
b. used with precaution in IUFD
c. should administer crystalline fluid before insertion
d. better pain relief than opioids .

22. about complete mole, all true except:


a. is tripleody .
b. no fetal tissue
c. risk of malignant sequel
d. safe to use OCP during follow up period
e. contraception during whole follow up .
23.about malignant GTN, all true except:
a. lung mets treated by radiotherapy
b. HCG is enough marker
c. treatment started after histopathological confirmation*(mostly.
d. chemotherapy sufficient cure .

24. diamniotic monochorionic twins, twin A has oligohydromnios, small size, twin B will have:
a. hypovolumia
b. reduced amniotic fluids
c. congestive heart failure .

25. single most imp. Factor in perinatal mortality in twins is:


a. prematurity .
b. cong. Anomalies .
c. growth restriction
d. abnormal lie .

26. 30 year , pregnant with twins, at term:


a. G2P2
b. G1P1
c. G1P1

27. relation of lowest bony part of presenting part to mother ischial spine:
a. fetal station .

28. when head is engaged, u can feel its lowest part at level of:
a. ischial spine (station 0 ) .
b. ischial tuberosity.

29. denominator of breech presentation:


a. sacrum .
b. coccyx
c. occiput
d. vertex .

30. false about mechanism of labor:


a. gynecoid pelvis bring vertex into occipititransverse while enter pelvic inlet .
b. shoulders should lie in AP diameter to be delivered into pelvic outlet .
c. flexion of fetus increase thru labor .

31. all indication for C/S, except:


a. transverse lie, cervix 3 cm dilated
b. placenta previa major degree
c. footling, ruptured membrane, 5cm dilated
d. cord presentation .
32. lab investigation not used for recurrent abortion:
a. thrombophilia scan
b. TORCH*( cause of abortion not of recurrent abortion) .
c.parents karyotyping

33. principle of management of septic abortion, include all except:


a. immediate evacuation of uterus*(I think bcz evacuation after 12 hrs of antibiotics.
b. IV antibiotics

34. 25 yrs, has missed abortion at 10 weeks, all should be discussed during counseling for post
abortion management, except:
a. Investigate cause of abortion
b. discuss most likely cause of abortion
c. contraception
d. avoid pregnancy for next 3 moths

35. not indication for anti D:


a. preterm labor*
b. cordocentesis
c. manual removal of placenta

36. about cervical incompetence, all true, except:


a. congenital cervical weakness is a cause
b. caused by previous multiple pregnancy .
c. diagnostic cone biopsy can cause it
d .tt by cervical cerclage in 13.15 weeks

37. not true about abortion:


a. cervical dilated in inevitable abortion
b. 20% of threatened abortion will progress to fetal loss .
c. US not essential for dx of incomplete
d. silent infx is most common cause .

38. during peuriperuim, one false:


a. external os stay open permanent
b. internal os close by 2nd day (by 1st week) .
c. lochia alba seen after 2nd week
d. lochia rubra seen in 1st few days

39. pregnant, 28 weeks, fundal height 22 weeks, all possible causes, except:
a. GDM .
b. placental insufficiency
c. wrong date
d. oligohydromnios .
40. 20 weeks, low lying placenta, ur action:
a. rescan weekly
b. rescan at 28 weeks .
c. plan for CS .

41. simple most imp. Factor in abruption placenta:


a. Pre.eclampsia
b. trauma
c. polyhydromnios
d. short cord .

42. about placenta previa, only one true:


a. increase risk of IUGR
b. increase risk of PPH*
c. abdominal US benefits but not accurate
d. vaginal exam safe .

43. all complication of progesterone mini pill, except:


a. breast tenderness
b. acne
c. irregular menstrual cycle
d. complex ovarian cyst (by exclusion) .

44. copper IUCD cause all, except:


a. dysmenorrhea
b. heavy menustruation
c. ectopic pregnancy
d. BV .

45. about sterilization, all true except:


a. sterilization is highly effective esp. in 1st 6.12 moths
b. tubal ligation done by rings or clips
c. antisperm ab is known complication
d. vas deference is …..(procedure of operation ) .

46. regarding lactation, all true except:


a. maintained by placental lactogen .
b. controlled by prolactin
c. suppressed by bromocripitine
d. human milk contain ptn less than cow milk .

47. tt for mild PID:


a. IV cephalosporin +gentamycin
b. IV cephalosporin+ IV metroniadazole
c. IV metroniadazole
d. doxycycline + metroniadazole orally .
48. definitive diagnosis of PID by:
a. laprascope .
b. U/S .
c. MRI .
d. hysteroscopy .

49. all correct about fibroids, except:


a. cervical fibroids are reassuring .
b. 3 types, subserosal, intramural, submucosa
c. subserosal is peduncleated
d. often are multiple

50. about fibroids & pregnancy, all true except:


a. hyaline degeneration is known complication& occur in 2nd trimester .

51. all radiological investigation for fibroid, except:


a. plain xray .
85
b. U/S .
c. MRI .

52. definitive diagnosis of endometriosis:


a. laprascopy .
b. lapratomy .
c. hysteroscopy .
d. histopathological .

53. all Sx of fibroids except :


a.urinary frequency
b.subfertility
c.menorrhagia
d.asymptomatic
e. All true .

54. about endometriosis , one is correct :


a. mostly present with menorrhagia
b. increase during pregnancy
c. tt affected by extent of disease*
d. no familial tendency .

55. One is correct about DUB


a. hypothyrodism is recognized cause
b. adenomyosis is cause
c. DUB occur in non ovulatory cycles
d. endometrial ablation used for tt .
56. menorrhagia can be treated by all except
a. mefanamic acid
b. GnRH analogue
c. progesterone
d. cupper IUCD .

57. lower abdominal pain in pregnant, all except:


a. cervical ecropion .
b. preterm labor
c. acute appendicitis
d. SCD
e. acute polyhydromnios .

58. about RH isoimmunization, all true except:


a. 500 IU antiD cover 5 ml blood .
b. liley’s chart & whitfeild action line are imp. In management .
c. all RH (.. people have “d” in each side of genotype
d. tends to be less severe in subsequent pregnancy .

59. all true for tt of anemia, except :


a. parentral iron to increase Hb rise*
b. S/E of oral ferrous is cause of reduced compliance
c. 0.4 mg daily folic acid is minimal requirement
d. iron increase level of Hb 0.8 mg every week .

60. all indication for delivery in PROM , except:


a. chorioamnionitis
b. fetal distress
c. abruptio placenta
d. multiple pregnancy .

61. causes of preterm, except:


a. abdominal surgery known trigger factor*
b. young maternal age
c. subclinical infection.

62 . C/I of ritodrine:
a. hyperthyroidism .
b. uterine atony .
c. GA 32 weeks .
d. ruptured membranes .
63. DM associated with all, except:
a. VSD
b. HTN .
c. fetal anal atresia
d. sacral agenesis is the commonest

64. hyperinsulinemia cause increase in fetal tissue, except:


a. heart
b. brain “not sure ‘ .
c. muscles
d. subcutaneous fat .

65. about HTN disorder of pregnancy, all true, except:


a. proteinuria & no HTN is hypertensive disorder of pregnancy .
b. proteinuria is 300 mg on 24 hr urine collection
c. proteinuria & HTN are imp. For dx of HELLP syndrome
d. hypotensive medication can't stop progression of pathology

66. about ANC, all true except:


a. reduce risk of hydrops fetalis
b. reduce risk of post term
c. urine dipstick routine every visit
e. pelvic exam is routine .

67. all correct about ANC, except:


a. air travel should be
b. rubella (–. lady should be immunized when deliver
c. exercise reduce need for induction
d. should take vit A .

68. all can be seen by US on 1st trimester, except:


a. placenta previa .
b. twin pregnancy .

69. about IUGR all true, except :


a. rare presentation of chromosomal defect .
b. caused by dieting in pregnancy .
c. caused by long standing DM .

70. about thyroid disorder in pregnancy, all true, except:


a. thyroid level is diagnostic*
b. clinical diagnosis is not significant
c. graves gets worse in postpartum
d. can cause preterm delivery
71. regarding epilepsy all true, except:
a. 3% risk of congenital anomaly*
b. 1/3 of pts have increased seizure frequency
c. monotherapy is gold standard
d. give folic acid .

72. all increase in DIC, except:


a. fibrin degradation product
b. PT
c. APTT
d. fibrinogen .

73. Patient with 5 cm simple cyst:


a. observe for 3 months to regress .
b. remove by laprascopy
c. lapratomy & cystectomy
d. TAHBSO .

74. a feto.protein is tumor marker of:


a. endodermal sinus tumor .

75. all prognostic factor for endometrial CA, except:


a. age
b. size
c. grade
d. presence or absence of fibroids .
ً‫ًإنًاألٌَىًرشرفع‬،ً‫ًٌَىهًثزسحًاٌخٍشًرثمش‬،ً‫ثزسحًاٌششًرٍٍج‬
ً‫ف‬
ً‫ًٌَىهًجزَسٌبًفًًاٌزشثخًلشٌجخًحزى‬،ً‫اٌفضبءًسشٌعًا‬
‫ٌزحجتًعه‬
ًً‫ًٌَىهًشجشحًاٌخٍشًرظًًف‬،ً‫شجشحًاٌخٍشًاٌىُسًَاٌٍُاء‬
‫ومٌُب‬
‫ًألنًعمكًجزَسٌبًفًًاٌزشثخًٌعُضٍبًعهًاٌذفء‬،ً‫اٌجطًء‬
ً‫َاٌٍُاء‬....
‫معًأوىبًحٍهًوزجبَصًاٌمظٍشًاٌمضَسًاٌجشاقًٌشجشحًاٌششً‬
‫َوفحص‬
‫عهًلُرٍبًاٌحمٍمٍخًَصٍجزٍبًرجذًٌَىبًَاٌىخًٌشخًوبفشخًفًً‬
‫غٍش‬
‫صٍجخًحمٍمٍخًعٍىًحٍهًرصجشًشجشحًاٌخٍشًعٍىًاٌجًءً‪،‬‬
‫َرزمبسهًٌٍعبصفخًَرظًًفًًومٌُبًاٌٍبدئًاٌجطًء‬
‫يًرحفًًثمبًرشجمٍبًثًًشجشحًاٌششًمهًألزاءًَأشُان‬ ‫__‬

‫‪Credits and gratitude to all dedicated members of Group C Medicomaniax.‬‬


‫‪We are sorry for any mistake done.‬‬

You might also like